Download as pdf or txt
Download as pdf or txt
You are on page 1of 143

NGUYỄN THÀNH NHÂN

(LỚP 10T1)

BÀI GIẢNG TOÁN CHUYÊN

HỌC KỲ I

10
ĐẠI SỐ

Năm học 2023-2024


MỤC LỤC

CHƯƠNG 1 Mệnh đề-Tập hợp 3

1 Mệnh đề 3

A Lý thuyết cần nắm 3

1 Mệnh đề là gì? 3

2 Các phép toán về mệnh đề 3

3 Các luật mệnh đề 5

B Ví dụ giải toán 5

C Bài tập 5

2 Mệnh đề chứa biến 5

TOÁN CHUYÊN
A Lý thuyết cần nắm 6

1 Mệnh đề chứa biến 6

2 Lượng từ "với mọi" và lượng từ "tồn tại" 6

3 Phủ định của mệnh đề chứa lượng từ " với mọi" và lượng từ "tồn tại" 8

B Bài tập 8

3 Áp dụng mệnh đề vào suy luận toán học 9

A Lý thuyết cần nắm 9

1 Diễn đạt một định lý 9

2 Điều kiện cần, điều kiện đủ, điều kiện cần và đủ 9

3 Phương pháp chứng minh trực tiếp và gián tiếp (phương pháp phản chứng) 10

4 Phương pháp quy nạp toán học 10

A Cơ sở lý thuyết 10

1 Phương pháp quy nạp toán học 10

2 Phương pháp quy nạp tổng quát 13

3 Phương pháp quy nạp mạnh 13


Các Bài Giảng 2023-2024 Đại Số 10 Chuyên

5 Chuyên đề: Phương pháp chứng minh phản chứng 15

A Cơ sở lý thuyết 15

1 Phương pháp chứng minh bằng phản chứng) 15

2 Bài tập áp dụng 16

4 Tập hợp 17

A Lý thuyết cần nắm 17

1 Tập hợp 17

2 Các phép toán trên tập hợp 19

3 Công thức bao hàm loại trừ 20

B Bài tập 20
LATEX:Nguyễn Thành Nhân

CHƯƠNG 2 Bất phương trình, hệ bất phương trình bậc nhất hai ẩn 27

1 Bất phương trình và hệ bất phương trình bậc nhất hai ẩn 27

A Cơ sở lý thuyết 27

1 Bất phương trình bậc nhất hai ẩn 27

2 Hệ bất phương trình bậc nhất hai ẩn 27

3 Áp dụng vào bài toán kinh tế 28

B Bài tập 28

CHƯƠNG 3 Hàm số bậc hai và đồ thị 33

1 Khái niệm hàm số 33

A Lý thuyết cần nắm 33

1 Định nghĩa 33

2 Các phương pháp biểu diễn hàm số 33

3 Đồ thị của hàm số và phép tịnh tiến đồ thị 33

4 Tập xác định của hàm số cho bởi công thức 34

5 Sự đồng biến nghịch biến của hàm số 34

6 Hàm số chẵn, hàm số lẻ 35

7 Hàm số hợp 35

ÓNguyễn Thành Nhân Chuyên Hùng Vương-BD


Các Bài Giảng 2023-2024 Đại Số 10 Chuyên

2 Hàm số bậc nhất 36

A Cơ sở lý thuyết 36

1 1. Nhắc lại về hàm số bậc nhất 36

2 Hàm số y = |ax + b| 37

3 Một tính chất đặc trưng của hàm số bậc nhất (phần mở rộng) 37

B Bài tập luyện tập 37

3 HÀM SỐ BẬC HAI 38

A Cơ sở lý thuyết 38

1 Định nghĩa 38

2 Chiều biến thiên của hàm số bậc hai 39

3 Đồ thị của hàm số bậc hai 39

TOÁN CHUYÊN
4 Một vài lưu ý 40

5 Một tính chất đặc trưng của hàm số bậc hai 40

B Bài tập 40

4 HÀM SỐ BẬC HAI-BÀI TẬP 43

A Cơ sở lý thuyết 43

B Bài tập 43

CHƯƠNG 4 Các chuyên đề chuyên toán 53

1 Ánh xạ 53

A Lý thuyết cần nắm 53

1 Ánh xạ là gì? 53

2 Đơn ánh, toàn ánh, song ánh. 54

3 Ánh xạ ngược của một song ánh 54

4 Ánh xạ hợp 54

B Các dạng toán 55

1 Ứng dụng vào bài toán đếm 55

2 Phương trình hàm 55

ÓNguyễn Thành Nhân Chuyên Hùng Vương-BD


Các Bài Giảng 2023-2024 Đại Số 10 Chuyên

2 Luyện tập 56

A Các kỹ thuật cơ bản giải phương trình hàm 56

B Bài tập luyện tập 57

1 Phương pháp thế biến 57

2 Phương pháp thêm biến 57

3 Phương pháp quy nạp 58

4 Phương pháp sử dụng tính đơn ánh, toàn ánh, song ánh 61

3 Ôn tập chương I 66

A Cơ sở lý thuyết 66

1 Phương pháp sử dụng điểm bất động 66


LATEX:Nguyễn Thành Nhân

2 Lớp hàm liên tục (phần tham khảo thêm đối với lớp 10) 68

4 KĨ THUẬT THẾ GIÁ TRỊ ĐẶC GIẢI PHƯƠNG TRÌNH HÀM 72

A Lý thuyết cần nắm 72

5 Giải phương trình hàm bằng phương pháp quy nạp 72

1 Phương pháp quy nạp 73

2 Bài tập 74

6 Giải phương trình hàm bằng phương pháp sử dụng tính đơn ánh-toàn ánh-song ánh 75

CHƯƠNG 5 BẤT ĐẲNG THỨC 79

1 Bất đẳng thức Cauchy 79

A Cơ sở lý thuyết 79

1 Bất đẳng thức Cauchy cho trường hợp hai biến 79

2 Bất đẳng thức Cauchy cho trường hợp ba biến 79

3 Bất đẳng thức Cauchy cho trường hợp tổng quát 80

4 Các kết quả thu được từ bất đẳng thức Cauchy 80

5 Một số kỹ thuật căn bản trong áp dụng bất đẳng thức Cauchy 80

2 Bất đẳng thức Cauchy-Bài tập 83

A BÀI TẬP 83

ÓNguyễn Thành Nhân Chuyên Hùng Vương-BD


Các Bài Giảng 2023-2024 Đại Số 10 Chuyên

3 Bất đẳng thức Cauchy-Schwarz 84

A Cơ sở lý thuyết 85

1 Đẳng thức Lagrange 85

2 Bất đẳng thức Cauchy-Schwarz 85

3 Bất đẳng thức Cauchy-Schwarz dạng Engle 85

B Bài tập 86

1 Qua các đề thi 87

4 Bất đẳng thức Holder và Bất đẳng thức Abel 87

A Bất đẳng thức Holder 87

1 Bất đẳng thức Holder cho hai cặp số dương 88

2 Bất đẳng thức Holder cho hai bộ ba số dương 88

3 Bất đẳng thức Holder cho m bộ, mỗi bộ n số dương 88

TOÁN CHUYÊN
4 Một số kết quả 88

B Bài tập 89

C Khai triển Abel và ứng dụng 89

1 Khai triển Abel 90

2 Bất đẳng thức Abel 90

3 Ứng dụng khai triển Abel 90

5 Bất đẳng thức hoán vị- Bất đẳng thức Chebyshev 91

A Cơ sở lý thuyết 91

1 Bất đẳng thức hoán vị 91

2 Bất đẳng thức Chebyshev 91

B Bài tẬp 92

6 Bất đẳng thức Schur 93

A Cơ sở lý thuyết 93

1 Bất đẳng thức Schur dạng thông thường 93

2 Bất đẳng thức Schur dạng mở rộng 94

3 Kỹ thuật p, q, r trong bất đẳng thức Schur 94

ÓNguyễn Thành Nhân Chuyên Hùng Vương-BD


Các Bài Giảng 2023-2024 Đại Số 10 Chuyên

B Bài tập áp dụng 94

C Bài tập 95

7 Các kỹ thuật căn bản chứng minh bất đẳng thức 96

A Cơ sở lý thuyết 96

1 Kỹ thuật chuẩn hóa 96

2 Kỹ thuật đánh giá đại diện 97

3 Kỹ thuật hệ số bất định 97

4 Sử dụng phần tử cực hạn 97

5 Kỹ thuật tiếp tuyến hóa 98

B Bài tập 98
LATEX:Nguyễn Thành Nhân

CHƯƠNG 6 Phương Trình-Hệ Phương Trình 101

1 Đại cương về phương trình, bất phương trình 101

A Cơ sở lý thuyết 101

1 Khái niệm về phương trình một ẩn 101

2 Phương trình tương đương 101

3 Phương trình hệ quả 101

4 Phương trình nhiều ẩn 102

5 Phương trình có chứa tham số 102

2 Phương trình, bất phương trình bậc nhất và bậc hai 102

A Cơ sở lý thuyết 102

1 Giải và biện luận phương trình dạng ax + b = 0 102

2 Giải và biện luận bất phương trình dạng ax + b ≥ 0 102

3 Dấu của nhị thức bậc nhất 103

4 Giải và biện luận phương trình dạng ax2 + bx + c = 0 103

5 Dấu của tam thức bậc hai 103

6 Định lý đảo về dấu của tam thức bậc hai 103

7 Các hệ quả 103

8 Các dạng toán thường gặp 104

ÓNguyễn Thành Nhân Chuyên Hùng Vương-BD


Các Bài Giảng 2023-2024 Đại Số 10 Chuyên

9 Một số kết quả thường dùng 105

3 Bài tập 106

A Bài tập 106

4 Phương trình đa thức bậc cao đặc biệt 107

A Cơ sở lý thuyết 107

1 Phương trình dạng hồi quy 107

2 Phân tích thành các nhân tử bậc hai 107

3 Phương trình lũy thừa bậc 4 108

4 Phương trình ghép cặp 108

5 Phương trình đẳng cấp theo hai biến 108

B Bài tập tổng hợp 109

TOÁN CHUYÊN
5 Bài tập tam thức bậc hai 109

A Bài tập 109

6 Phương trình, bất phương trình chứa căn 110

A Cơ sở lý thuyết 111

1 Một số phép biến đổi tương đương phương trình chứa căn 111

2 Một số phép biến đổi tương đương bất phương trình chứa căn 112

3 Phương trình, bất phương trình chứa căn nâng cao 112

7 Tổng hợp về phương trình vô tỉ 113

1 Bài tập tam thức bậc hai 114

A Bài tập 114

2 Hệ phương trình bậc nhất hai ẩn, ba ẩn 115

A Cơ sở lý thuyết 115

1 Hệ hai phương trình bậc nhất hai ẩn 115

2 Hệ ba phương trình bậc nhất ba ẩn 116

B Bài tập 117

ÓNguyễn Thành Nhân Chuyên Hùng Vương-BD


Các Bài Giảng 2023-2024 Đại Số 10 Chuyên

3 Hệ phương trình bậc cao mẫu mực 118

A Cơ sở lý thuyết 118

1 Hệ đối xứng kiểu I 118

2 Hệ đối xứng kiểu II 120

3 Hệ đẳng cấp 121

4 Hệ phương trình bậc cao không mẫu mực 122

A Cơ sở lý thuyết 122

1 Kỹ thuật sử dụng phép thế 122

2 Kỹ thuật biến đổi tương đương 123


LATEX:Nguyễn Thành Nhân

3 Kỹ thuật đặt ẩn số phụ 123

4 Kỹ thuật đưa về phương trình tích 124

5 Kỹ thuật sử dụng tính đơn điệu của hàm số 124

5 PHƯƠNG TRÌNH-HỆ PHƯƠNG TRÌNH GIẢI BẰNG PHƯƠNG PHÁP ĐÁNH GIÁ 125

A Cơ sở lý thuyết 125

1 Các tình huống thường gặp 125

2 Các bất đẳng thức thường sử dụng 125

B Các bài toán 126

1 Các bài toán phương trình 126

2 Các bài toán hệ phương trình 126

6 Hệ ba phương trình ba ẩn qua các đề Olympic 128

A Các dạng toán 128

1 Hệ hoán vị vòng quanh 128

2 Sử dụng phép cộng đại số 130

3 Giải hệ dựa vào đánh giá 130

4 Hệ có tham số 131

B Bài tập rèn luyện 131

ÓNguyễn Thành Nhân Chuyên Hùng Vương-BD


Các Bài Giảng 2023-2024 Đại Số 10 Chuyên

7 Ôn tập phương trình- Hệ phương trình 132

A MỘT SỐ DẠNG TOÁN 132

1 Đặt ẩn số phụ cho biểu thức đồng dạng 132

2 Đặt ẩn phụ cho phương trình đối xứng 133

3 Đặt ẩn phụ cho phương trình đẳng cấp 133

4 Đặt hai ẩn phụ đưa về phương trình tích 133

5 Sử dụng biểu thức liên hợp 134

TOÁN CHUYÊN

ÓNguyễn Thành Nhân 1 Chuyên Hùng Vương-BD


LATEX:Nguyễn Thành Nhân Các Bài Giảng 2023-2024 Đại Số 10 Chuyên

ÓNguyễn Thành Nhân 2 Chuyên Hùng Vương-BD


CHƯƠNG 1 MỆNH ĐỀ-TẬP HỢP

BÀI 1. MỆNH ĐỀ
Nguyễn Thành Nhân- Chuyên Hùng Vương- Bình Dương

A. LÝ THUYẾT CẦN NẮM

1. Mệnh đề là gì?
Định nghĩa 1. Một mệnh đề lôgic (gọi tắt là mệnh đề)là một câu khẳng định đúng hoặc một câu khẳng định
sai.
• Một câu khẳng định đúng gọi là một mệnh đề đúng.
• Một câu khẳng định sai gọi là một mệnh đề sai.
• Một mệnh đề không thể vừa đúng vừa sai.

VÍ DỤ 1. Lấy ba ví dụ khác nhau về mệnh đề.

TOÁN CHUYÊN
VÍ DỤ 2. Lấy hai ví dụ khác nhau về các câu không phải là mệnh đề.

Lưu ý: Người ta gán cho mệnh đề đúng giá trị chân lý là 1, mệnh đề sai giá trị chân lý là 0.

2. Các phép toán về mệnh đề

a) Phép phủ định và mệnh đề phủ định


Định nghĩa 2. Cho mệnh đề P . Mệnh đề: " Không phải P " được gọi là mệnh đề phủ định của mệnh đề P và
được ký hiệu là P .
P 1 0
• Bảng chân trị:
P 0 1

VÍ DỤ 3. Lấy ví dụ về mệnh đề P và tìm mệnh đề phủ định P .

b) Phép hội và mệnh đề hội


Định nghĩa 3. Cho hai mệnh đề P và Q . Mệnh đề "P và Q " được gọi là mệnh đề hội của hai mệnh đề P và Q
và được ký hiệu là P ∧ Q .
Phép toán ∧ được gọi là phép hội.
• Mệnh đề "P và Q " chỉ đúng khi cả P và Q cùng đúng. Sai trong các trường hợp còn lại.
P Q P ∧Q
1 1 1
• Bảng chân trị: 1 0 0
0 1 0
0 0 0

VÍ DỤ 4. Cho P : "100 chia hết cho 5" và Q : "100 chia hết cho 2".
Khi đó mệnh đề P ∧ Q : "100 chia hết cho 10".

3
Các Bài Giảng 2023-2024 Đại Số 10 Chuyên

c) Phép tuyển và mệnh đề tuyển


Định nghĩa 4. Cho hai mệnh đề P và Q . Mệnh đề "P hoặc Q " được gọi là mệnh đề tuyển của hai mệnh đề P
và Q và được ký hiệu là P ∨ Q .
Phép toán ∨ được gọi là phép tuyển.
• Mệnh đề "P hoặc Q " chỉ sai khi cả P và Q cùng sai. Đúng trong các trường hợp còn lại.
P Q P Q
W

1 1 1
• Bảng chân trị: 1 0 1
0 1 1
0 0 0

VÍ DỤ 5. Lấy một ví dụ về mệnh đề tuyển.

d) Mệnh đề kéo theo và phép kéo theo


Định nghĩa 5. Cho hai mệnh đề P và Q . Mệnh đề " Nếu P thì Q " được gọi là mệnh đề kéo theo của hai mệnh
đề P và Q và được ký hiệu là P ⇒ Q .
LATEX:Nguyễn Thành Nhân

Phép toán ⇒ được gọi là phép kéo theo.


• Mệnh đề "P kéo theo Q " chỉ sai khi P đúng và Q sai. Đúng trong các trường hợp còn lại.
P Q P ⇒Q
1 1 1
• Bảng chân trị: 1 0 0
0 1 1
0 0 1

• Diễn đạt mệnh đề kéo theo P ⇒ Q bằng một trong các cách:
+ "Nếu P thì Q "
+ "Vì P nên Q "
+ "P kéo theo Q "
+ "P là điều kiện đủ để có Q "
+ "Q là điều kiện cần để có P "

Chú ý 1.
• Mệnh đề P ⇒ Q có nghĩa rộng lớn, không bắt buộc các mệnh đề thành phần P và Q phải có liên quan với
nhau.
• Mệnh đề kéo theo P ⇒ Q đúng không bắt buộc các mệnh đề thành phần P và Q phải đúng.

e) Mệnh đề đảo, mệnh đề phản và mệnh đề phản đảo


Định nghĩa 6. Cho mệnh đề kéo theo
P ⇒Q (1)

• Mệnh đề Q ⇒ P được gọi là mệnh đề đảo của (1).


• Mệnh đề P ⇒ Q được gọi là mệnh đề phản của (1).
• Mệnh đề Q ⇒ P được gọi là mệnh đề phản đảo của (1).

VÍ DỤ 6. Cho mệnh đề kéo theo P ⇒ Q : "Nếu bạn chăm chỉ thì bạn học tốt". Hãy lập các mệnh đề đảo,
mệnh đề phản và mệnh đề phản đảo.

VÍ DỤ 7. Hãy lập bảng chân trị của hai mệnh đề P ⇒ Q và Q ⇒ P . Từ đó rút ra nhận xét về chân trị
của hai mệnh đề đó khi mà giá trị chân lý của các mệnh đề thành phần giống nhau.

ÓNguyễn Thành Nhân 4 Chuyên Hùng Vương-BD


Các Bài Giảng 2023-2024 Đại Số 10 Chuyên

f) Mệnh đề tương đương


Định nghĩa 7. Cho hai mệnh đề P và Q . Mệnh đề " P nếu và chỉ nếu Q " gọi là mệnh đề tương đương và được
ký hiệu là P ⇔ Q .
• Phép toán ⇔ được gọi là phép tương đương.
• Mệnh đề P ⇔ Q đúng nếu cả hai mệnh đề P ⇒ Q và Q ⇒ P đều đúng. Tức là hai mệnh đề P và Q cùng đúng
hoặc cùng sai.
P Q P ⇔Q
1 1 1
• Bảng chân trị: 1 0 0
0 1 0
0 0 1

VÍ DỤ 8. Cho mệnh đề P : "Tứ giác ABCD là hình chữ nhật" và Q : "Tứ giác ABCD là hình bình hành có một góc vuô
Lập mệnh đề P ⇔ Q .

g) Sự tương đương và chứng minh sự tương đương của hai mệnh đề


Định nghĩa 8. Nếu mệnh đề P ⇔ Q là một mệnh đề đúng thì ta nói P và Q tương đương lôgic. Ta viết P = Q .

3. Các luật mệnh đề

TOÁN CHUYÊN
Định lí 1. Cho P , Q , R là các mệnh đề. Khi đó

1 Luật giao hoán: P ∨ Q = Q ∨ P ; P ∧ Q = Q ∧ P .

2 Luật kết hợp: (P ∨ Q ) ∨ R = P ∨ (Q ∨ R ); (P ∧ Q ) ∧ R = P ∧ (Q ∧ R ).

3 Luật phân phối: P ∨ (Q ∧ R ) = (P ∨ Q ) ∧ (P ∨ R ); P ∧ (Q ∨ R ) = (P ∧ Q ) ∨ (P ∧ R ).

4 Quy tắc De Morgan: P ∨ Q = P ∧ Q ; P ∧ Q = P ∨ Q .

B. VÍ DỤ GIẢI TOÁN
BÀI 1. Cho hai mệnh đề P và Q . Chứng minh rằng

1 P ⇒ Q = P ∨ Q. 2 Q ⇒ P = P ⇒ Q.

BÀI 2. Cho hai mệnh đề P và Q . Xét mệnh đề "P ⇒ Q ". Trong các mệnh đề sau đây, mệnh đề nào tương đương
với mệnh đề trên

1 P ∨ Q. 2 Q ⇒ P. 3 P ⇒ Q. 4 Q ⇒ P.

BÀI 3. Cho hai mệnh đề P và Q . Chứng minh rằng

1 P = P ∨ ( P ∧ Q ). 2 P = P ∧ (P ∨ Q ).

BÀI 4. Cho hai mệnh đề P và Q . Chứng minh rằng

P ⇔ Q = P ⇔ Q.

C. BÀI TẬP

BÀI 2. MỆNH ĐỀ CHỨA BIẾN


Nguyễn Thành Nhân- Chuyên Hùng Vương- Bình Dương

ÓNguyễn Thành Nhân 5 Chuyên Hùng Vương-BD


Các Bài Giảng 2023-2024 Đại Số 10 Chuyên

A. LÝ THUYẾT CẦN NẮM


1. Mệnh đề chứa biến
Định nghĩa 1. Mệnh đề chứa biến (biến x) là những câu có dạng "P ( x) với x ∈ X "
• Tính đúng sai của mệnh đề chứa biến "P ( x) với x ∈ X " còn phụ thuộc vào giá trị x0 mà biến x nhận.
n o
• Mệnh đề chứa biến "P ( x) với x ∈ X " còn được gọi là hàm mệnh đề với tập xác định là X và tập giá trị là 0; 1 .

• Ta cũng gặp những mệnh đề chứa nhiều biến x, y, z . . .. Tổng quát

"P ( x1 , x2 , . . . , xn ) với x i ∈ X i i = 1, n" (1)

2. Lượng từ "với mọi" và lượng từ "tồn tại"


a) Lượng từ ∀ (đọc là "với mọi")
Để diễn đạt câu sau
"Với mọi x ∈ X , P ( x) đúng" (2)
ta sử dụng ký hiệu sau
"∀ x ∈ X , P ( x)" hoặc "∀ x ∈ X : P ( x)" hoặc "P ( x), ∀ x ∈ X ".
LATEX:Nguyễn Thành Nhân

• Mệnh đề chứa lượng từ " với mọi" đúng trong trường hợp nào? Sai trong trường hợp nào?

VÍ DỤ 1. Lấy một số ví dụ có sử dụng lượng từ ∀.

VÍ DỤ 2. Cho mệnh đề chứa biến :" Bình phương của mọi số nguyên tố lớn hơn 2 khi chia cho 3 đều có
số dư là 1 ".
1 Sử dụng lượng từ ∀ để viết lại mệnh đề trên.

2 Xét tính đúng sai của mệnh đề trên.

VÍ DỤ 3. Xét mệnh đề chứa biến: "Bình phương của tổng ba số thực bất không lớn hơn ba lần tổng
bình phương của ba số thực đó ".
1 Sử dụng lượng từ ∀ để viết lại mệnh đề trên.

2 Xét tính đúng sai của mệnh đề trên.

VÍ DỤ 4. Xét mệnh đề chứa biến: " Bình phương của mọi số chính phương thì chia hết cho 4 hoặc chia
cho 4 dư 1 ".
1 Sử dụng lượng từ ∀ để viết lại mệnh đề trên.

2 Xét tính đúng sai của mệnh đề trên.

VÍ DỤ 5. Xét mệnh đề chứa biến: " Trung bình cộng của n số thực không âm không bé hơn trung bình
nhân của n số thực đó".
1 Sử dụng lượng từ ∀ để viết lại mệnh đề trên.

2 Viết lại mệnh đề trên bằng ký hiệu sử dụng lượng từ ∀ cho hai trường hợp n = 2 và n = 3.

3 Xét tính đúng sai của mệnh đề trên.

VÍ DỤ 6. Xét mệnh đề chứa biến: " Hiệu lũy thừa bậc 7 của một số nguyên với số đó luôn chia hết cho
42 ".

1 Sử dụng lượng từ ∀ để viết lại mệnh đề trên.

ÓNguyễn Thành Nhân 6 Chuyên Hùng Vương-BD


Các Bài Giảng 2023-2024 Đại Số 10 Chuyên

2 Xét tính đúng sai của mệnh đề trên.

3 Sử dụng kết quả trên, hãy chứng minh rằng với mọi số nguyên dương n, ta có
¢7 ¡ ¢7 ¡ ¢7
(27 n + 5)7 + 10 + (10 n + 27)7 + 5 + (5 n + 10)7 + 27 chia hết cho 42.
¡

(Chuyên KHTN Hà Nội 2019).

VÍ DỤ 7 (Giả thuyết Goldbach (Đức-1690-1764)).


Cho mệnh đề chứa biến: "Mỗi số tự nhiên chẵn lớn hơn 2 đều có thể biểu diễn bằng tổng của hai số
nguyên tố".

1 Sử dụng lượng từ ∀ để viết lại mệnh đề trên.

2 Giả thuyết Goldbach đã chứng minh được chưa?

VÍ DỤ 8 (Định lý Fermat lớn (Pháp-1601-1665)).


Cho mệnh đề chứa biến: "Không tồn tại các số nguyên khác không x, y, z thỏa mãn phương trình

x n + yn = z z ,

trong đó n là một số nguyên lớn hơn 2 ".

1 Sử dụng lượng từ ∀ để viết lại mệnh đề trên.

TOÁN CHUYÊN
2 Định lý Fermat lớn đã chứng minh được chưa?

VÍ DỤ 9 (Chuyên Sư Phạm Hà Nội 2016). Chứng minh rằng biểu thức sau nhận giá trị nguyên dương
với mọi giá trị nguyên dương n:
·q p ¸q p
P= n2 + ( n + 1)2 + 2
( n − 1) + n 2 4 n2 + 2 − 2 4 n4 + 1.

b) Lượng từ ∃ (đọc là "tồn tại")


Cho mệnh đề chứa biến "P ( x) với x ∈ X ". Khi đó câu khẳng định

"Tồn tại phần tử x thuộc X sao cho P ( x) đúng (2)

được ký hiệu là
"∃ x ∈ X , P ( x)" hoặc "∃ x ∈ X : P ( x)".

• Ký hiếu ∃ đọc là: Tồn tại, tồn tại ít nhất, có ít nhất.

• Mệnh đề chứa lượng từ " tồn tại" đúng trong trường hợp nào? Sai trong trường hơp nào?

VÍ DỤ 10. Lấy một số ví dụ có sử dụng lượng từ " tồn tại".

VÍ DỤ 11. Cho mệnh đề chứa biến: "Tồn tại một số nguyên mà lũy thừa bậc 2019 của nó bằng chính
nó."
1 Sử dụng lượng từ ∃ để viết lại mệnh đề trên.

2 Xét tính đúng sai của mệnh đề trên.

ÓNguyễn Thành Nhân 7 Chuyên Hùng Vương-BD


Các Bài Giảng 2023-2024 Đại Số 10 Chuyên

c) Lượng từ "với mọi" và lượng từ "tồn tại" cho trường hợp nhiều biến
Ta gặp các mệnh đề có dạng

1 "∀ x ∈ X , ∀ y ∈ Y : P ( x; y)". 2 "∃ x ∈ X , ∃ y ∈ Y : P ( x; y)".

3 "∃ x ∈ X , ∀ y ∈ Y : P ( x; y)". 4 "∀ x ∈ X , ∃ y ∈ Y : P ( x; y)".

VÍ DỤ 12 (Chuyên KHTN Hà Nội 2017). Với a, b là các số dương thỏa mãn ab + a + b = 1, chứng minh
rằng
a b 1 + ab
+ =p .
1 + a2 1 + b 2 2(1 + a2 )(1 + b2 )

VÍ DỤ 13. Biết rằng x, y là các số thực thỏa mãn


³ p ´ µ q ¶
x+ x2 + 2020 2
· y + y + 2020 = 2020.

Chứng minh rằng x + y = 0.


LATEX:Nguyễn Thành Nhân

3. Phủ định của mệnh đề chứa lượng từ " với mọi" và lượng từ "tồn tại"
Cho mệnh đề chứa biến P ( x) với x ∈ X . Khi đó

1 Phủ định của mệnh đề "∀ x ∈ X , P ( x)" là mệnh đề "∃ x ∈ X , P ( x)".

2 Phủ định của mệnh đề "∃ x ∈ X , P ( x)" là mệnh đề "∀ x ∈ X , P ( x)".

Đối với trường hợp hai biến, ta có

1 Phủ định của mênh đề "∀ x ∈ X , ∀ y ∈ Y : P ( x, y)" là mệnh đề "∃ x ∈ X , ∃ y ∈ Y : P ( x, y)".

2 Phủ định của mênh đề "∃ x ∈ X , ∃ y ∈ Y : P ( x, y)" là mệnh đề "∀ x ∈ X , ∀ y ∈ Y : P ( x, y)".

3 Phủ định của mênh đề "∀ x ∈ X , ∃ y ∈ Y : P ( x, y)" là mệnh đề "∃ x ∈ X , ∀ y ∈ Y : P ( x, y)".

4 Phủ định của mênh đề "∃ x ∈ X , ∀ y ∈ Y : P ( x, y)" là mệnh đề "∀ x ∈ X , ∃ y ∈ Y : P ( x, y)".

VÍ DỤ 14. Lấy một số ví dụ minh họa ch lý thuyết vừa nêu.

B. BÀI TẬP
BÀI 1. Cho a, b, c là các số nguyên. Chứng minh rằng

a3 + b3 + c3 chia hết cho 6 khi và chỉ khi a + b + c chia hết cho 6.

BÀI 2. Cho các số thực a, b, c thỏa mãn a + b + c = 0. Chứng minh đẳng thức
¢2
a2 + b 2 + c 2 = 2 a4 + b 4 + c 4 .
¡ ¡ ¢

BÀI 3. Cho các số thực a, b, c thỏa mãn đồng thời hai đẳng thức sau

i) (a + b) ( b + c) ( c + a) = abc.

ii) a3 + b3 c3 + b3 a3 + c3 = a3 b3 c3 .
¡ ¢¡ ¢¡ ¢

Chứng minh rằng abc = 0.


BÀI 4. Với mỗi số nguyên dương n, kí hiệu S n là tổng n số nguyên tố đầu tiên (S1 = 2; S2 = 2+3; S3 = 2+3+5, · · · ).
Chứng minh rằng trong dãy số S1 ; S2 ; S3 ; · · · không tồn tại hai số hạng liên tiếp đều là số chính phương.
2 · 6 · 10 . . . (4 n − 2)
BÀI 5. Chứng minh rằng với mỗi số nguyên n ≥ 6 thì số a n = 1 + là một số chính phương.
( n + 5)( n + 6) · · · (2 n)

ÓNguyễn Thành Nhân 8 Chuyên Hùng Vương-BD


Các Bài Giảng 2023-2024 Đại Số 10 Chuyên

BÀI 6. Cho hai số nguyên a, b thỏa mãn a2 + b2 + 1 = 2(ab + a + b). Chứng minh a và b là hai số chính phương
liên tiếp.
BÀI 7. Cho a, b, c là các số nguyên. Chứng minh rằng

a3 + b3 + c3 chia hết cho 6 khi và chỉ khi a + b + c chia hết cho 6.

BÀI 8. Cho p là số nguyên tố có dạng 4 k + 3 ( k ∈ N). Giả sử x, y là các số nguyên thỏa mãn x2 + y2 chia hết cho
p. Chứng minh x và y đều chia hết cho p.
BÀI 9. Chứng minh rằng các số n(n + 1) và n(n + 2) không thể là các số chính phương với mọi số nguyên dương
n.
1 1
BÀI 10. Giả sử có x ∈ R sao cho x + ∈ Z. Chứng minh x n + n ∈ Z, ∀ n ∈ N∗ .
x x
BÀI 11.
( a + b )2
1 Cho biểu thức M = , với a, b là hai số nguyên dương phân biệt. Chứng minh rằng M
a3 + ab2 − a2 b − b3
không thể nhận giá trị nguyên.

2 Cho a, b là hai số nguyên dương, đặt A = (a + b)2 − 2a2 , B = (a + b)2 − 2 b2 . Chứng minh rằng A và B không
đồng thời là số chính phương.

BÀI 12. Cho ba số nguyên dương a, b, c thỏa mãn a3 + b3 + c3 chia hết cho 14. Chứng minh abc cũng chia hết
cho 14.

TOÁN CHUYÊN
BÀI 3. ÁP DỤNG MỆNH ĐỀ VÀO SUY LUẬN TOÁN HỌC
Nguyễn Thành Nhân- Chuyên Hùng Vương- Bình Dương

A. LÝ THUYẾT CẦN NẮM

1. Diễn đạt một định lý


Mỗi định lý là một mệnh đề đúng.
Chúng ta có thể diễn đạt một định lý bằng một trong các cách sau

1 "∀ x ∈ X , P ( x)". 2 "∃ x ∈ X , P ( x)".

3 "∀ x ∈ X , P ( x) ⇒ Q ( x)". 4 "∀ x ∈ X , P ( x) ⇔ Q ( x)".

Với trường hợp nhiều biến ta có thể phát biểu tương tự.

2. Điều kiện cần, điều kiện đủ, điều kiện cần và đủ


Ta xét các định lý sau

1 "∀ x ∈ X , P ( x) ⇒ Q ( x)" (1).


P ( x) gọi là giả thiết, Q ( x) là kết luận.
P ( x) là điều kiện đủ để có Q ( x), Q ( x) là điều kiện cần để có P ( x).

2 "∀ x ∈ X ,Q ( x) ⇒ P ( x)" (2).


Nếu mệnh đề (2) đúng thì định lý (2) được gọi là định lý đảo của định lý (1). Định lý (1) được gọi là định
lý thuận.

3 Định lý thuận và định lý đảo ta có thể viết gộp thành

∀ x ∈ X , P ( x) ⇔ Q ( x) (3).

Ta nói P ( x) là điều kiện cần và đủ để có Q ( x).


P ( x) khi và chỉ khi Q ( x).
P ( x) nếu và chỉ nếu Q ( x).

ÓNguyễn Thành Nhân 9 Chuyên Hùng Vương-BD


Các Bài Giảng 2023-2024 Đại Số 10 Chuyên

3. Phương pháp chứng minh trực tiếp và gián tiếp (phương pháp phản chứng)
Ta xét tình huống cụ thể cần chứng minh định lý

∀ x ∈ X , P ( x) ⇒ Q ( x) (4)

• Chứng minh định lý (4) tức là ta dùng suy luận toán học để chỉ ra (4) là mệnh đề đúng.

1 Phương pháp trực tiếp:


+ Ta lấy x0 tùy ý thuộc X mà P ( x0 ) đúng.
+ Dùng suy luận toán học để suy ra Q ( x0 ) đúng.
+ Từ đó suy ra mệnh đề P ( x) ⇒ Q ( x) đúng với mọi x.
2 Phương pháp gián tiếp:
+ Để chứng minh (4) đúng thì ta chứng minh phủ định của (4) là mệnh đề sai, tức là mệnh đề sau sai.

∃ x ∈ X , P ( x) ⇒ Q ( x) (5)

Tuy nhiên mệnh đề (5) là tương đương logic với mệnh đề

∃ x ∈ X , P ( x) ∧ Q ( x) (6)

Chứng minh (6) sai ta làm như sau:


+ Giả sử x0 ∈ X mà P ( x0 ) đúng và Q ( x0 ) sai.
LATEX:Nguyễn Thành Nhân

+ Dùng suy luận toán học để chỉ ra mâu thuẫn. Dẫn đến (6) sai. Do đó (4) đúng.

VÍ DỤ 1. Chứng minh rằng với mọi số nguyên n > 1 thì n có một ược nguyên tố.

VÍ DỤ 2. Chứng minh rằng tập hợp các số nguyên tố là vô hạn.

VÍ DỤ 3. Chứng minh rằng P (a) = a5 + 4a3 + 3a và Q (a) = a4 + 3a2 + 1 là hai số nguyên tố cùng nhau với
mọi số nguyên a.

VÍ DỤ 4. Cho p là một số nguyên tố. Chứng minh rằng mọi ước nguyên tố của 2 p − 1 đều lớn hơn p.

BÀI 4. PHƯƠNG PHÁP QUY NẠP TOÁN HỌC


Nguyễn Thành Nhân- Chuyên Hùng Vương- Bình Dương

A. CƠ SỞ LÝ THUYẾT
1. Phương pháp quy nạp toán học
Ta chia phương pháp quy nạp ra làm hai trường hợp
1 Quy nạp thông thường
Đặt vấn đề cần chứng minh định lý
∀ n ∈ N∗ , P ( n) (7)
Ta chứng minh định lý bằng phương pháp quy nạp toán học như sau
• Bước 1 (Bước cơ sở): Chỉ ra P (1) là mệnh đề đúng.
• Bước 2 (Bước quy nạp): Giả sử mệnh đề đúng với n = k ( k ≥ 1), ta cần chứng minh mệnh đề cũng
đúng khi n = k + 1.
Theo nguyên lý quy nạp, mệnh đề đúng với mọi n ∈ N∗ .
Nhận xét 1. Ta cũng gặp những định lý có dạng

∀ n ∈ N∗ , n ≥ a, P ( n) (8)

Lúc đó ở bước cở sở ta phải kiểm tra P (a) là mệnh đề đúng.

ÓNguyễn Thành Nhân 10 Chuyên Hùng Vương-BD


Các Bài Giảng 2023-2024 Đại Số 10 Chuyên

VÍ DỤ 1. Với mỗi k = 1, 2, 3, 4 ta đặt S k (n) = 1k + 2k + · · · + n k với n ∈ N∗ .

(a) Đưa ra một số cách để tính các tổng S k (n) theo n.


(b) Chứng minh các kết quả vừa đạt được bằng quy nạp.

2 Quy nạp mạnh

• Bước 1: Kiểm tra P (a) đúng.


• Bước 2: Giả thiết P ( k) đúng với mỗi số nguyên dương k thỏa a ≤ k ≤ n. Ta phải chứng minh P ( n + 1)
là mệnh đề đúng.

VÍ DỤ 2 (Trại hè Phương Nam 2017).


1
1 Chứng minh rằng với mỗi số nguyên dương n thì biểu thức x n + luôn biểu diễn được dưới dạng
xn
1
biểu thức đa thức hệ số nguyên của x + .
x
n p
p p n p
p p
2 Chứng minh rằng với mỗi số nguyên dương n bất kỳ thì 3+ 2+ 3 − 2 là một số vô tỉ.

Nhận xét 2. Ta có thể chứng minh được bài toán tổng quát sau đây, và sử dụng nó để giải một cách dễ dàng
các bài toán đặt ra ở dưới đây:
Gọi x1 , x2 là các nghiệm của phương trình bậc hai ax2 + bx + c = 0 (a ̸= 0). Đặt S n = x1n + x2n (n ∈ N∗ . Khi đó ta có

TOÁN CHUYÊN
hệ thức
aS n+2 + bS n+1 + cS n = 0, ∀ n ∈ N∗ .
Bài toán được chứng minh khá đơn giản dựa vào định lý Viète và quy nạp mạnh.

VÍ DỤ 3. Chứng minh rằng với mọi số nguyên dương n, biểu thức

n5 n4 n3 n
S ( n) = + + −
5 2 3 30

luôn là một số nguyên dương.

Dung khai triển Newton và biến đổi ta được điều cần chứng minh

VÍ DỤ 4. Gọi x1 , x2 là hai nghiệm của phương trình x2 − 27 x + 14 = 0. Đặt S n = x1n + x2n . Chứng minh rằng
với mọi số nguyên dương n thì S n là một số nguyên và S n không chia hết cho 715 với mọi n ∈ N∗ .

VÍ DỤ 5. Chứng minh rằng với mọi số nguyên a, b và số nguyên không âm c thì biểu thức S n =
p p
(a + b c)n + (a − b c)n là số nguyên với mọi số tự nhiên n.

Khi đó x1 , x2 là các nghiệm của phương trình

t2 − 2at + a2 − b2 c = 0.

Ta có S n = x1n + x2n thì ta có hệ thức liên hệ

S n+1 = 2aS n − a2 + b2 cS n−1 .

Hệ thức cuối cùng với nguyên lý quy nạp mạnh cho ta kết quả bài toán.

BÀI TẬP

Bài tập cơ bản


BÀI 1. Chứng minh rằng với mọi số tự nhiên n ≥ 2, ta có bất đẳng thức
1 1 1 1 13
+ +···+ + > .
n+1 n+2 2 n − 1 2 n 24

ÓNguyễn Thành Nhân 11 Chuyên Hùng Vương-BD


Các Bài Giảng 2023-2024 Đại Số 10 Chuyên

BÀI 2. Chứng minh rằng với mọi số tự nhiên n ≥ 2, ta có bất đẳng thức
1 1 1 1 1
+ +···+ + < 2− .
12 22 ( n − 1)2 n2 n

BÀI 3. Chứng minh rằng với mọi số tự nhiên n ≥ 2, ta có bất đẳng thức
1 1 1 1 p
p + p +···+ p + p < n.
1 2 n−1 n

BÀI 4. Chứng minh rằng với mọi số tự nhiên n ̸= 0, ta có bất đẳng thức
1 1 1 n
1+ + +···+ n > .
2 3 2 −1 2

BÀI 5. Chứng minh rằng với mọi số nguyên dương n, ta có bất đẳng thức
1 3 5 2n − 1 1
· · ··· <p .
2 4 6 2n 3n + 1

BÀI 6. Chứng minh rằng với mọi số nguyên dương n, ta có bất đẳng thức
1 1 1 1
+ + +···+ > 1.
n+1 n+2 n+3 n + 2n + 1
LATEX:Nguyễn Thành Nhân

BÀI 7. Chứng minh rằng với mọi số nguyên dương n, ta có bất đẳng thức
1 1 1 1 7
+ + +···+ < .
n+1 n+2 n+3 2 n 10

BÀI 8. Chứng minh rằng n 2n2 + 7 chia hết cho 3 với mọi số nguyên dương n.
¡ ¢

BÀI 9. Chứng minh rằng 4n + 15n − 1 chia hết cho 9 với mọi số nguyên dương n.
BÀI 10. Chứng minh rằng 52n + 7 chia hết cho 8 với mọi số nguyên dương n.
BÀI 11. Chứng minh rằng 7 · 22n−2 + 32n−1 chia hết cho 5 với mọi số nguyên dương n.
BÀI 12. Chứng minh rằng với mọi số tự nhiên n ≥ 1, ta có
n( n + 1)( n + 2)
1 · 2 + 2 · 3 + · · · + n( n + 1) = .
3

BÀI 13. Chứng minh rằng với mọi số tự nhiên n ≥ 1, ta có


n( n + 1)(2 n + 1)
12 + 22 + 32 + · · · + n2 = .
6

BÀI 14. Chứng minh rằng với mọi số tự nhiên n ≥ 1, ta có


1 2 3 n 3 2n + 3
+ + +···+ n = − .
3 32 33 3 4 4 · 3n

BÀI 15. Chứng minh rằng với n ∈ N∗ , ta có

( n + 1)( n + 2) · · · ( n + n) = 2n · 1 · 3 · 5 · · · (2 n − 1).

BÀI 16. Chứng minh rằng với mọi số nguyên dương n ta có

n5 n4 n3 n
S ( n) = + + −
5 2 3 30

là một số nguyên.
BÀI 17. Cho n (n ∈ N∗ ) đường thẳng nằm trong cùng một mặt phẳng và ở vị trí tổng quát (tức là không có hai
đường thẳng nào song song và không có ba đường thẳng nào đồng quy). Chứng minh rằng n đường thẳng
n2 + n + 2
này chia mặt phẳng thành miền.
2
BÀI 18. Chứng minh rằng với mọi số nguyên dương n ta có đẳng thức

1 + 5 + 9 + · · · + (4 n − 3) = n(2 n − 1).

BÀI 19. Chứng minh rằng với mọi n ∈ N∗ , ta có n3 + 2n chia hết cho 3.

ÓNguyễn Thành Nhân 12 Chuyên Hùng Vương-BD


Các Bài Giảng 2023-2024 Đại Số 10 Chuyên

2. Phương pháp quy nạp tổng quát


Để chứng minh mệnh đề
P ( n) đúng ∀ n ∈ N, n ≥ a (a ∈ N∗ ). (2)
Ta thực hiện theo các bước
• Bước 1 (Bước cơ sở)
Kiểm tra mệnh đề đúng với n = a.
• Bước 2 (Bước quy nạp)
Giả sử mệnh đề đúng với n = k (k ≥ a). Ta cần chứng minh mệnh đề đúng với n = k + 1.
Theo nguyên lý quy nạp toán học, ta kết luận mệnh đề đúng với mọi n ∈ N, n ≥ a.
BÀI 20. Chứng minh rằng với n ∈ N, n ≥ 4, ta có 3n−1 > n(n + 2).

3. Phương pháp quy nạp mạnh


Để chứng minh mệnh đề
P ( n) đúng ∀ n ∈ N, n ≥ a. (3)
Ta thực hiện theo các bước
• Bước 1: Kiểm tra P (a) đúng.

• Bước 2: Giả thiết P ( k) đúng với mỗi số nguyên dương k thỏa a ≤ k ≤ n. Ta phải chứng minh P ( n + 1) là
mệnh đề đúng.
Theo nguyên lý quy nạp toán học ta có mệnh đề đúng với mọi số nguyên dương n ≥ a.

TOÁN CHUYÊN
BÀI 21. Gọi x1 , x2 là hai nghiệm của phương trình x2 − 27 x + 14 = 0. Đặt S n = x1n + x2n (n ∈ N∗ ). Chứng minh rằng
với mọi số nguyên dương n thì S n là một số nguyên và S n không chia hết cho 715 với mọi n ∈ N∗ .
BÀI 22. Chứng minh rằng với n ∈ N∗ , ta có
n( n + 1)
1+2+···+ n = .
2

BÀI 23. Chứng minh rằng với n ∈ N∗ , ta có


n( n + 1)(2 n + 1)
12 + 22 + · · · + n2 = .
6

BÀI 24. Chứng minh rằng với n ∈ N∗ , ta có 2n+2 > 2n + 5.


BÀI 25. Chứng minh rằng với mọi n ∈ N∗ , ta có 7 · 22n−2 + 32n−1 chia hết cho 5.
BÀI 26. Chứng minh rằng với n ∈ N∗ , ta có

1 · 1! + 2 · 2! + · · · + n · n! = ( n + 1)! − 1.

n2 ( n + 1)2
BÀI 27. Chứng minh rằng với n ∈ N∗ , ta có 13 + 23 + · · · + n3 = .
4
(
u 1 = 2; u 2 = 3
BÀI 28. Cho dãy (u n ) được xác định . Chứng minh rằng u n = 2n−1 + 1 với mọi
u n = 3 u n−1 − 2 u n−2 , n = 3, 4, . . .
n ∈ N∗ .
BÀI 29. Chứng minh rằng với n ∈ N∗ , ta có
n( n + 1)( n + 2)( n + 3)
1 · 2 · 3 + 2 · 3 · 4 + · · · + n( n + 1)( n + 2) = .
4

BÀI 30. Chứng minh rằng với n ∈ N∗ , ta có

( n + 1)( n + 2) · · · ( n + n) = 2n · 1 · 3 · 5 · · · (2 n − 1).

BÀI 31. Chứng minh rằng với mọi số nguyên dương n ≥ 2 và | x| < 1 thì bất đẳng thức

(1 − x)n + (1 + x)n < 2n

luôn đúng.

ÓNguyễn Thành Nhân 13 Chuyên Hùng Vương-BD


Các Bài Giảng 2023-2024 Đại Số 10 Chuyên

BÀI 32. Chứng minh rằng với mọi số nguyên dương n ta có S (n) = 32n+1 + 40n − 67 chia hết cho 64.
BÀI 33. Chứng minh rằng với n ∈ N∗ và x ̸= k2π, k ∈ Z, ta có
n+1
sin x nx
sin x + sin 2 x + · · · + sin nx = 2
x · sin 2 .
sin
2
(
u 1 = 1; u 2 = 2 n2 − n + 2
BÀI 34. Cho dãy (u n ) được xác định . Chứng minh rằng u n = với
u n = 2 u n−1 − u n−2 + 1, n = 3, 4, . . . 2
mọi n ∈ N∗ .
BÀI 35. Cho n (n ∈ N∗ ) đường thẳng nằm trong cùng một mặt phẳng và ở vị trí tổng quát (tức là không có hai
đường thẳng nào song song và không có ba đường thẳng nào đồng quy). Chứng minh rằng n đường thẳng
n2 + n + 2
này chia mặt phẳng thành miền.
2
BÀI 36. Chứng minh rằng với mọi số nguyên dương n ta có

n5 n4 n3 n
S ( n) = + + −
5 2 3 30

là một số nguyên.
LATEX:Nguyễn Thành Nhân

BÀI 37. Cho n là số nguyên dương 0 < x1 ≤ x2 ≤ . . . ≤ xn . Chứng minh rằng với mọi n ≥ 3 ta có
x1 x2 xn−1 xn x2 x3 xn x1
+ +···+ + ≥ + +···+ + .
x2 x3 xn x1 x1 x2 xn−1 xn

Bài tập chuyên


BÀI 38. Chứng minh rằng với mỗi số nguyên dương n ≥ 12 đều có thể viết được dưới dạng n = 4 x + 5 y, trong
đó x, y là các số tự nhiên.
BÀI 39 (Đề thi vào Chuyên KHTN Hà Nội năm 2010, vòng 1).
Với mỗi số thực a, ta gọi phần nguyên của số a là số nguyên lớn nhất không vượt quá a và kí hiệu là [a].
Chứng minh rằng với mọi n nguyên dương ta luôn có

3 7 n2 + n + 1
· ¸
+ +...+ = n.
1·2 2·3 n( n + 1)

1 3 2n − 1 n2
BÀI 40. Chứng minh rằng + + · · · + = với mọi n nguyên dương.
4 + 14 4 + 34 4 + (2 n − 1)4 4 n2 + 1
BÀI 41. Với mỗi số thực a ta gọi phần nguyên của a là số nguyên lớn nhất không vượt quá a và kí hiệu là [a].
·r ¸2
3 1 1
Chứng minh rằng với mọi số nguyên dương n, biểu thức n + n− + không biểu diễn được dưới dạng
27 3
lập phương của một số nguyên dương.
BÀI 42. Cho x, y là các số nguyên lớn hơn 1 sao cho 4 x2 y2 − 7 x + 7 y là số chính phương. Chứng minh x = y.
BÀI 43. Tìm số tự nhiên n để n + 5 và n + 30 đều là số chính phương (số chính phương là bình phương của
một số nguyên).
BÀI 44. Ký hiệu tập hợp S gồm 2015 điểm phân biệt trên một mặt phẳng. Giả sử tất cả các điểm của S không
cùng nằm trên một đường thẳng. Chứng minh có ít nhất 2015 đường thẳng phân biệt mà mỗi đường thẳng
đi qua ít nhất 2 điểm của S .
x2 − 1 y2 − 1
BÀI 45. Với x, y là các số nguyên thỏa mãn đẳng thức = . Chứng minh rằng x2 − y2 chia hết cho
2 3
40.
BÀI 46. Chứng minh rằng không tồn tại các số nguyên x, y thỏa mãn đẳng thức:

12 x2 + 26 x y + 15 y2 = 4617.

BÀI 47. Giả sử p, q là hai số nguyên tố thỏa mãn đẳng thức

p( p − 1) = q( q2 − 1). (*)

1 Chứng minh rằng tồn tại số nguyên dương k sao cho p − 1 = kq, q2 − 1 = k p.

ÓNguyễn Thành Nhân 14 Chuyên Hùng Vương-BD


Các Bài Giảng 2023-2024 Đại Số 10 Chuyên

2 Tìm tất cả các số nguyên tố p, q thỏa mãn đẳng thức (*).

BÀI 48. Trên mặt phẳng cho hai điểm P,Q phân biệt. Xét 10 đường thẳng nằm trong mặt phẳng trên thỏa
mãn các tính chất sau:

a) Không có hai đường thẳng nào song song hoặc trùng nhau.

b) Mỗi đường thẳng đi qua P hoặc Q , không có đường thẳng nào đi qua cả P và Q . Hỏi 10 đường thẳng
trên có thể chia mặt phẳng thành tối đa bao nhiêu miền? Hãy giải thích.

BÀI 49. Giả sử m và n là những số nguyên dương với n > 1. Đặt S = m2 n2 − 4 m + 4n. Chứng minh rằng

1 Nếu m > n thì ( mn2 − 2)2 < n2 S < m2 n4 .

2 Nếu S là số chính phương thì m = n.

BÀI 50. Cho hai số hữu tỷ a và b thỏa mãn đẳng thức: a3 b + ab3 + 2a2 b2 + 2a + 2 b + 1 = 0.
Chứng minh rằng 1 − ab là bình phương của một số hữu tỉ.
BÀI 51. Cho 5 số nguyên dương phân biệt sao cho mỗi số dương trong chúng không có ước số nguyên nào
khác 2 và 3. Chứng minh rằng trong 5 số đó tồn tại 2 số mà tích của chúng là một số chính phương.
BÀI 52. Cho đa thức

P ( x) = m 1 x2 + n 1 x + k 1 , Q ( x) = m 2 x2 + n 2 x + k 2 , R ( x) = m 3 x2 + n 3 x + k 3

với m i , n i , k i là các số thực và m i > 0, i = 1, 2, 3. Giả sử phương trình P ( x) = 0 có hai nghiệm phân biệt a 1 , a 2 ,
phương trình Q ( x) = 0 có hai nghiệm phân biệt b1 , b2 , phương trình R ( x) = 0 có hai nghiệm phân biệt c 1 , c 2
thoả mãn

TOÁN CHUYÊN
P ( c 1 ) + Q ( c 2 ) = P ( c 2 ) + Q ( c 2 ),
P ( b 1 ) + R ( b 1 ) = P ( b 2 ) + R ( b 2 ),
Q (a 1 ) + R (a 1 ) = Q (a 2 ) + R (a 2 ).

Chứng minh: a 1 + a 2 = b1 + b2 = c 1 + c 2 .

BÀI 5. CHUYÊN ĐỀ: PHƯƠNG PHÁP CHỨNG MINH PHẢN


CHỨNG
Nguyễn Thành Nhân- Chuyên Hùng Vương- Bình Dương

A. CƠ SỞ LÝ THUYẾT

1. Phương pháp chứng minh bằng phản chứng)


Ta xét tình huống cụ thể cần chứng minh định lý

∀ x ∈ X , P ( x) ⇒ Q ( x) (4)

1 Phương pháp gián tiếp:


+ Để chứng minh (4) đúng thì ta chứng minh phủ định của (4) là mệnh đề sai, tức là mệnh đề sau sai.

∃ x ∈ X , P ( x) ⇒ Q ( x) (5)

Tuy nhiên mệnh đề (5) là tương đương logic với mệnh đề

∃ x ∈ X , P ( x) ∧ Q ( x) (6)

Chứng minh (6) sai ta làm như sau:


+ Giả sử x0 ∈ X mà P ( x0 ) đúng và Q ( x0 ) sai.
+ Dùng suy luận toán học để chỉ ra mâu thuẫn. Dẫn đến (6) sai. Do đó (4) đúng.

VÍ DỤ 1. Chứng minh rằng với mọi số nguyên n > 1 thì n có một ược nguyên tố.

ÓNguyễn Thành Nhân 15 Chuyên Hùng Vương-BD


Các Bài Giảng 2023-2024 Đại Số 10 Chuyên

VÍ DỤ 2. Chứng minh rằng tập hợp các số nguyên tố là vô hạn.

VÍ DỤ 3. Chứng minh rằng P (a) = a5 + 4a3 + 3a và Q (a) = a4 + 3a2 + 1 là hai số nguyên tố cùng nhau với
mọi số nguyên a.

VÍ DỤ 4. Cho p là một số nguyên tố. Chứng minh rằng mọi ước nguyên tố của 2 p − 1 đều lớn hơn p.

2. Bài tập áp dụng


BÀI 1. Trên mặt phẳng cho hai điểm P,Q phân biệt. Xét 10 đường thẳng nằm trong mặt phẳng trên thỏa
mãn các tính chất sau:

a) Không có hai đường thẳng nào song song hoặc trùng nhau.

b) Mỗi đường thẳng đi qua P hoặc Q , không có đường thẳng nào đi qua cả P và Q . Hỏi 10 đường thẳng
trên có thể chia mặt phẳng thành tối đa bao nhiêu miền? Hãy giải thích.

BÀI 2. Giả sử m và n là những số nguyên dương với n > 1. Đặt S = m2 n2 − 4 m + 4n. Chứng minh rằng
LATEX:Nguyễn Thành Nhân

1 Nếu m > n thì ( mn2 − 2)2 < n2 S < m2 n4 .

2 Nếu S là số chính phương thì m = n.

BÀI 3. Cho hai số hữu tỷ a và b thỏa mãn đẳng thức: a3 b + ab3 + 2a2 b2 + 2a + 2 b + 1 = 0.
Chứng minh rằng 1 − ab là bình phương của một số hữu tỉ.
BÀI 4. Cho 5 số nguyên dương phân biệt sao cho mỗi số dương trong chúng không có ước số nguyên nào khác
2 và 3. Chứng minh rằng trong 5 số đó tồn tại 2 số mà tích của chúng là một số chính phương.
BÀI 5. Cho đa thức

P ( x) = m 1 x2 + n 1 x + k 1 , Q ( x) = m 2 x2 + n 2 x + k 2 , R ( x) = m 3 x2 + n 3 x + k 3

với m i , n i , k i là các số thực và m i > 0, i = 1, 2, 3. Giả sử phương trình P ( x) = 0 có hai nghiệm phân biệt a 1 , a 2 ,
phương trình Q ( x) = 0 có hai nghiệm phân biệt b1 , b2 , phương trình R ( x) = 0 có hai nghiệm phân biệt c 1 , c 2
thoả mãn

P ( c 1 ) + Q ( c 2 ) = P ( c 2 ) + Q ( c 2 ),
P ( b 1 ) + R ( b 1 ) = P ( b 2 ) + R ( b 2 ),
Q (a 1 ) + R (a 1 ) = Q (a 2 ) + R (a 2 ).

Chứng minh: a 1 + a 2 = b1 + b2 = c 1 + c 2 .

ÁP DỤNG MỆNH ĐỀ VÀO SUY LUẬN TOÁN HỌC-BÀI TẬP


LÀM THÊM
Nguyễn Thành Nhân- Chuyên Hùng Vương- Bình Dương

Bài tập làm thêm


BÀI 10. Cho a ≥ 3. Chứng minh rằng a3 − 5a2 + 7a − 3 ≥ 0.
a2 b2 c2 a+b+c
BÀI 11. Cho a, b, c > 0. Chứng minh rằng + + ≥ .
b+c c+a a+b 2
BÀI 12. Cho a, b, c, d > 0. Chứng minh rằng

a2 b2 c2 d2 a+b+c+d
+ + + ≥ .
b+c+d c+d+a d+a+b a+b+c 3

BÀI 13. Chứng minh rằng mọi số chính phương thì chia hết cho 3 hoặc chia 3 dư 1.

ÓNguyễn Thành Nhân 16 Chuyên Hùng Vương-BD


Các Bài Giảng 2023-2024 Đại Số 10 Chuyên

BÀI 14. Chứng minh rằng nếu hai số nguyên có hiệu chia hết cho 2021 thì hiệu các lập phương của chúng
cũng chia hết cho 2021.
BÀI 15. Chứng minh rằng với mọi số nguyên dương n, ta có
1 1 1 p
p +p p +···+ p p = n − 1.
2+1 3+ 2 n+ n−1
BÀI 16. Chứng minh rằng với mọi số nguyên dương n, ta có
1 1 1 n
+ +···+ = .
1·3 3·5 (2 n − 1) · (2 n + 1) 2 n + 1
BÀI 17. Thầy Nhân viết các số nguyên 1, 2, 3, . . . , 2021, 2022 lên bảng. Thầy Nhân xoá đi 1010 số bất kì trên
bảng. Chứng minh rằng trong các số còn lại trên bảng luôn tìm được
1 3 số có tổng các bình phương là hợp số.

2 504 số có tổng các bình phương chia hết cho 4 .


BÀI 18. Cho đa thức P ( x) = ax2 + bx + c. Chứng minh rằng nếu P ( x) nhận giá trị nguyên với mỗi số nguyên x
thì ba số 2a, a + b, c đều là những số nguyên. Sau đó, chứng tỏ rằng nếu 2a, a + b, c là những số nguyên thì
P ( x) cūng nhận giá trị nguyên với mỗi số nguyên x.
BÀI 19. Cho đa thức P ( x) với các hệ số nguyên thỏa mãn P (2021) · P (2022) = 2023. Chứng minh rằng đa thức
P ( x) − 2024 không có nghiệm nguyên.
1 1 1
BÀI 20. Cho các số thực dương a, b, c thỏa mãn + + = 1. Chứng minh rằng
a b c
¶ s
1 1 1 1 abc
µ
+ + = .
2 a + bc b + ca c + ab (a + bc)( b + ca)( c + ab)

TOÁN CHUYÊN
BÀI 21. Chứng minh rằng với x là số nguyên bất kỳ thì 25 x + 1 không thể viết được dưới dang tích hai số
nguyên liên tiếp.
BÀI 22. Có 10 bạn học sinh tham gia thi đấu bóng bàn. Hai bạn bất kì đều phải đấu với nhau một trận, bạn
nào cũng phải gặp 9 đối thủ của mình và không có trận đấu hòa. Chứng minh rằng có thể xếp 10 bạn này
thành một hàng dọc sao cho bạn đứng trước thắng bạn đứng kề sau.
BÀI 23. Thầy giáo viết lên bảng các số tự nhiên 1, 2, . . . , n theo thứ tự đó từ trái sang phải và yêu cầu An đặt
vào trước mỗi số một dấu (+) hoặc dấu (-) để thu được một tổng có kết quả bằng 0.
1 Khi n = 2021, chứng minh rằng An không thể hoàn thành được yêu cầu của thầy giáo.

2 Hãy xác định tất cả các số tự nhiên n sao cho với mỗi n đó, An có thể thực hiện được yêu cầu thầy giáo
đề ra.
BÀI 24. Cho p 1 , p 2 , ..., p 12 là các số nguyên tố lớn hơn 3. Chứng minh p21 + p22 + ... + p212 chia hết cho 12.
BÀI 25. Trong mặt phẳng Ox y, điểm X được gọi là điểm "đẹp" nếu hoành độ và tung độ của X đều là các số
hữu tỷ. Chứng minh rằng nếu tam giác ABC đều thì một trong ba điểm A , B, C có ít nhất một điểm không
là điểm đẹp.
BÀI 26. Giả sử n là số tự nhiên thỏa mãn điều kiện n (n + 1) + 7 không chia hết cho 7. Chứng minh rằng
4 n2 − 5 n − 1 không là số chính phương.

BÀI 4. TẬP HỢP


Nguyễn Thành Nhân- Chuyên Hùng Vương- Bình Dương

A. LÝ THUYẾT CẦN NẮM


1. Tập hợp
a) Khái niệm
Khái niệm. Tập hợp là một khái niệm cơ bản của toán học và không được định nghĩa.
Ký hiệu: Dùng các chữ cái in hoa để ký hiệu tập hợp.
Để chỉ phần tử a thuộc tập A , ta viết a ∈ A .
Để chỉ phần tử b không thuộc tập B, ta viết b ̸∈ B.
Tập rỗng: Là tập không chứa phần tử nào, ký hiệu: ∅.

ÓNguyễn Thành Nhân 17 Chuyên Hùng Vương-BD


Các Bài Giảng 2023-2024 Đại Số 10 Chuyên

b) Cách cho một tập hợp


1 Cho bằng cách liệt kê tất cả các phần tử.

2 Chỉ ra các tính chất đặc trưng của các phần tử, lúc đó ta viết

A = { x ∈ X | x có tính chất P }.

3 Các tập hợp số N, Z, Q, R, N∗ .

4 Các tập con của tập R.


Trong toán học ta thường gặp các tập hợp con sau đây của tập hợp các số thực R với quy ước a, b là các
số thực và a < b.

a. Khoảng
¡ ¢
(a; b) = { x ∈ R|a < x < b}
a b
¡
(a; +∞) = { x ∈ R|a < x}
a
¢
(−∞; b) = { x ∈ R| x < b}
LATEX:Nguyễn Thành Nhân

b. Đoạn [a; b] = { x ∈ R|a ≤ x ≤ b}


£ ¤

a b

c. Nửa khoảng
£ ¢
[a; b) = { x ∈ R|a ≤ x < b}
a b
¡ ¤
(a; b] = { x ∈ R|a < x ≤ b}
a b
£
[a; +∞) = { x ∈ R|a ≤ x}
a
¤
(−∞; b) = { x ∈ R| x ≤ b}
b

Chú ý. Kí hiệu +∞ đọc là dương vô cực (hoặc dương vô cùng), kí hiệu −∞ đọc là âm vô cực (hoặc âm vô
cùng).

c) Hai tập hợp bằng nhau


Định nghĩa 1. Hai tập hợp A và B được gọi là bằng nhau nếu mỗi phần tử của A là một phần tử của B và
mỗi phần tử của B cũng là một phần tử của A .
Ký hiệu A = B.
Ta có
A = B ⇔ (∀ x, x ∈ A ⇔ x ∈ B) (1)
Nếu hai tập A và B không bằng nhau, ta nói A khác B và ký hiệu A ̸= B.

VÍ DỤ 1. Lấy ví dụ về hai tập bằng nhau, hai tập khác nhau.

d) Tập con
Định nghĩa 2. Tập A được gọi là tập con của tập B nếu mỗi phần tử của tập A đều là một phần tử của tập B.
Ký hiệu: A ⊂ B. Ta có
A ⊂ B ⇔ (∀ x : x ∈ A ⇒ x ∈ B) (2)

ÓNguyễn Thành Nhân 18 Chuyên Hùng Vương-BD


Các Bài Giảng 2023-2024 Đại Số 10 Chuyên

VÍ DỤ 2. Lấy ví dụ về tập con.

Tính chất 1. Cho A , B, C là các tập hợp. Ta có các tính chất.

1 A ⊂ A, ∀ A . 2 ∅ ⊂ A, ∀ A . 3 A ⊂ B, B ⊂ C ⇒ A ⊂ C .

e) Biểu đồ Venn
Dùng một đường cong khép kín để biểu diễn một tập hợp.

2. Các phép toán trên tập hợp


a) Phép hợp
Định nghĩa 3. Cho hai tập hợp A và B. Hợp của A và B là tập hợp bao gồm tất cả các phần tử thuộc A hoặc
thuộc B. Ký hiệu A ∪ B.
Ta có
A ∪ B = { x|( x ∈ A ) ∨ ( x ∈ B)} (3)
Định nghĩa mở rộng tự nhiên cho n (n ≥ 3) tập hợp.

VÍ DỤ 3. Lấy ví dụ về hợp của hai tập hợp.

TOÁN CHUYÊN
b) Phép giao
Định nghĩa 4. Cho hai tập hợp A và B. Giao của A và B là tập hợp bao gồm tất cả các phần tử vừa thuộc A
vừa thuộc B. Ký hiệu A ∩ B.
Ta có
A ∩ B = { x | ( x ∈ A ) ∧ ( x ∈ B )} (4)
Định nghĩa mở rộng tự nhiên cho giao của n (n ≥ 3) tập hợp.

VÍ DỤ 4. Lấy ví dụ về giao của hai tập hợp.

c) Phép lấy hiệu


Định nghĩa 5. Cho hai tập hợp A và B. Hiệu của A và B là tập hợp bao gồm tất cả các phần tử thuộc A nhưng
không thuộc B. Ký hiệu A \B.
Ta có
A \B = { x|( x ∈ A ) ∧ ( x ̸∈ B)} (5)

VÍ DỤ 5. Lấy ví dụ về phép lấy hiệu của hai tập hợp.

d) Phép lấy phần bù


Định nghĩa 6.
• Cho tập A là con của tập E . Phần bù của A trong E là tập hợp các phần tử thuộc E nhưng không thuộc A .
Ký hiệu C E A . Ta có
CE A = E\ A (6)
• Phần bù của tập A , ký hiệu là A , là tập hợp các phần tử không thuộc A .
Ta có n o
A = x| x ∈ A (7)

VÍ DỤ 6. Lấy ví dụ về phép lấy phần bù.

ÓNguyễn Thành Nhân 19 Chuyên Hùng Vương-BD


Các Bài Giảng 2023-2024 Đại Số 10 Chuyên

3. Công thức bao hàm loại trừ


Ký hiệu n ( A ) hoặc | A | là số phần tử của một tập hữu hạn A . Khi đó với A , B, C là các tập hữu hạn thì
1 | A ∪ B | = | A | + | B | − | A ∩ B |;

2 | A ∪ B ∪ C | = | A | + | B | + | C | − | A ∩ B | − | B ∩ C | − | A ∩ C | + | A ∩ B ∩ C |.

B. BÀI TẬP
BÀI 1. Lấy một số ví dụ đơn giản về tập hợp.
BÀI 2. Cho tập A có n phần tử. Tính số tập con của tập A (kể cả tập rỗng).
BÀI 3 (Công thức bao hàm loại trừ). Cho A , B lần lượt có a và b phần tử. Biết số phần tử chung của A và B
là c. Tính số phần tử của tập A ∪ B.
Hãy mở rộng bài toán cho trường hợp ba tập hợp, mỗi tập có hữu hạn phần tử.
BÀI 4. Với mỗi tập X ta gọi P ( X ) là tập tất cả các tập con của X . Chứng minh rằng

A ⊂ B ⇔ P ( A ) ⊂ P ( B ).

BÀI 5. Cho các tập A 1 , A 2 , . . . , A n sao cho A i ̸= A j , ∀ i ̸= j . Chứng minh rằng có ít nhất một tập A i không chứa
tập nào trong các tập còn lại.
LATEX:Nguyễn Thành Nhân

BÀI 6. Cho tập hợp S ⊂ R thỏa mãn các tính chất sau đây
p p
1 S ⊃ Z; 2 2 + 3 ∈ S; 3 ∀ x, y ∈ S : x + y ∈ S, x y ∈ S .

1
Chứng minh rằng p p ∈ S.
2+ 3
BÀI 7. Ba số a, b, c theo thứ tự lập thành một cấp số cộng nếu b − a = c − b.
Cho tập X = {1; 2; . . . ; 9}. Chia tập X thành hai tập con rời nhau A và B ( A và B không có phần tử chung).
Chứng minh rằng với mọi cách chia, luôn tồn tại một tập chứa ba số lập thành một cấp số cộng.
Nhận xét 3. Để dễ nhận ra sự có mặt của các phần tử đã xuất hiện trong các tập theo tiến trình lập luận, ta
nên chia ra hai cột A , B khi lập luận.
BÀI 8. Cho tập X = {1; 2; . . . ; 15} và M là tập con của X sao cho tích của ba phần tử khác nhau bất kỳ của M
đều không là số chính phương.
1 Hãy chỉ ra tập M gồm 10 phần tử. 2 Hãy xác định số phần tử lớn nhất của M .

BÀI 9. Cho tập X = {1; 2; . . . ; 2019} và hai tập con A , B của X có tổng số phần tử lớn hơn 2020. Chứng minh
rằng tồn tại ít nhất một phần tử của A và một phần tử của B có tổng bằng 2020.
BÀI 10. Cho A và B là các tập hợp khác rỗng và có hữu hạn phần tử. Biết rằng số phần tử nằm trong cả A
và B bằng một phần ba số phần tử của A và hợp của A và B gồm 9 phần tử. Tìm số phần tử của mỗi tập.
BÀI 11. Cho tập hữu hạn X . Ta chọn ra 50 tập con A 1 , A 2 , . . . , A 50 , mỗi tập đều chứa quá nửa số phần tử của X .
Chứng minh rằng, tồn tại tập con A của X sao cho số phần tử của A không vượt quá 5 và A ∩ A i ̸= ∅, ∀ i = 1, 50.
BÀI 12. Cho a, b là các số nguyên dương sao cho a + b là một số lẻ. Chia tập các số nguyên dương N∗ thành
hai tập rời nhau A và B. Chứng minh rằng luôn tồn tại hai phần tử x, y thuộc cùng một tập sao cho

| x − y| ∈ {a; b}.

BÀI 13. Cho tập hợp A = {1; 2; 3; . . . ; 3n } (n ≥ 2).


1 Hãy chỉ ra tập con B của A có 2n phần tử sao cho không có ba phần tử nào của tập B lập thành một
cấp số cộng khi n = 2.
2 Chứng minh rằng, với mọi n ≥ 2, tồn tại tập con B của A có 2n phần tử sao cho không có ba phần tử nào
của B lập thành một cấp số cộng.
BÀI 14 (Chọn đội Ams-2020). Xét tập A gồm các số nguyên dương thỏa mãn đồng thời các điều kiện sau
1 Phần tử lớn nhất của A là 100.

2 Với mọi x ∈ A , x không phải là phần tử nhỏ nhất thì tồn tại a b c ∈ A (không nhất thiết phân biệt) sao
cho x = a + b + c.
Chứng minh rằng tất cả các phần tử của A đều chia hết cho 4.

ÓNguyễn Thành Nhân 20 Chuyên Hùng Vương-BD


Các Bài Giảng 2023-2024 Đại Số 10 Chuyên

BÀI 15 (Mỹ 1989). Giả sử S là tập con của {1, 2, . . . , 2005} có tính chất hiệu của hai phần tử bất kỳ của S đều
khác 4 và 7. Hãy tìm số phần tử lớn nhất có thể có của S .
BÀI 16 (Chuyên ĐHSP Hà Nội 2014). Có bao nhiêu tâp con A của tập hợp {1, 2, 3, . . . , 2014} thỏa mãn : A có ít
y2
nhất hai phần tử và nếu x ∈ A , y ∈ A , x > y thì ∈ A.
x− y
BÀI 17. Cho tập hợp A gồm 31 phần tử và dãy gồm m tập con của A thỏa mãn đồng thời các điều kiện sau
i) Mỗi tập thuộc dãy có ít nhất hai phần tử
ii) Nếu hai tập thuộc dãy có chung nhau ít nhất hai phần tử thì số phần tử của hai tập này khác nhau.
Chứng minh rằng m ≤ 900.
BÀI 18.
Số nguyên a được gọi là số "đẹp" nếu với mọi cách sắp xếp theo thứ tự tùy ý của 100 số 1, 2, 3,. . . , 100
luôn tồn tại 10 số liên tiếp có tổng không nhỏ hơn a. Tìm số đẹp lớn nhất.
BÀI 19. Tìm hợp số nguyên dương n sao cho tồn tại một cách sắp xếp các số 1, 2, . . . , n thành a 1 , a 2 , . . . , a n mà
khi chia các số a 1 , a 1 a 2 , a 1 a 2 a 3 , . . . , a 1 a 2 . . . a n cho n ta được các số dư đôi một khác nhau.
BÀI 20. Cho n là số nguyên dương, n ≥ 5. Xét một đa giác lồi n cạnh. Người ta muốn kẻ một số đường chéo
của đa giác mà các đường chéo này chia đa giác đã cho thành đúng k miền, mỗi miền là một ngũ giác lồi (
hai miền bất kỳ không có điểm trong chung).
a. Chứng minh rằng ta có thể thực hiện được với n = 2018, k = 672.
b. Với n = 2017, k = 672 ta có thể thực hiện được không? Hãy giải thích.
BÀI 21. Gọi M là tập hợp tất cả 4039 số nguyên liên tiếp từ -2019 đến 2019. Chứng minh rằng trong 2021 số

TOÁN CHUYÊN
đôi một phân biệt được chọn bất kì từ tập M luôn tồn tại 3 số đôi một phân biệt có tổng bằng 0.
BÀI 22.
Gọi M là tập hợp tất cả 4039 số nguyên liên tiếp từ -2019 đến 2019. Chứng minh rằng trong 2021 số đôi một
phân biệt được chọn bất kì từ tập M luôn tồn tại 3 số đôi một phân biệt có tổng bằng 0.
p 1
BÀI 23. Trên một bảng đen ta viết ba số 2, 2, p . Ta bắt đầu thực hiện một trò chơi như sau: Mỗi lần chơi
2
a+b
ta xóa hai số nào đó trong ba số trên bảng, giả sử là a và b, rồi viết vào hai vị trí vừa xóa hai số mới là p
2
|a − b|
và p , đồng thời giữ nguyên số còn lại. Như vậy sau mỗi lần chơi trên bảng luôn có ba số. Chứng minh
2
1 p p
rằng dù ta có chơi bao nhiêu lần đi chăng nữa thì trên bảng không thể có đồng thời ba số p ; 2; 1 + 2.
2 2
BÀI 24. Cho a 1 , a 2 , · · · , a 11 là các số nguyên dương lớn hơn hoặc bằng 2, đôi một khác nhau và thỏa mãn
a 1 + a 2 + · · · + a 11 = 407. Tồn tại hay không số nguyên dương n sao cho tổng các số dư của các phép chia n cho
22 số a 1 , a 2 , · · · , a 11 , 4a 1 , 4a 2 , · · · , 4a 11 bằng 2012.
BÀI 25. Có bao nhiêu tập hợp con A của tập hợp {1; 2; 3; 4; . . . ; 2014} thỏa mãn điều kiện A có ít nhất hai phần
y2
tử và nếu x ∈ A, y ∈ A, x > y, thì ∈ A.
x− y
BÀI 26. Cho S là tập hợp các số nguyên dương n có dạng n = x2 + 3 y2 trong đó x, y là các số nguyên. Chứng
minh rằng:
1 Nếu a, b ∈ S thì ab ∈ S .
N
2 Nếu N ∈ S và N chẵn thì N chia hết cho 4 và 4 ∈ S.

BÀI 27. Giả sử mỗi điểm của mặt phẳng được tô bởi một trong ba màu: xanh, đỏ, vàng. Chứng minh rằng
tồn tại ba điểm cùng màu là ba đỉnh của một tam giác cân.
BÀI 28. Có 45 người tham gia một cuộc họp. Quan sát sự quen nhau giữa họ, người ta thấy rằng: nếu hai
người có số người quen bằng nhau thì lại không quen nhau. Gọi S là số cặp người quen nhau trong một cuộc
họp (cặp người quen nhau không kể thứ tự sắp xếp giữa hai người trong cặp).
1 Xây dựng ví dụ để S = 870.

2 Chứng minh S ≤ 870.

BÀI 29 (Chọn đội tuyển 10 chuyên KHTN 2020).


Chứng minh rằng tồn tại vô hạn số nguyên dương n sao cho 3n − 1 chia hết cho n2 .

ÓNguyễn Thành Nhân 21 Chuyên Hùng Vương-BD


Các Bài Giảng 2023-2024 Đại Số 10 Chuyên

BÀI 30. Cho tập hợp A gồm 2022 số tự nhiên liên tiếp từ 1 đến 2022. Tìm một số tự nhiên n nhỏ nhất sao cho
mọi tập hợp con gồm n phần tử của A đều chứa 3 phần tử là các số đôi một nguyên tố cùng nhau.
BÀI 31. Trên bàn có n viên kẹo. Hai bạn An và Bình cùng chơi một trò chơi như sau: Hai bạn luân phiên lấy
kẹo trên bàn, mỗi lần chỉ lấy 1, 2, 3, 4 hoặc 5 viên kẹo và phải lấy số viên kẹo khác với số viên kẹo của bạn
còn lại vừa lấy ngay trước đó. Bạn đầu tiên không thể thực hiện được lượt chơi của mình là người thua cuộc.
Nếu An là người đi trước lấy kẹo.
1 Với n = 7, hãy chỉ ra chiến thuật chơi của bạn Bình khiến bạn An là người thua cuộc.

2 Với n = 22, hãy chỉ ra chiến thuật chơi của bạn An khiến bạn Bình là người thua cuộc.

BÀI 32. Cho tập A = {1, 2, 3, . . . , 2021}. Tìm số nguyên dương k lớn nhất (k > 2) sao cho ta có thể chọn được k số
phân biệt từ tập A mà tổng của hai số phân biệt bất kỳ trong k số được chọn không chia hết cho hiệu của
chúng.

BÀI TẬP MỆNH ĐỀ- TẬP HỢP (TOÁN THƯỜNG)


Nguyễn Thành Nhân- Chuyên Hùng Vương- Bình Dương

CÁC BÀI TOÁN


LATEX:Nguyễn Thành Nhân

BÀI 1. Để phục vụ cho một hội nghị quốc tế, ban tổ chức huy động 35 người phiên dịch tiếng Anh, 30 người
phiên dịch tiếng Pháp, trong đó có 16 người phiên dịch được cả hai thứ tiếng Anh, và Pháp. Hãy trả lời các
câu hỏi sau:
1 Ban tổ chức đã huy động bao nhiêu người phiên dịch cho hội nghị đó?

2 Có bao nhiêu người chỉ phiên dịch được tiếng Anh?

3 Có bao nhiêu người chỉ phiên dịch được tiếng Pháp?

BÀI 2. Trong một khoảng thời gian nhất định tại một địa phương, Đài khí tượng thủy văn đã thống kê được:
Số ngày có mưa: 10 ngày; số ngày có gió to: 8 ngày; số ngày lạnh: 6 ngày; số ngày có mưa và gió to: 5 ngày;
số ngày mưa và lạnh: 4 ngày; số ngày lạnh và có gió to: 3 ngày; số ngày có mưa, lạnh và có gió to: 1 ngày.
Hỏi trong khoảng thời gian đó, địa phương trên có bao nhiêu ngày có thời tiết xấu (có mưa hoặc gió to hoặc
lạnh)?
BÀI 3. Cho 2 tập khác rỗng A = (m − 1; 4]; B = (−2; 2 m + 2), m ∈ R. Tìm tất cả các giá trị thực của m để A ⊂ B.
BÀI 4. Cho tập A = (−∞; 1), B = { x ∈ R | x2 − 3 x − m + 9 = 0}. Tìm tất cả các giá trị của tham số m để A ∩ B ̸= ∅.
BÀI 5. Cho A = {0; 2; 4; 6; 8}, B = {0; 3; 6; 9}, E = {0; 1; 2; 3; 4; 5; 6; 8; 9}. Xác định các tập hợp A ∩ B, A ∪ B, A \ B, C E B.
BÀI 6. Cho các tập hợp A = (−∞; 2), B = (−3; +∞), C = (1; 4). Tìm ( A ∩ B) ∪ C .
BÀI 7. Cho hai tập hợp A = (−∞; 3] và B = (−1; 4]. Tìm A ∩ B, A ∪ B, CR A , A \ B.
BÀI 8. 1 Xét tính đúng sai của các mệnh đề sau và nêu mệnh đề phủ định

P ( x) : “∃ x ∈ N, x2 = 2 x + 3”

Q ( x) : “∀ x ∈ Z, x + 1 > 0”.

2 Cho mệnh đề A : “Nếu một tam giác có một góc bằng 60◦ thì tam giác đó đều ”.
Xét tính đúng sai của mệnh A và nêu điều kiện cần, điều kiện đủ.
3 Xét tính đúng sai của mệnh đề B: “Hai tam giác bằng nhau khi và chỉ khi hai tam giác đó có chu vi
bằng nhau ”.
BÀI 9. 1 Cho hai tập hợp
A = { x ∈ Z, | x − 1| < 3}
B = x ∈ Q, x2 − 9 (2 x − 1) = 0
© ¡ ¢ ª

Liệt kê phần tử của các tập hợp trên. Tìm phần tử các tập hợp A ∪ B, A \ B, B \ A .
2 Cho hai tập hợp M = { x ∈ R, | x + 2| ≤ 4}, N = x ∈ R, x2 + 2 > x( x + 1) > 0 . Tìm M ∩ N , M \ N , C R M , C R N .
© ª

BÀI 10. Cho A = (−∞; m), B = [5 m − 2; 5 m + 5]. Tìm m để


1 A ∩ B = ∅.

ÓNguyễn Thành Nhân 22 Chuyên Hùng Vương-BD


Các Bài Giảng 2023-2024 Đại Số 10 Chuyên

2 B ⊂ A.

3 A ⊂ (R\B).

4 (R\ A ) ∩ B ̸= ∅.

BÀI 11.
1 Xét tính đúng, sai và lập mệnh đề phủ định của mỗi mệnh đề sau: ∃ x ∈ Q : x2 − 7 ̸= 7 x.
1
2 Cho mệnh đề chứa biến “Q ( x) : x < ”. Tìm một giá trị thực của x để được mệnh đề đúng và một giá trị
x
của x để được mệnh đề sai?
BÀI 12. Biểu diễn các tập sau trên trục số thực và tìm A ∩ B, A ∪ B.
1 A = [−5; 4) và B = [2; +∞).

2 A = { x ∈ R | x ≤ 3} và B = { x ∈ R : | x| > 1}.

BÀI 13. 1 Xét tính đúng sai và viết mệnh đề phủ định của các mệnh đề sau: ∀ x ∈ R, x2 > 0.

2 Cho hai mệnh đề P và Q


P : ABCD là tứ giác nội tiếp.
Q : Tổng số đo hai góc đối nhau bằng 180◦ .
Hãy phát biểu mệnh đề P ⇒ Q dưới dạng điều kiện cần và và điều kiện đủ.
3 Phát biểu mệnh đề P ⇔ Q và xét tính đúng sai của nó. Giải thích?
P : “Bất phương trình x2 − 3 x + 1 > 0 có nghiệm”.
Q : “Bất phương trình x2 − 3 x + 1 ≤ 0 vô nghiệm”.

TOÁN CHUYÊN
¯p
1 Cho A = x3 + 1¯ x ∈ Z, x2 ≤ 5 , B = x ∈ N¯ x ≤ 4 và x chia hết cho 3 . Xác định A ∩ B, A ∪ B, B \ A .
© ¯ ª © ª
BÀI 14.
2 Cho E = x ∈ R¯2 x − 3 > 0 , F = x ∈ R¯ | x − 1| ≤ 2 . Xác định E ∩ F , E ∪ F , E \ F, C R E .
© ¯ ª © ¯ ª

BÀI 15. Xét tính đúng sai của mệnh đề và lập mệnh đề phủ định của mệnh đề A : “∀ x ∈ R : ( x − 2)2 > 0”.
BÀI 16. Xét tính đúng sai và phát biểu dưới dạng sử dụng thuật ngữ
p “điều kiện cần”, “điều kiện đủ” của
3
mệnh đề B “Nếu G là trọng tâm tam giác đều ABC cạnh a thì AG = a ”.
3
BÀI 17. Xét tính đúng sai của mệnh đề C : “Hai tam giác bằng nhau khi và chỉ khi chúng có các góc bằng
nhau”.
BÀI 18. Cho các tập hợp A = {−2; −1; 1; 2; 3; 4}, B = { x ∈ Z | x2 − 3 x + 2 2 x2 − 5 = 0}. Tìm B \ A .
¡ ¢¡ ¢

3
µ ¸
BÀI 19. Cho các tập hợp A = − ; 4 , B = { x ∈ R | 1 ≤ | x| ≤ 2}. Tìm A ∩ B.
2
BÀI 20. Biểu diễn tập nghiệm của bất phương trình x − 2 y > 4.
BÀI 21. Cho mệnh đề P : “∃ x ∈ R, x < y ⇒ x2 > y2 ”. Hãy xét tính đúng sai của mệnh đề P và nêu mệnh đề phủ
định của mệnh đề P .
BÀI 22. Cho △ ABC có O là trung điểm của BC , xét hai mệnh đề:
P : “△ ABC vuông tại A ” và Q : “Đường trung tuyến AM bằng nửa độ dài BC ”.
Phát biểu mệnh đề P ⇒ Q và sử dụng các thuật ngữ “điều kiện cần”, “điều kiện đủ” để phát biểu lại mệnh
đề.
BÀI 23. Xét hai mệnh đề P : “Số nguyên có chữ số tận cùng là 0 hoặc 5” và Q : “Số nguyên chia hết cho 5”.
Hai mệnh đề P và Q có tương đương không? Nếu có, hãy phát biểu một định lí thể hiện điều này.
2 2
¢ ¡ 2A = x ∈ Q
© ¡ ¢¡ ¢ ª
BÀI ©24. Cho
¡ tập hợp ¢ | xª − 9 x − 2 x = 0 và
2
B = x ∈ Q | 8 − 2x x − 5x + 6 = 0 .
Viết các tập hợp A và B bằng cách liệt kê các phần tử đồng thời xác định A ∩ B, A \ B.
BÀI 25. Cho hai tập hợp A = x ∈ R | 16 − x2 ≥ 0 và B = { x ∈ R | | x − 3| ≤ 5}.
© ª

Xác định các tập hợp sau C = A ∩ B, D = B \ A , E = B ∪ A .


BÀI 26. Nêu mệnh đề phủ định của các mệnh đề sau và cho biết tính đúng sai của mệnh đề phủ định đó
A : “∀ x ∈ R, x3 − x2 + 1 > 0”.
1
B: “Tồn tại số thực a sao cho a + 1 + ≤ 2.”
a+1
BÀI 27. Cho các tập hợp A = x ∈ R | x2 ≤ 4 , B = { x ∈ R | x < 1}. Viết các tập hợp sau đây A ∪ B, A ∩ B, A \ B, CR B
© ª

dưới dạng các khoảng, nửa khoảng, đoạn.

ÓNguyễn Thành Nhân 23 Chuyên Hùng Vương-BD


Các Bài Giảng 2023-2024 Đại Số 10 Chuyên

BÀI 28. Cho hai tập hợp A = [1; 8] và B = x ∈ R¯ mx2 − 2( m + 1) x + 3m + 3 = 0 , với m ∈ R. Tìm m để tập B có đúng
© ¯ ª

hai tập con đồng thời B ⊂ A .


BÀI 29. Để phục vụ cho một hội nghị quốc tế, ban tổ chức huy động 35 người phiên dịch tiếng Anh, 30 người
phiên dịch tiếng Pháp, trong đó có 16 người phiên dịch được cả hai thứ tiếng Anh, và Pháp. Hãy trả lời các
câu hỏi sau:
1 Ban tổ chức đã huy động bao nhiêu người phiên dịch cho hội nghị đó?

2 Có bao nhiêu người chỉ phiên dịch được tiếng Anh?

3 Có bao nhiêu người chỉ phiên dịch được tiếng Pháp?

BÀI 30. Cho hai tập hợp A = [1; 8] và B = x ∈ R¯ mx2 − 2( m + 1) x + 3m + 3 = 0 , với m ∈ R. Tìm m để tập B có đúng
© ¯ ª

hai tập con đồng thời B ⊂ A .


BÀI 31. 1 Xét tính đúng sai của mệnh đề sau và lập mệnh đề phủ định của mệnh đề đó: ∃ n ∈ N, n2 + 1 chia
hết 8.
2 Dùng phương pháp chứng minh phản chứng. Chứng minh mệnh đề: Nếu x, y là hai số không dương
thì tích x · y là một số không âm.
BÀI 32. Chứng minh với mọi x, y, ta có
1 x2 − x y + y2 + 1 > 0. 2 4 x2 + 4 y2 + 6 x + 3 Ê 4 x y.
LATEX:Nguyễn Thành Nhân

BÀI 33. Chứng minh rằng


1 Nếu a > 2 thì a3 − 4a2 + 5a − 2 > 0.

aA + bB a + b A + B
2 Nếu a Ê b, A Ê B thì Ê · .
2 2 2
BÀI 34. Nêu mệnh đề phủ định của các mệnh đề sau và cho biết tính đúng sai của mệnh đề phủ định đó.
1 A : “Hình thoi có hai đường chéo vuông góc với nhau”.

2 B : “Tổng hai cạnh của một tam giác nhỏ hơn cạnh còn lại”.

3 C : “Trong tam giác tổng ba góc không bằng 180◦ ”.

4 D : “Tồn tại hình thang là hình vuông”.

BÀI 35. Phát biểu mệnh đề P ⇔ Q bằng các thuật ngữ “khi và chỉ khi”, “nếu và chỉ nếu” và và xét tính đúng
sai của nó.
1 P : “Tứ giác ABCD là hình thoi” và Q : “Tứ giác ABCD là hình bình hành có hai đường chéo vuông góc
với nhau”.
p p
2 P : “Bất phương trình x2 − 3 x > 1 có nghiệm” và Q : “ (−1)2 − 3.(−1) > 1”.

BÀI 36. Lập mệnh đề kéo theo và mệnh đề tương đương của hai mệnh đề sau đây và cho biết tính đúng, sai
của chúng:
P : “Điểm M nằm trên phân giác của góc Ox y” và Q : “Điểm M cách đều hai cạnh Ox, O y”.
BÀI 37. Phát biểu mệnh đề P ⇔ Q bằng hai cách và và xét tính đúng sai của nó.
1 Cho tứ giác ABCD . Xét hai mệnh đề P : “Tứ giác ABCD là hình vuông” và Q : “Tứ giác ABCD là hình
chữ nhật có hai đường chéo bằng vuông góc với nhau”.
2 P : “Bất phương trình x2 − 3 x + 1 > 0 có nghiệm” và Q : “Bất phương trình x2 − 3 x + 1 É 0 vô nghiệm”.

BÀI 38. Hãy phát biểu mệnh đề kéo theo P ⇒ Q , Q ⇒ P và xét tính đúng sai của mệnh đề này.
1 Cho tứ giác ABCD và hai mệnh đề P : “Tổng hai góc đối của tứ giác lồi bằng 180◦ ” và Q : “Tứ giác nội
tiếp được đường tròn”.
p p p ¢2
2 P : “ 2 − 3 > −1” và Q : “ 2 − 3 > (−1)2 ”.
¡p

BÀI 39. Nêu mệnh đề phủ định của các mệnh đề sau và cho biết tính đúng sai của mệnh đề phủ định đó.
1 A : “6 là số nguyên tố”.
¡p p ¢2
2 B : “ 3 − 27 là số nguyên”.

ÓNguyễn Thành Nhân 24 Chuyên Hùng Vương-BD


Các Bài Giảng 2023-2024 Đại Số 10 Chuyên

3 C : “∃ n ∈ N, n( n + 1) là một số chính phương”.

4 D : “∀ n ∈ N, n4 − n2 + 1 là hợp số”.

BÀI 40. Xét tính đúng sai của mệnh đề sau và nêu mệnh đề phủ định của nó.
1 P ( x) : “∃ x ∈ Z, x2 = 3”.

2 P ( n) : “∀ n ∈ N∗ : 2n + 3 là một số nguyên tố”.

3 P ( x) : “∀ x ∈ R, x2 + 4 x + 5 > 0”.

4 P ( x) : “∀ x ∈ R, x4 − x2 + 2 x + 2 Ê 0”.

BÀI 41. Để phục vụ cho một hội nghị quốc tế, ban tổ chức huy động 35 người phiên dịch tiếng Anh, 30 người
phiên dịch tiếng Pháp, trong đó có 16 người phiên dịch được cả hai thứ tiếng Anh, và Pháp. Hãy trả lời các
câu hỏi sau:
1 Ban tổ chức đã huy động bao nhiêu người phiên dịch cho hội nghị đó?

2 Có bao nhiêu người chỉ phiên dịch được tiếng Anh?

3 Có bao nhiêu người chỉ phiên dịch được tiếng Pháp?

BÀI TẬP TRẮC NGHIỆM

Câu 1. Cho tập hợp A = { x ∈ R | x2 + 2 x − m − 1 = 0} (m là tham số). Tìm m để tập A khác rỗng.
A. m > −2. B. m ≤ −2. C. m ≥ −2. D. m < −2.
Câu 2. Liệt kê các phần tử của tập hợp A = { x ∈ Z | | x − 2| < 3}.

TOÁN CHUYÊN
A. A = {0; 1; 2; 3; 4}. B. A = {0; 1; 2}. C. A = {−1; 0; 1; 2; 3; 4; 5}. D. A = {0; 1; 2; 3}.
Câu 3. Trong các mệnh đề sau, mệnh đề nào đúng?
A. ∃ n ∈ N, n3 − n không chia hết cho 3. B. ∀ x ∈ R, x < 4 ⇔ x2 < 16.
2 x3 − 6 x2 + x − 3
C. ∃ k ∈ Z, k2 + k + 1 là một số chẵn. D. ∀ x ∈ Z, ∈ Z.
2 x2 + 1
Câu 4. Cho tập hơp A = {−1; 0; 1} và B = {−2; −1; 0; 1; 2}. Gọi X là tập hợp thỏa mãn A ⊂ X ⊂ B. Số các tập X

A. 5. B. 4. C. 3. D. 2.
Câu 5. Cho tập hợp A = { x ∈ R | x2 + 2 x − m − 1 = 0} ( m là tham số). Tìm m để tập A khác rỗng.
A. m > −2. B. m ≤ −2. C. m ≥ −2. D. m < −2.
Câu 6. Liệt kê các phần tử của tập hợp A = { x ∈ Z | | x − 2| < 3}.
A. A = {0; 1; 2; 3; 4}. B. A = {0; 1; 2}. C. A = {−1; 0; 1; 2; 3; 4; 5}. D. A = {0; 1; 2; 3}.
Câu 7. Trong các mệnh đề sau, mệnh đề nào đúng?
A. ∃ n ∈ N, n3 − n không chia hết cho 3. B. ∀ x ∈ R, x < 4 ⇔ x2 < 16.
2 x3 − 6 x2 + x − 3
C. ∃ k ∈ Z, k2 + k + 1 là một số chẵn. D. ∀ x ∈ Z, ∈ Z.
2 x2 + 1
Câu 8. Cho tập hơp A = {−1; 0; 1} và B = {−2; −1; 0; 1; 2}. Gọi X là tập hợp thỏa mãn A ⊂ X ⊂ B. Số các tập X

A. 5. B. 4. C. 3. D. 2.
Câu 9. Mệnh đề nào sau đây đúng?
A. Tứ giác là hình vuông ⇔ tứ giác có 2 đường chéo vuông góc.
B. △ ABC vuông ở B ⇔ BC 2 + AB2 = AC 2 .
C. a = b ⇔ a2 = b2 .
D. a < b ⇔ a2 < b2 .
Câu 10. Cho tập hợp A = {a; b; c; d }, phát biểu nào là sai?
A. a ∈ A . B. {a; d } ⊂ A . C. b ⊂ A . D. {d } ⊂ A .
Câu 11. Cho A = {a; b; c; d ; e}. Số tập con của A là
A. 10. B. 12. C. 32. D. 16.
Câu 12. Cho X = {n ∈ N | n là bội số của 4 và 6} và Y = {n ∈ N | n là bội số của 12}. Mệnh đề nào sau đây sai?
A. X ⊂ Y . B. Y ⊂ X . C. X = Y . D. ∃ n : n ∈ X và n ∉ Y .
Câu 13. Cho H là tập hợp các hình bình hành, V là tập hợp các hình vuông, N là tập hợp các hình chữ nhật,
T là tập hợp các hình thoi. Tìm mệnh đề sai.
A. V ⊂ T . B. V ⊂ N . C. H ⊂ T . D. N ⊂ H .

ÓNguyễn Thành Nhân 25 Chuyên Hùng Vương-BD


Các Bài Giảng 2023-2024 Đại Số 10 Chuyên

Câu 14. Cho M là tập hợp các tứ giác, N là tập hợp các hình bình hành, P là tập hợp các hình thang, Q là
tập hợp các hình chữ nhật. Khi đó
A. M ⊂ N ⊂ P ⊂ Q . B. N ⊂ M ⊂ Q ⊂ P . C. Q ⊂ N ⊂ P ⊂ M . D. P ⊂ Q ⊂ N ⊂ M .
Câu 15. Cho M là tập hợp các hình bình hành, N là tập hợp các hình thang, P là tập hợp các hình vuông,
Q là tập hợp các hình thoi. Khi đó
A. M ⊂ N ⊂ P ⊂ Q . B. M ⊂ P ⊂ N ⊂ Q . C. Q ⊂ P ⊂ N ⊂ M . D. P ⊂ Q ⊂ M ⊂ N .
Câu 16. Cho ba tập hợp A = {2; 5}, B = {5; x}, C = { x; y; 5}. Khi A = B = C thì
A. x = y = 2. B. x = y = 2 hay x = 2, y = 5.
C. x = 2, y = 5. D. x = 5, y = 2 hay x = y = 5.
Câu 17. Cho ba tập hợp E, F và G , biết E ⊂ F , F ⊂ G và G ⊂ E . Câu nào sau đây đúng?
A. G ⊂ F . B. F = G . C. E = G . D. E = F = G .

ĐÁP ÁN

1 C 3 D 5 C 7 D 9 B 11 C 13 C 15 D 17 C

2 A 4 B 6 A 8 B 10 C 12 D 14 C 16 B
LATEX:Nguyễn Thành Nhân

ÓNguyễn Thành Nhân 26 Chuyên Hùng Vương-BD


CHƯƠNG 2 BẤT PHƯƠNG TRÌNH, HỆ BẤT PHƯƠ
TRÌNH BẬC NHẤT HAI ẨN

BÀI 1. BẤT PHƯƠNG TRÌNH VÀ HỆ BẤT PHƯƠNG TRÌNH BẬC


NHẤT HAI ẨN
Nguyễn Thành Nhân- Chuyên Hùng Vương- Bình Dương

A. CƠ SỞ LÝ THUYẾT

1. Bất phương trình bậc nhất hai ẩn

a) Bất phương trình bậc nhất hai ẩn và miền nghiệm của nó


Định nghĩa 1. Bất phương trình bậc nhất hai ẩn là bất phương trình có một trong các dạng
1 ax + b y + c < 0; 2 ax + b y + c > 0;

3 ax + b y + c ≤ 0; 4 ax + b y + c ≥ 0,

TOÁN CHUYÊN
trong đó a, b và c là nhưng số cho trước và a2 + b2 > 0; x, y là các ẩn.
Mỗi cặp ( x0 ; y0 ) sao cho ax0 + b y0 + c < 0 được gọi là một nghiệm của bất phương trình ax + b y + c < 0.
Nghiệm của các bất phương trình ax + b y + c > 0, ax + b y + c ≤ 0, ax + b y + c ≥ 0 được định nghĩa tương tự.

b) Cách xác định miền nghiệm của bất phương trình bậc nhất hai ẩn
Định lí 1. Trong mặt phẳng tọa độ, đường thẳng d : ax + b y + c = 0 chia mặt phẳng thành hai nửa mặt phẳng.
Một trong hai nửa mặt phẳng ấy (không kể bờ d ) gồm các điểm có tọa độ thỏa mãn bất phương trình
ax + b y + c > 0, nửa mặt phẳng còn lại (không kể bờ d ) gồm các điểm thỏa mãn bất phương trình ax + b y + c < 0.
Từ định lý ta suy ra
Nếu ( x0 ; y0 ) là một nghiệm của bất phương trình ax + b y + c < 0 thì nửa mặt phẳng (không kể bờ d ) chứa điểm
M ( x0 ; y0 ) chính là miền nghiệm của bất phương trình ấy. Từ đó ta cách vẽ miền nghiệm của bất phương trình
ax + b y + c < 0 như sau:

• Vẽ đường thẳng d : ax + b y + c = 0;

• Xét một điểm M0 ( x0 ; y0 ) không nằm trên d ;

• Nếu ax0 + b y0 + c < 0 thì nửa mặt phẳng (không kể bờ d ) chứa điểm M là miền nghiệm của bất phương
trình ax + b y + c < 0.
Nếu ax0 + b y0 + c > 0 thì nửa mặt phẳng (không kể bờ d ) không chứa điểm M là miền nghiệm của bất
phương trình ax + b y + c < 0.
Chú ý 2. Đối với các bất phương trình ax + b y + c ≤ 0, ax + b y + c ≥ 0 thì miền nghệm là nửa mặt phẳng kể cả
bờ.

VÍ DỤ 1. Biểu diễn hình học miền nghiệm của các bất phương trình

1 2 x − y + 4 < 0; 2 3 x − 2 y + 5 ≥ 0.

2. Hệ bất phương trình bậc nhất hai ẩn


Định nghĩa 2. Hệ bất phương trình bậc nhất hai ẩn là hệ gồm nhiều bất phương trình bậc nhất hai ẩn.
Ta biểu diễn miền nghiệm của hệ bất phương trình bậc nhất hai ẩn như sau
- Với mỗi bất phương trình trong hệ, ta xác định miền nghiệm của nó và gạch bỏ miền còn lại.

27
Các Bài Giảng 2023-2024 Đại Số 10 Chuyên

- Sau khi làm như trên lần lượt đối với các bất phương trình còn lại của hệ cùng một mặt phẳng tọa độ,
miền còn lại không bị gạch chính là miền nghiệm của hệ bất phương trình đã cho.


3 x − y + 3 > 0

VÍ DỤ 2. Xác định miền nghiệm của hệ bất phương trình − 2 x + 3 y − 6 < 0

2 x + y + 4 > 0.

3. Áp dụng vào bài toán kinh tế


Vấn đề tìm miền nghiệm của hệ bất phương trình bậc nhất có liên quan chặt chẽ đến Quy hoạch tuyến tính.
Ở Việt Nam, cố giáo sư Hoàng Tụy nổi tiếng với lĩnh vực tối ưu và là cha đẻ của lý thuyết tối ưu toàn cục.

BÀI TOÁN
Người ta dự định dùng hai loại nguyên liệu để chiết xuất ít nhất 140 kg chất A và 9 kg chất B. Từ mỗi tấn
nguyên liệu loại I giá 4 triệu đồng, có thể chiết xuất được 20 kg chất A và 0, 6 kg chất B. Từ mỗi tấn nguyên
liệu loại I I giá 3 triệu đồng, có thể chiết suất được 10 kg chất A và 1, 5 kg chất B. Hỏi phải dùng bao nhiêu
LATEX:Nguyễn Thành Nhân

tấn nguyên liệu mỗi loại để chi phí mua nguyên liệu là ít nhất, biết rằng cơ sở cung cấp nguyên liệu chỉ có
thể cung cấp không quá 10 tấn nguyên liệu loại I và 9 tấn nguyên liệu loại I I ? ĐS: Loại I: 5 tấn; Loại II: 4
tấn.

B. BÀI TẬP
BÀI 1. Xác định miền nghiệm của các hệ bất phương trình bậc nhất hai ẩn
 
x + y > 0 2 x − 3 y − 6 ≤ 0
(
x+ y−2 ≥ 0
 
1 ; 2 2x − 3 y + 6 > 0 3 x+ y+2 > 0
x − 3y + 3 ≤ 0  
x − 2 y + 1 ≥ 0. x − 2 y + 3 ≤ 0.
 

BÀI 2. Tìm giá trị lớn nhất, giá trị nhỏ nhất của biểu thức F ( x; y) = 30 x − 4 y − 6 với ( x; y) thỏa mãn hệ bất

 x− y+5 ≥ 0

2 x + y + 4 ≥ 0

phương trình


 x+ y−5 ≤ 0

2 x − y − 4 ≤ 0.

BÀI 3. Một công ty kinh doanh chuẩn bị một đợt khuyến mãi nhằm thu hút khách hàng bằng cách tiến hành
quảng cáo trên đài phát thanh và truyền hình. Chi phí cho một phút quảng cáo trên phát thanh là 800.000
đồng, trên truyền hình là 4000.000 đồng. Đài phát thanh chỉ nhận các chương trình quảng cáo có độ dài ít
nhất là 5 phút, đài truyền hình nhận các chương trình quảng cáo có độ dài tối đa là 4 phút. Theo phân tích
thì cùng thời lượng phát sóng, hiệu quả của quảng cáo trên truyền hình gấp 6 lần trên phát thanh. Công ty
dự định chi tối đa 16.000.000 đồng cho quảng cáo. Hỏi công ty cần đặt thời lượng trên sóng phát thanh và
trên truyền hình như thế nào để sao cho hiệu quả đem lại do quảng cáo là cao nhất.
ĐS: Phát thanh: 5 phút; Truyền hình: 3 phút.
BÀI 4. Một xưởng sản xuất hai loại sản phẩm, mỗi kg sản phẩm loại I cần 2 kg nguyên liệu và 30 giờ, đem
lại mức lời 40.000 đồng. Mỗi kg sản phẩm loại II cần 4 kg nguyên liệu và 15 giờ, đem lại mức lời 30.000 đồng.
Xưởng có 200 kg nguyên liệu và 1200 giờ làm việc. Nên sản xuất mỗi loại sản phẩm bao nhiêu để có mức lời
cao nhất?
ĐS: 20 kg loại I; 40 kg loại II.
BÀI 5. Nhân dịp tết Trung Thu, xí nghiệp sản xuất bánh Kinh Đô muốn sản xuất hai loại bánh: Đậu xanh,
Bánh dẻo nhân đậu xạnh. Để sản xuất hai loại bánh này, xí nghiệp cần: Đường, Đậu, Bột, Trứng, Mứt. Giả
sử số đường có thể huy động được là 300 kg đậu là 200 kg, các nguyên liệu khác bao nhiêu cũng có. Sản xuất
một cái bánh đậu xanh cần 0, 06 kg đường, 0, 08 kg đậu và cho lãi 2 ngàn đồng. Sản xuất một cái bánh dẻo
cần 0, 07 kg đường, 0, 04 kg đậu và cho lãi 1, 8 ngàn đồng. Cần lập kế hoạch để sản xuất mỗi loại bánh bao
nhiêu cái để không bị động về đường, đậu và tổng số lãi thu được là lớn nhất (nếu sản xuất bao nhiêu cũng
bán hết).
ĐS: 625 bánh đậu xanh; 3750 bánh dẻo.

ÓNguyễn Thành Nhân 28 Chuyên Hùng Vương-BD


Các Bài Giảng 2023-2024 Đại Số 10 Chuyên

BÀI 6. Một công ty cần thuê xe vận chuyển 140 người và 9 tấn hàng hóa. Nơi cho thuê xe chỉ có 10 xe hiệu
Toyota và 9 xe hiệu Ford. Một xe hiệu Toyota có thể chở 20 người và 0, 6 tấn hàng hóa. Một chiếc xe hiệu Ford
có thể chở 10 người và 1, 5 tấn hàng hóa. Tiền thuê một xe hiệu Toyota giá 4 triệu đồng, một xe hiệu Ford
tiền thuê giá 3 triệu đồng. Hỏi phải thuê bao nhiêu xe mỗi loại để chi phí thấp nhất. ĐS: 5 xe hiệu Toyota; 4
xe hiệu Ford.
BÀI 7. Cho x, y là các số thực thỏa mãn đồng thời các điều kiện −1 ≤ x ≤ 3 và −2 ≤ y ≤ 5. Biết rằng khi đó biểu
thức F = 2 x − 3 y + 2019 đạt giá trị lớn nhất tại ( x; y) = (a; b). Tính S = a + b.
A. S = 8. B. S = 14. C. S = −3. D. S = 1.

BÀI TẬP BẤT PHƯƠNG TRÌNH VÀ HỆ BẤT PHƯƠNG TRÌNH


BẬC NHẤT HAI ẨN
Nguyễn Thành Nhân- Chuyên Hùng Vương- Bình Dương

CÁC BÀI TOÁN


BÀI 1. Bểu diễn miền nghiệm của bất phương trình x + 2 y − 8 ≤ 0 trên mặt phẳng tọa độ Ox y.
BÀI 2. Một phân xưởng cần sản xuất ra hai loại sản phẩm. Để sản xuất 1 kilogam sản phẩm loại I cần sử
dụng máy trong 20 giờ và tiêu tốn 2 kilogam nguyên liệu. Để sản xuất 1 kilogam sản phẩm loại I I cần sử
dụng máy trong 10 giờ và tiêu tốn 3 kilogam nguyên liệu. Biết rằng 1 kilogam sản phẩm loại I thu lãi được
50 000 đồng, 1 kilogam sản phẩm loại I I thu lãi được 40 000 đồng, có thể sử dụng máy tối đa 1 200 giờ và có
300 kilogam nguyên liệu. Hỏi phân xưởng đó nên sản xuất mỗi loại sản phẩm bao nhiêu kilogam để thu lãi
cao nhất.

TOÁN CHUYÊN
BÀI 3. Biểu diễn miền nghiệm của bất phương trình 2 x − 5 y − 10 < 0.
BÀI 4. Một bạn học sinh lớp 10 muốn làm 2 loại sản phẩm A và B để tham gia hội Xuân. Biết rằng mỗi sản
phẩm loại A cần 100 ngàn đồng tiền nguyên liệu, 2 giờ công và bán được 450 ngàn đồng; mỗi sản phẩm loại
B cần 200 ngàn đồng tiền nguyên liệu, 3 giờ công và bán được 750 ngàn đồng. Bạn có 700 ngàn đồng tiền vốn
và có 12 giờ chuẩn bị. Hỏi bạn ấy cần làm bao nhiêu sản phẩm mỗi loại để số tiền thu được là lớn nhất?

x + y ≤ 4

BÀI 5. Biểu diễn miền nghiệm của hệ bất phươnng trình x ≥ 1 trên mặt phẳng (Ox y).

y≥1

BÀI 6. Một gia đình sản xuất gạo, do điều kiện nhà xưởng nên mỗi đợt gia đình đó sản xuất được x kg gạo.
Biết rằng mỗi kí bán được với giá 350 − 5 x (nghìn đồng) và chi phí sản xuất x kg gạo là x2 + 5 x + 1000 (nghìn
đồng). Hỏi gia đình đó phải sản xuất được bao nhiêu kg gạo để đạt được lợi nhuận tối đa?

2 x + 3 y ≤ 12

BÀI 7. Biểu diễn miền nghiệm của hệ bất phương trình x ≥ 0

y ≥ 0.


x ≥ 0

BÀI 8. Tìm miền nghiệm của hệ bất phương trình y ≤ 0

2 x − 3 y − 6 ≤ 0.

BÀI 9. Một bác nông dân cần trồng lúa và khoai trên diện tích đất gồm 6 ha, với lượng phân bón dự trữ là
100 kg. Để trồng 1 ha lúa cần sử dụng 20 kg phân bón với lợi nhuận là 30 triệu đồng, để trồng 1 ha khoai
cần sử dụng 10 kg phân bón với lợi nhuận là 20 triệu đồng. Để đạt được lợi nhuận cao nhất, bác nông dân
đã trồng x ha lúa và y ha khoai ( x, y ≥ 0). Tìm giá trị của x.
BÀI 10. Có ba nhóm máy A, B, C dùng để sản xuất ra hai loại sản phẩm I và I I . Để sản xuất một đơn vị sản
phầm thuộc mỗi loại phải lần lượt dùng các máy thuộc các nhóm khác nhau. Số máy trong một nhóm và số
máy của từng nhóm cần thiết để sản xuất ra một đơn vị sản phẩm thuộc mỗi loại được cho trong bảng sau

Nhóm Số máy Loại I Loại II

A 10 2 2

B 4 0 2

C 12 2 4

ÓNguyễn Thành Nhân 29 Chuyên Hùng Vương-BD


Các Bài Giảng 2023-2024 Đại Số 10 Chuyên

Mỗi đơn vị sản phẩm I lãi 4 000 đồng, mỗi đơn vị sản phẩm I I lãi 7 000 đồng. Để việc sản xuất hai loại sản
phẩm trên có lãi cao nhất thì cần dùng đến bao nhiêu máy thuộc nhóm A ?
BÀI 11. Biểu diễn miền nghiệm của bất phương trình x + 2 y − 2 < 0.
BÀI 12. Một gia đình cần ít nhất 900 đơn vị protein và 400 đơn vị lipit trong thức ăn mỗi ngày. Mỗi kg thịt
bò chứa 800 đơn vị protein và 200 đơn vị lipit. Mỗi kg thịt lợn chứa 600 đơn vị proteinvà 400 đơn vị lipit. Biết
rằng gia đình này chỉ mua nhiều nhất 1,6 kg thịt bò và 1,1 kg thịt lợn, giá tiền mỗi kg thịt bò là 250.000
đồng, giá tiền mỗi kg thịt lợn là 85.000 đồng. Hỏi chi phí ít nhất để mua thịt mỗi ngày của gia đình đó là bao
nhiêu?
BÀI 13.
1 Biểu diễn miền nghiệm của bất phương trình 3( x − 1) + 4( y − 2) < 5 x − 3.

2 Giá trị nhỏ nhất Fmin của biểu thức F ( x; y) = y − x trên miền nghiệm xác định bởi hệ bất phương trình

 y − 2x ≤ 2

2y − x ≥ 4

x y ≤ 5.

BÀI 14. Béo phì hiện nay là một vấn đề y tế đáng quan tâm trong các nước đang phát triển, trong đó có Việt
Nam. Để xác định sự béo phì, cần phải căn cứ vào việc đo tỷ lệ chất béo có trong cơ thể. Phương pháp chuẩn
để đo tỷ lệ chất béo là dùng máy X quang có tên là Dual-energy X-ray Absorptiometry (viết tắt là DXA), tuy
nhiên máy này còn quá đắt tiền đối với các cơ sở y tế nên người ta đã nghiên cứu ra công thức đo tỷ lệ chất
LATEX:Nguyễn Thành Nhân

735
mỡ ở nam giới như sau: P = 51,6 − + 0,029T trong đó T là số tuổi, X là chỉ số khối của cơ thể. Đối với nam
X
giới, chỉ số P > 25 thì xem như béo phì và cần kiểm soát chất mỡ.
Bạn Huy (cao 172cm, cân nặng 82kg) và bạn Nam (cao 160cm, cân nặng 70kg) có chỉ số khối của cơ thể lần
lượt là 27,7 và 27,3. Tổng số tuổi hiện tại của hai bạn Huy và Nam là 32. Tuổi của bạn Huy sau 13 năm nữa
gấp đôi tuổi của bạn Nam hiện tại. Hãy tìm số tuổi của mỗi bạn và cho biết bạn nào bị béo phì?
5x + y − 1
BÀI 15. Biểu diễn miền nghiệm của bất phương trình ≥ x + y + 1.
2
BÀI 16. Một người thợ mộc làm những cái bàn và những cái ghế. Mỗi cái bàn khi bán lãi 150 nghìn đồng,
mỗi cái ghế khi bán lãi 50 nghìn đồng. Người thợ mộc có thể làm 40 giờ/tuần và tốn 6 giờ để làm một cái bàn,
3 giờ để làm một cái ghế. Khách hàng yêu cầu người thợ mộc làm số ghế ít nhất là gấp ba lần số bàn. Một
cái bàn chiếm chỗ bằng 4 cái ghế và ta có phòng để được nhiều nhất 4 cái bàn/tuần. Hỏi người thợ mộc phải
sản xuất như thế nào để số tiền lãi thu về là lớn nhất.
BÀI 17. Một hộ nông dân dự định trồng đậu và cà trên diện tích 8 ha. Nếu trồng đậu thì cần 20 công và thu
3 triệu đồng trên diện tích mỗi ha, nếu trồng cà thì cần 30 công và thu 4 triệu đồng trên diện tích mỗi ha.
Hỏi cần trồng mỗi loại cây trên với diện tích là bao nhiêu để thu về được nhiều tiền nhất, biết rằng tổng số
công không quá 180.
BÀI 18. Biểu diễn miền nghiệm của bất phương trình 2 x − y + 3 ≥ 0.
BÀI 19. Một nhóm học sinh tiến hành làm đèn trung thu hình ngôi sao và hình con cá để bán. Nhóm học
sinh nam làm một cái đèn ông sao cần 2 giờ và đèn con cá cần 3 giờ, nhóm học sinh nữ làm đèn ông sao cần
4 giờ và đèn con cá cần 2 giờ. Trong 1 tuần nhóm học sinh nam làm việc không quá 24 giờ và nhóm học sinh
nữ không quá 26 giờ. Hãy tính số tiền lãi nhiều nhất khi nhóm học sinh bán hết toàn bộ số đèn lồng làm
được, biết rằng mỗi cái đèn ông sao bán được sẽ lời 75 000 đồng và mỗi cái đèn con cá bán được sẽ lời 60 000
đồng.
BÀI 20.
x− y
1 Xác định miền nghiệm của bất phương trình sau < x + y + 1.
−2
2 Một công ty kinh doanh thương mại chuẩn bị cho một đợt khuyến mại nhằm thu hút khách hàng bằng
cách tiến hành quảng cáo sản phẩm của công ty trên hệ thống phát thanh và truyền hình. Chi phí cho
1 phút quảng cáo trên sóng phát thanh là 800.000 đồng, trên sóng truyền hình là 4.000.000 đồng. Đài
phát thanh chỉ nhận phát các chương trình quảng cáo dài ít nhất là 5 phút. Do nhu cầu quảng cáo
trên truyền hình lớn nên đài truyền hình chỉ nhận phát các chương trình dài tối đa là 4 phút. Theo
các phân tích cùng thời lượng một phút quảng cáo trên truyền hình sẽ có hiệu quả gấp 6 lần trên sóng
phát thanh. Công ty dự định chi tối đa là 16.000.000 đồng cho quảng cáo. Công ty cần đặt thời lượng
quảng cáo trên sóng phát thanh và truyền hình như thế nào để hiệu quả nhất?
BÀI 21. Một khách sạn có 50 phòng. Hiện tại mỗi phòng cho thuê với giá 400 ngàn đồng một ngày thì toàn
bộ phòng được thuê hết. Biết rằng cứ mỗi lần tăng giá thêm 20 ngàn đồng thì có thêm 2 phòng trống. Giám
đốc phải chọn giá phòng mới là bao nhiêu để thu nhập của khách sạn trong ngày là lớn nhất.

ÓNguyễn Thành Nhân 30 Chuyên Hùng Vương-BD


Các Bài Giảng 2023-2024 Đại Số 10 Chuyên

BÀI 22. Trong một cuộc thi gói bánh vào dịp năm mới, mỗi đội chơi được sử dụng tối đa 25 kg nếp, 3 kg thịt
ba chỉ, 5 kg đậu xanh để gói bánh chưng và bánh ống (bánh tét). Để gói một cái bánh chưng cần 0,4 kg gạo
nếp, 0,05 kg thịt, 0,1 kg đậu xanh; để gói một cái bánh ống cần 0,6 kg gạo nếp, 0,075 kg thịt, 0,1 kg đậu xanh.
Mỗi cái bánh chưng nhận được 5 điểm thưởng, mỗi cái bánh ống nhận được 7 điểm thưởng. Hỏi cần phải gói
mấy cái bánh mỗi loại để đạt được điểm thưởng cao nhất?
BÀI 23. Một xưởng cơ khí có hai công nhân là Chiến và Bình. Xưởng sản xuấ loại sản xuất loại I và I I . Mỗi
sản phẩm I bán lãi 500 nghìn đồng, mỗi sản phẩm I I bán lãi 400 nghìn đồng. Để sản xuất được một sản
phẩm I thì Chiến phải làm việc trong 3 giờ, Bình phải làm việc trong 1 giờ. Để sản xuất được một sản phẩm
I I thì Chiến phải làm việc trong 2 giờ, Bình phải làm việc trong 6 giờ. Một người không thể làm được đồng
thời hai sản phẩm. Biết rằng trong một tháng Chiến không thể làm việc quá 180 giờ và Bình không thể làm
việc quá 220 giờ. Tính số tiền lãi lớn nhất trong một tháng của xưởng.
BÀI 24. Một xưởng sản xuất có 2 máy đặc chủng A và B để sản xuất 2 loại sản phẩm X và Y . Để sản xuất 1
tấn sản phẩm loại X cần dùng máy A trong 6 giờ và dùng máy B trong 2 giờ. Để sản xuất 1 tấn sản phẩm
loại Y cần dùng máy A trong 2 giờ và dùng máy B trong 2 giờ. Cho biết mỗi máy không thể sản xuất đồng
thời 2 loại sản phẩm. Máy A làm việc không quá 12 giờ 1 ngày, máy B làm việc không quá 8 giờ 1 ngày. Một
tấn sản phẩm loại X lãi 10 triệu đồng và 1 tấn sản phẩm loại Y lãi 8 triệu đồng. Hãy lập kế hoạch sản xuất
mỗi ngày sao cho tiền lãi thu được là lớn nhất.

TOÁN CHUYÊN

ÓNguyễn Thành Nhân 31 Chuyên Hùng Vương-BD


LATEX:Nguyễn Thành Nhân Các Bài Giảng 2023-2024 Đại Số 10 Chuyên

ÓNguyễn Thành Nhân 32 Chuyên Hùng Vương-BD


CHƯƠNG 3 HÀM SỐ BẬC HAI VÀ ĐỒ THỊ

BÀI 1. KHÁI NIỆM HÀM SỐ


Nguyễn Thành Nhân- Chuyên Hùng Vương- Bình Dương

A. LÝ THUYẾT CẦN NẮM


1. Định nghĩa
Khi có hai đại lương x, y cùng biến thiên trong R, trong đó mỗi giá trị của x cho ta một và chỉ một giá trị
tương ứng của y, khi đó ta xác lập một hàm số y theo x.

1 Chu vi đường tròn y thay đổi theo bán kính R theo công thức y = 2πR .

2 Doanh số bán ra của một công ty theo từng tháng của một năm nào đó.

3 Biểu đồ biểu diễn lượng mưa trong một năm theo từng tháng.

4 Dân số gần đúng của một quốc gia qua từng năm.

Tất cả các ví dụ trên đều xác lập một hàm số.


Định nghĩa 1. Cho tập D ⊂ R và D ̸= ∅. Hàm số f xác định trên D là một quy tắc đặt tương ứng mỗi số x
thuộc D với một và chỉ một số, ký hiệu là f ( x); số f x) đó được gọi là giá trị của hàm số f tại x (đọc là f ( x)

TOÁN CHUYÊN
hay f của x). Vậy hàm số là một ánh xạ từ tập con D của R vào R và viết

f: D →R
x 7→ f ( x)

Tập D được gọi là tập xác định (hay miền xác định), x được gọi là biến số hay đối số của hàm số f .
Tập hợp tất cả các giá trị của f ( x) khi x chạy qua tất cả các giá trị của D được gọi là miền giá trị của hàm số
f.
Khi y = f ( x) thì ta gọi x là biến số độc lập còn y gọi là biến số phụ thuộc.

2. Các phương pháp biểu diễn hàm số


Một hàm số có thể được biểu diễn bằng một trong bốn cách sau đây

1 Mô tả bằng lời. 2 Bằng đồ thị.

3 Bằng bảng (bảng biểu). 4 Bằng công thức tường minh.

VÍ DỤ 1. Lấy một số ví dụ về hàm số.

3. Đồ thị của hàm số và phép tịnh tiến đồ thị


Định nghĩa 2.
Cho hàm số y = f ( x) xác định trên tập D. Đồ thị của hàm số f là tập tất cả các y
điểm M ( x; f ( x)) trong mặt phẳng tọa độ Ox y.

O x
−4 2 4

33
Các Bài Giảng 2023-2024 Đại Số 10 Chuyên

Quy tắc tịnh tiến đồ thị


Cho hàm số y = f ( x) có đồ thị (C ) và các số a, b dương. Khi đó
• Đồ thị của hàm số y = f ( x + a) được suy ra bằng cách tịnh tiến đồ thị (C ) sang trái a đơn vị.

• Đồ thị của hàm số y = f ( x − a) được suy ra bằng cách tịnh tiến đồ thị (C ) sang phải a đơn vị.

• Đồ thị của hàm số y = f ( x) + b được suy ra bằng cách tịnh tiến đồ thị (C ) lên trên b đơn vị.

• Đồ thị của hàm số y = f ( x) − b được suy ra bằng cách tịnh tiến đồ thị (C ) xuống dưới b đơn vị.

VÍ DỤ 2. Lấy một số ví dụ về đồ thị hàm số đã học.

4. Tập xác định của hàm số cho bởi công thức


Định nghĩa 3. Tập xác định của hàm số được cho bởi công thức y = f ( x) là tập tất cả các giá trị thực của biến
số x để biểu thức f ( x) được xác định.

VÍ DỤ 3. Tìm tập xác định của các hàm số sau


LATEX:Nguyễn Thành Nhân

p p
x + 2019 − 2020 − x 2019
1 y= . 2 y= ¡ ¢ p .
x2 − x − 2 | x|2 − 10| x| + 9 · x − 8

VÍ DỤ 4. Tìm m để các hàm số sau đây xác định với mọi x thuộc khoảng (0; +∞).
p p
1 y= x − m + 2 x − m − 1. ĐS: m ≤ −1.
p x−m 4
2 y= 2 x − 3m + 4 + . ĐS: 1 ≤ m ≤ .
x+m−1 3

VÍ DỤ 5. Tìm m để các hàm số sau xác định trên R.


2x + 1
1 y= p . ĐS: m > 11.
x2 − 6 x + m − 2
p
m+1 1
2 y= 2 . ĐS: m > .
3x − 2x + m 3

VÍ DỤ 6. Tìm m để các hàm số sau xác định trên tập đã chỉ ra


p 1
1 y= x+m−1− p xác định trên (−1; 3). ĐS: m ≥ 2.
2m − x
1 p
2 p − − x − 2 m + 6 xác định trên (−1; 0). ĐS: 1 ≤ m ≤ 3.
x+m

5. Sự đồng biến nghịch biến của hàm số


Định nghĩa 4. Cho hàm số xác định trên tập K .
1 Hàm số f được gọi là đồng biến (hay tăng) trên K nếu

∀ x1 , x2 ∈ K , x1 < x2 ⇒ f ( x1 ) < f ( x2 ).

2 Hàm số f được gọi là nghịch biến (hay giảm) trên K nếu

∀ x1 , x2 ∈ K , x1 < x2 ⇒ f ( x1 ) > f ( x2 ).

3 Hàm số đồng biến hoặc nghịch biến trên K được gọi là hàm số đơn điệu trên K .

ÓNguyễn Thành Nhân 34 Chuyên Hùng Vương-BD


Các Bài Giảng 2023-2024 Đại Số 10 Chuyên

Định nghĩa 5. Nếu f ( x1 ) = f ( x2 ) = c, ∀ x1 , x2 ∈ K thì f được gọi là hàm số không đổi (hàm hằng) trên K .
Ta viết y = c.
Chú ý 3.
• Đồ thị của hàm số đồng biến trên K thì đi lên từ trái qua phải (chiều tăng của biến số x).

• Đồ thị của hàm số nghịch biến trên K thì đi xuống từ trái qua phải (chiều tăng của biến số x).

• Đồ thị của hàm hằng thì nằm ngang.


Nhận xét 4. Ta có kết quả sau đây
1 Hàm số f đồng biến trên K khi và chỉ khi
f ( x1 ) − f ( x2 )
∀ x1 , x2 ∈ K , x1 ̸= x2 : T = > 0.
x1 − x2

2 Hàm số f nghịch biến trên K khi và chỉ khi


f ( x1 ) − f ( x2 )
∀ x1 , x2 ∈ K , x1 ̸= x2 : T = < 0.
x1 − x2

VÍ DỤ 7. Lấy một số ví dụ về xét tính đơn điệu của hàm số.

6. Hàm số chẵn, hàm số lẻ


Định nghĩa 6. Cho hàm số y = f ( x) xác định trên tập D.

TOÁN CHUYÊN
• Hàm số f được gọi là hàm số chẵn nếu với mọi x ∈ D, ta có − x ∈ D và f (− x) = f ( x).

• Hàm số f được gọi là hàm số lẻ nếu với mọi x ∈ D, ta có − x ∈ D và f (− x) = − f ( x).

VÍ DỤ 8. Lấy ví dụ về hàm số chẵn và hàm số lẻ.

Chú ý 4.
1 Có những hàm số không chẵn, không lẻ.

2 Hàm số chẵn hoặc lẻ thì phải có tập xác định đối xứng qua số 0.

Đồ thị của hàm số chẵn và hàm số lẻ

Định lí 1.
• Đồ thị của hàm số chẵn nhận trục tung làm trục đối xứng.

• Đồ thị của hàm số lẻ nhận gốc tọa độ làm tâm đối xứng.
y y

y = f ( x)

−2 2 x
O
O x

Chú ý 5. Một hàm số có tập xác định là tập đối xứng thì luôn biểu diễn được thành tổng của một hàm số
chẵn và một hàm số lẻ.

7. Hàm số hợp
Xem lại ánh xạ hợp.

ÓNguyễn Thành Nhân 35 Chuyên Hùng Vương-BD


Các Bài Giảng 2023-2024 Đại Số 10 Chuyên

VÍ DỤ 9. Xét tính chẵn lẻ của các hàm số sau


p
| x − 1| + | x + 1| x2 − 8 x + 16 x6 + 1
1 f ( x) = . 2 f ( x) = . 3 f ( x) = .
| x − 1| − | x + 1| | x| − 1 x5

VÍ DỤ 10. Tìm m để hàm số f ( x) = x4 + 2(m2 − 1) x3 + 2 x2 + ( m + 1) x + 2019 là hàm số chẵn. ĐS: m = −1.

BÀI 2. HÀM SỐ BẬC NHẤT


Nguyễn Thành Nhân- Chuyên Hùng Vương- Bình Dương

A. CƠ SỞ LÝ THUYẾT

1. 1. Nhắc lại về hàm số bậc nhất


1 Hàm số bậc nhất được cho bởi công thức y = ax + b trong đó a ̸= 0.
LATEX:Nguyễn Thành Nhân

2 Tập xác định là D = R, tập giá trị T = R.

3 Hàm số đồng biến trên R khi a > 0, nghịch biến trên R khi a < 0.

4 Hệ số a được gọi là hệ số góc, a = tan ϕ với ϕ là góc xAt


d trong đó t là giao điểm của đồ thị với trục hoành,
tia At nằm trên trục hoành.

5 Đồ thị hàm số y = ax + b ta còn gọi là đường thẳng y = ax + b ( d ).


Xét hai đường thẳng y = ax + b (d ) và y = a′ x + b′ (d ′ ) với a · a′ ̸= 0. Ta có các kết quả sau:
a = a′ a = a′
( (
′ ′
• d∥d ⇔ ′ • d≡d ⇔
b ̸= b . b = b′ .

• d cắt d ′ ⇔ a ̸= a′ . • d ⊥ d ′ ⇔ a · a′ = −1.

VÍ DỤ 1. Tìm m để hàm số y = (m − 2019) x + 2020

1 Đồng biến trên R. 2 Nghịch biến trên R.

VÍ DỤ 2. Một hình chữ nhật có các kích thước là 40 cm và 50 cm. Người ta thêm vào mỗi kích thước
của hình đó đi x cm được hình chữ nhật mới có chu vi là y cm. Hãy lập công thức tính y theo x.

VÍ DỤ 3. Tìm m để các hàm số sau đây là hàm số bậc nhất


p m−1
1 y= 2 m − 4( x − 2). 2 y= ( x + 3). 3 y = ( m − 3) x2 + (3 m − 6) + 3.
m+1

VÍ DỤ 4.
2 2
1 Vẽ đồ thị các hàm số y = 2 x, y = 2 x + 5, y = − x và y = − x + 5 trên cùng một mặt phẳng tọa độ.
3 3
2 Bốn đường thẳng trên cắt nhau tạo thành tứ giác O ABC với O là gốc tọa độ. Tứ giác O ABC có
phải là hình bình hành không? Vì sao?

VÍ DỤ 5. Cho hai hàm số bậc nhất y = (m − 1) x + 3 và y = (3 m − 5) x − 2. Tìm m để


1 Hai đường thẳng cắt nhau. 2 Hai đường thẳng song song.

ÓNguyễn Thành Nhân 36 Chuyên Hùng Vương-BD


Các Bài Giảng 2023-2024 Đại Số 10 Chuyên

2. Hàm số y = |ax + b|
Để vẽ đồ thị hàm số = |ax + b| ta có thể vẽ hai đồ thị các hàm số y = ax + b và y = −ax − b rồi xóa đi phần phía
dưới trục hoành (vì y = |ax + b| ≥ 0).

VÍ DỤ 6. Vẽ đồ thị của các hàm số trị tuyệt đối


1 y = | x − 1|.

2 y = | x + 1| + |2 x − 4|.

3 Tìm m để phương trình | x + 1| + |2 x − 4| = m có đúng hai nghiệm.

VÍ DỤ 7.

1 Vẽ đồ thị các hàm số y = x và y = 2 x + 2.

2 Gọi A là giao điểm của hai đồ thị trên, tìm tọa độ điểm A .

3 Vẽ qua B(0; 2) một đường thẳng song song với trục Ox, cắt đường thẳng y = x tại điểm C . Tìm tọa
độ điểm C rồi tính diện tích tam giác ABC .

VÍ DỤ 8. Công ty điên lực Bình Dương tính giá cước tiền điền hàng tháng của hộ gia đình theo các
mức giá như sau. Nếu mỗi tháng tiêu thụ từ 0 Kw/h đến 100 Kw/h thì đơn giá mỗi Kw/h là 3 nghìn
đồng. Nếu mỗi tháng tiêu thụ từ trên 100 Kw/h đến 200 Kw/k thì đơn giá của mỗi Kw/h lớn hơn 100 là
4 nghìn đồng. Nếu mỗi tháng tiêu thụ lớn hơn 200 Kw/h thì đơn giá mỗi Kw/h lớn hơn 200 là 5 nghìn

TOÁN CHUYÊN
đồng. Gọi x là số Kw/h tiêu thụ trong một tháng, y là số tiền (nghìn đồng) mà hộ gia đình phải trả
trong tháng đó. Hãy lập hàm số biểu diễn sự phụ của y theo x. Tính y(50), y(150), y(250).

3. Một tính chất đặc trưng của hàm số bậc nhất (phần mở rộng)
Hoc sinh cần chú ý tính chất quan trọng sau đây của hàm số bậc nhất để giải quyết vấn đề bất phương trình
bậc nhất không đổi dấu trên một đoạn nào đó.
Tính chất 1. Cho hàm số y = f ( x) = ax + b. Khi đó

1 f ( x) ≥ 0 với mọi x ∈ [ m; n] khi và chỉ khỉ min{ f ( m); f ( n)} ≥ 0.

2 f ( x) ≤ 0 với mọi x ∈ [ m; n] khi và chỉ khỉ max{ f ( m); f ( n)} ≤ 0.

Cái đặc sắc của tính chất này là chuyển từ điều kiện trên một đoạn với vô số điểm về tính tại hai đầu mút
của đoạn đó chỉ với hai điểm. Nếu như chưa thể xác định được f (m), f (n) giá trị nào lớn hơn, giá trị nào nhỏ
hơn thì ta tính luôn cả hai giá trị f (m) và f (n).

VÍ DỤ 9. Tìm m để các bất phương trình


1 (2 m − 1) x + 2 m − 2 ≤ 0 với mọi x ∈ [−1; 1]. 2 ( m + 1) x − 4 m − 3 ≥ 0 với mọi x ∈ [0; 2].

B. BÀI TẬP LUYỆN TẬP


BÀI 1. Lập phương trình đường thẳng đi qua hai điểm A ( x1 ; y1 ), B( x2 ; y2 ).
Áp dụng: Cho ba điểm A (−1; −5); B(2; 4); C (−2; −8). Chứng minh rằng ba điểm A , B, C thẳng hàng.
3 x 19
BÀI 2. Cho đường thẳng y = + . Đường thẳng d cắt hai trực tọa độ Ox, O y lần lượt tại B, C . Tìm các
2 2
điểm có tọa độ thuộc đoạn thẳng BC .
BÀI 3. Chứng minh rằng đồ thị của các hàm số sau đây luôn đi qua một điểm cố định dù m nhận giá trị nào

1 y = ( m − 2) x − 2 m + 5. 2 y = (2 m − 1) x + 3 m − 4.

BÀI 4. Chứng minh rằng đồ thị của hàm số y = mx + 2m − 3 luôn đi qua một điểm cố định thuộc đường thẳng
1
y= x − 2.
2

ÓNguyễn Thành Nhân 37 Chuyên Hùng Vương-BD


Các Bài Giảng 2023-2024 Đại Số 10 Chuyên

BÀI 5. Tìm điểm cố định mà đồ thị hàm số y = f ( x) = mx − 3 m + 4 ( d ) luôn đi qua. Chứng minh rằng khoảng
cách từ gốc tọa độ O đến đường thẳng y = f ( x) không vượt quá 5.
BÀI 6. Chứng minh rằng khi m thay đổi, giao điểm của hai đường thẳng sau đây luôn thuộc một đường
thẳng cố định.
1 y = 3 x + m − 2 ( d ) và y = 4 x − m − 2 ( d ′ ). 2 y = −2 x + 3 m + 1 ( d ) và y = − x − 3 m + 1 ( d ′ ).

BÀI 7. Tìm m để sao cho bất phương trình 2 x + 3m − 1 ≥ 0 với mọi x ∈ [0; 2019].
BÀI 8. Tìm x sao cho bất phương trình x2 + 2mx − 3 m ≥ 0 nghiệm đúng với mọi m ∈ [−4; 2].
2−m 1
BÀI 9. Cho hàm số y = x+ với m ̸= 1 và m ̸= 2. Xác định m để khoảng cách từ gốc tọa độ O đến đồ
m−1 m−1
3
thị hàm số đạt giá trị lớn nhất. ĐS: m = .
2
BÀI 10. Cho hàm số y = (m2 − 2m + 3) x + 6 có đồ thị là đường thẳng (d ).
1 Chứng minh rằng với mọi m, hàm số đã cho luôn là hàm số bậc nhất.

2 Tìm m để ( d ) cắt Ox và O y lần lượt tại A và B sao cho S OAB lớn nhất. ĐS: m = 1.
2
BÀI 11. Chứng minh rằng với mọi m ≤ 1 thì bất phương trình x − 2(3m − 1) x + m + 3 ≥ 0 với mọi x ≥ 1.

Sử dụng tính chất đặc trưng của hàm bậc nhất vào chứng minh bất đẳng thức
LATEX:Nguyễn Thành Nhân

BÀI 12. Chứng minh rằng


2(a + b + c) − (ab + bc + ca) ≤ 4, ∀a, b, c ∈ [0; 2] (1)
BÀI 13 (Tiệp Khắc). Cho a, b c ≥ 0 và a + b + c = 1. Chứng minh rằng
7
ab + bc + ca − 2abc ≤ (2)
27

BÀI 14. Chứng minh rằng với a, b, c, d ∈ [0; 1] ta luôn có

(1 − a)(1 − b)(1 − c)(1 − d ) + a + b + c + d ≥ 1 (3)

BÀI 15. Cho các số không âm a, b, c và có tổng bằng 1. Chứng minh rằng
1
a3 + b3 + c3 + 6abc ≥ (4)
4

BÀI 16 (Phan Văn Anh). Cho x, y, z ∈ [0; 1]. Chứng minh rằng

x + y + z − ( x y + yz + zx) ≤ 1 (5)

BÀI 17. Cho ba số không âm x, y, z có tổng bằng 1. Chứng minh rằng

4( x y + yz + zx) ≤ 9 x yz + 1 (6)

BÀI 18. Cho ba số không âm x, y, z có tổng bằng 1. Tìm giá trị lớn nhất của biểu thức sau tùy theo k

A = x y + yz + zx + kx yz (7)

BÀI 19. Cho ba số không âm x, y, z thỏa mãn x + y + z = 1. Chứng minh rằng


9 1
x2 + y2 + z2 + x yz ≥ (8)
2 2

BÀI 3. HÀM SỐ BẬC HAI


Nguyễn Thành Nhân- Chuyên Hùng Vương- Bình Dương

A. CƠ SỞ LÝ THUYẾT
1. Định nghĩa
Định nghĩa 1. Hàm số bậc hai được cho bởi công thức y = ax2 + bx + c (a ̸= 0).
Tập xác định của hàm số này là D = R.

ÓNguyễn Thành Nhân 38 Chuyên Hùng Vương-BD


Các Bài Giảng 2023-2024 Đại Số 10 Chuyên

2. Chiều biến thiên của hàm số bậc hai


Ta xét sự biến thiên trong hai trường hợp a > 0 và a < 0 như sau

+ Với a > 0

b
x −∞ − +∞
2a
+∞ +∞

y
−∆
4a

+ Với a < 0

b
x −∞ − +∞
2a
−∆
y 4a

−∞ −∞

Từ đó ta có định lí sau

TOÁN CHUYÊN
b
µ ¶
Định lí 1. Nếu a > 0 thì hàm số y = ax2 + bx + x nghịch biến trên khoảng −∞; − , đồng biến trên khoảng
2a
b
µ ¶
− ; +∞ .
2a
b b
µ ¶ µ ¶
Nếu a < 0 thì hàm số y = ax2 + bx + x đồng biến trên khoảng −∞; − , nghịch biến trên khoảng − ; +∞ .
2a 2a

3. Đồ thị của hàm số bậc hai


b −∆
µ ¶
Định nghĩa 2. Đồ thị của hàm số y = ax2 + bx + c (a ̸= 0) là một đường parabol có đỉnh là điểm I − ; , có
2a 4a
b
trục đối xứng là đường thẳng x = − . Parabol này quay bề lõm lên trên nếu a > 0, xuống dưới nếu a < 0.
2a
* Cách vẽ đồ thị hàm số bậc hai.
b −∆
µ ¶
1 Xác định tọa độ của đỉnh I − ; .
2a 4a
b
2 Vẽ trục đối xứng x = − .
2a

3 Lập bảng giá trị

b
x x1 x2 − x3 x4
2a
−∆
y y ( x1 ) y ( x2 ) y ( x3 ) y ( x4 )
4a

Chú ý. Đồ thị của hàm số y = ax2 + bx + c (a ̸= 0) cắt trục tung tại điểm (0; c).
Đồ thị của hàm số y = ax2 + bx + c (a ̸= 0) cắt trục hoành (nếu có) tại điểm có tọa độ ( x0 ; 0) với x0 là nghiệm
của phương trình ax2 + bx + c = 0.

4 Vẽ Parabol

Chú ý. Khi vẽ cần chú ý đến dấu của hệ số a (a > 0 bề lõm quay lên trên, a < 0 bề lõm quay xuống dưới).

ÓNguyễn Thành Nhân 39 Chuyên Hùng Vương-BD


Các Bài Giảng 2023-2024 Đại Số 10 Chuyên

VÍ DỤ 1. Lập bảng biến thiên và vẽ đồ thị của các hàm số sau:

1 y = − x2 + 2 x + 3. 2 y = − x 2 + 2 x − 2. 3 y = 3 x2 − 2 x + 1. 4 y = −3 x2 + 2 x − 1.

VÍ DỤ 2. Cho hàm số y = − x2 + 2 x + 3.
1 Lập bảng biến thiên và vẽ đồ thị (P ) của hàm số.

2 Tìm các giá trị của x để y > 0 và y < 0.

VÍ DỤ 3. Cho hàm số y = x2 − 4 x + 3.
1 Lập bảng biến thiên và vẽ đồ thị (P ) của hàm số.

2 Tìm các giá trị của x để y > 0 và y < 0.

4. Một vài lưu ý


Chú ý 6.
LATEX:Nguyễn Thành Nhân

• Ta có biến đổi
b 2 b2 − 4ac b 2 ∆
µ ¶ µ ¶
y = ax2 + bx + c = a x + − = a x+ − .
2a 4a 2a 4a
¯ ¯
¯ −b ¯
Do đó đồ thị này được suy ra từ đồ thị y = ax2 bằng cách tịnh tiến theo sang trái (hoặc phải) ¯¯ ¯ đơn
2a ¯
¯ −∆ ¯
¯ ¯
vị và tịnh tiến lên trên (hoặc xuống dưới) ¯¯ ¯ đơn vị.
4a ¯
b
• Với a > 0 thì hàm số f ( x) = ax2 + bx + c đạt giá trị nhỏ nhất tại x = − .
2a
b
• Với a < 0 thì hàm số f ( x) = ax2 + bx + c đạt giá trị lớn nhất tại x = − .
2a

Chú ý. Đồ thị của hàm số y = ax2 + bx + c (a ̸= 0) cắt trục tung tại điểm (0; c) (lấy x = 0 thế vào hàm số).
Chú ý. Đồ thị của hàm số y = ax2 + bx + c (a ̸= 0) cắt trục hoành (nếu có) tại điểm có tọa độ ( x0 ; 0) với x0 là
nghiệm của phương trình: ax2 + bx + c = 0 (1). Số nghiệm của phương trình (1) là số giao điểm của đồ thị với
trục hoành.

5. Một tính chất đặc trưng của hàm số bậc hai


Ở buổi học trước, khi học về hàm số bậc nhất chúng ta đã biết tính chất đặc trưng của hàm số bậc nhất và
được khai thác sâu sắc vào giải bất phương trình và đặc biệt là chứng minh bất đẳng thức. Sau đây ta sẽ xét
đến tính chất đặc trưng của hàm số bậc hai.

1 Nếu f ( x) = ax2 + bx + c với a > 0 và x ∈ [ m; n] ta có f ( x) ≤ max{ f ( m); f ( n)}.

2 Nếu f ( x) = ax2 + bx + c với a < 0 và x ∈ [ m; n] ta có f ( x) ≥ min{ f ( m); f ( n)}.

Hai tính chất nêu trên sẽ được khai thác vào chứng minh một số bài toán bất đẳng thức rất hiệu quả.

B. BÀI TẬP

VÍ DỤ 4. Cho parabol (P ): y = x2 − 4 x + 3. Dùng (P ) tìm tập hợp các giá trị của x để y ≤ 0.

VÍ DỤ 5. Cho parabol (P ): y = x2 − x − 2. Dùng đồ thị (P ), biện luận theo m số nghiệm của phương trình:
x2 − x − ( m − 2) = 0.

ÓNguyễn Thành Nhân 40 Chuyên Hùng Vương-BD


Các Bài Giảng 2023-2024 Đại Số 10 Chuyên

VÍ DỤ 6. Cho parabol (P ): y = x(2 − x) + 3 và đường thẳng d : y = − x + m. Định m để:

1 d cắt (P ) tại hai điểm phân 2 d và (P ) tiếp xúc. 3 d và (P ) không có điểm


biệt. chung.

VÍ DỤ 7. Cho hàm số: y = x2 − 2 x − 3 có đồ thị là parabol (P ) và đường thẳng d : y = 4 x + m. Biện luận


theo m số giao điểm của d và (P ).

1
VÍ DỤ 8. Cho parabol y = − x2 và đường thẳng y = x + m. Với giá trị nào của m thì parabol và đường
4
thẳng cắt nhau tại hai điểm phân biệt?

1
VÍ DỤ 9. Cho parabol y = x2 . Tìm giá trị của m và n để đường thẳng y = mx + n đi qua điểm (0; −1) và
2
tiếp xúc với parabol.

x2
VÍ DỤ 10. Cho hai parabol y = − x2 + 2 x + 3 và y = − 4 x + 3. Tìm m để đường thẳng y = m cắt cả hai
2
parabol.

VÍ DỤ 11. Xác định parabol y = ax2 + bx + 3, biết rằng parabol đi qua hai điểm A (1; 2) và B(−2; 11).

TOÁN CHUYÊN
VÍ DỤ 12. Cho parabol (P ) : y = − x2 + bx + c. Xác định b, c biết (P ) đi qua điểm M (−2; 4) và có trục đối
xứng x = −2.

VÍ DỤ 13. Cho parabol (P ) : y = ax2 − 2 x + c. Xác định parabol (P ) biết (P ) có đỉnh I (1; −3).

VÍ DỤ 14. Cho parabol (P ) : y = ax2 + bx + c. Xác định a, b, c biết (P ) có giá trị nhỏ nhất bằng −5 và đi
qua hai điểm M (1; −1), N (0; 4).

VÍ DỤ 15. Cho hàm số y = x2 − mx + m + 1 với m ∈ R. Xác định m để đồ thị hàm số là parabol có đỉnh
nằm trên đường thẳng y = x sao cho hoành độ đỉnh không âm.

VÍ DỤ 16. Cho parabol (P ) : y = ax2 + bx + 4. Xác định a, b biết:


1 (P ) đi qua hai điểm M (−1; 7) và N (−4; 4).

2 (P ) có trục đối xứng x = 3 và đi qua điểm A (2; −4).


9
3 (P ) có tung độ đỉnh bằng và cắt trục hoành tại điểm có hoành độ bằng 4.
2

VÍ DỤ 17. Cho parabol (P ) : y = − x2 + bx + c. Xác định b, c biết:


1 (P ) có đỉnh I (1; 4).

2 (P ) đối xứng qua trục tung và có giá trị lớn nhất bằng 3.

3 (P ) có hoành độ đỉnh bằng tung độ đỉnh và đi qua gốc tọa độ O .

VÍ DỤ 18. Cho parabol (P ) : y = ax2 + 3 x + c. Xác định a, c biết:


1 (P ) có hoành độ đỉnh bằng −1 và cắt trục tung tại điểm có tung độ bằng 4.

ÓNguyễn Thành Nhân 41 Chuyên Hùng Vương-BD


Các Bài Giảng 2023-2024 Đại Số 10 Chuyên

2 (P ) có tung độ đỉnh gấp 2 lần hoành độ đỉnh và đi qua điểm A (1; 4).
9
3 (P ) đạt giá trị nhỏ nhất bằng − và đi qua gốc tọa độ O .
2

VÍ DỤ 19. Xác định parabol (P ) biết:

1 (P ) đi qua ba điểm A (1; 7), B(0; 5) và C (4; 1). (− x2 + 3 x + 5)

2 (P ) có trục đối xứng x = 3 và đi qua hai điểm M (−1; 0), N (0; 7).

3 (P ) có đỉnh nằm trên trục hoành và đi qua hai điểm H (−2; 1), K (0; 9).

VÍ DỤ 20. Cho parabol (P ) : y = ax2 − 2ax + 2a với (a ̸= 0). Xác định a để (P ) có đỉnh nằm trên đường
thẳng 2 x − y = 0.

VÍ DỤ 21. Xác định parabol (P ) biết (P ) có đỉnh I cách đều hai trục tọa độ, đi qua gốc tọa độ O (0; 0) và
1
nhận x = làm trục đối xứng.
2
LATEX:Nguyễn Thành Nhân

VÍ DỤ 22. Xác định parabol (P ) biết (P ) đi qua điểm A (0; 1) và có đỉnh I thuộc đường thẳng x + y − 3 = 0
sao cho độ dài đoạn I M ngắn nhất, biết M (−1; 3).

VÍ DỤ 23. Biện luận theo tham số m số nghiệm của phương trình | x2 − 4 x + 3| = m.

VÍ DỤ 24. a) Vẽ đồ thị hàm số y = x2 − 4 x + 3.


b) Tìm m để phương trình x2 − 4| x| + 3 = m có 4 nghiệm phân biệt.

VÍ DỤ 25. Xét tính đơn điệu của hàm số y = x2 + 2| x|.

VÍ DỤ 26. Tìm tât cả các giá trị của tham số m để hàm số y = x2 + (m − 3) x + m đồng biến trên khoảng
(1; +∞).

VÍ DỤ 27. Cho hàm số y = (m − 1) x2 + 4 x − 5 với m ̸= 1. Tìm m sao cho hàm số đồng biến trên (1; 7).

BÀI TẬP TỰ LUYỆN

BÀI 1. Biện luận theo m số nghiệm của phương trình | x2 + 3 x| = m.


BÀI 2. Tìm m để phương trình 2 x2 − | x| = m có đúng 3 nghiệm.
BÀI 3. Hỏi phương trình |2 x2 − | x|| = m có tối đa bao nhiêu nghiệm?
BÀI 4. Xác định các khoảng đồng biến và nghịch biến của hàm số y = x| x − 2|.
BÀI 5. Tìm tất cả các giá trị của tham số m để hàm số y = − x2 + (2m − 3) x + 1 đồng biến trên khoảng (−∞; −5).
BÀI 6. Cho hàm số y = (m − 1) x2 + 4 x − 5 với m ̸= 1. Tìm m sao cho hàm số nghịch biến trên (−5; 2).
x2 − 4 x + 3
(
khi x ≥ 0
BÀI 7. Cho hàm số y = f ( x) = . Biện luận theo m số nghiệm của phương trình f ( x) = m.
x−1 khi x < 0

BÀI 8. Cho hàm số y = x2 − 2mx + m2 − 1 có đồ thị là (C m ). Chứng minh rằng (C m ) luôn cắt xx′ tại 2 điểm phân
biệt A , B. Tìm m để A , B đối xứng nhau qua gốc tọa độ O .
BÀI 9. Cho hàm số y = x2 − 4mx + 4m + 3 (P m ) với m ∈ R. Viết phương trình đường thẳng d vuông góc với đường
thẳng ∆ : y = x − 1 và đi qua điểm cố định của họ parabol (P m ).

ÓNguyễn Thành Nhân 42 Chuyên Hùng Vương-BD


Các Bài Giảng 2023-2024 Đại Số 10 Chuyên

BÀI 10. Cho hàm số y = x2 − (m − 1) x + m − 2 (P m ) với m ∈ R. Tìm tất cả các giá trị của m để khoảng cách từ
1
đỉnh của (P m ) đến trục hoành bằng .
2
BÀI 11. a) Vẽ đồ thị hàm số y = x2 + 4 x + 3.
b) Biện luận theo m số nghiệm của phương trình x2 − 4| x| + 3 = m.

BÀI TẬP TỔNG HỢP VÀ CHUYÊN


BÀI 12 (Thi HSG Hải Dương 2017-2018). Cho hàm số y = x2 − 2( m − 1) x − 2m và y = 2 x + 3. Tìm m để đồ thị của
các hàm số đó cắt nhau tại hai điểm phân biệt A , B sao cho O A 2 + OB2 nhỏ nhất, với O là gốc tọa độ. ĐS:
−11
m= .
10
BÀI 13. Cho parabol (P ) : y = x2 + 2mx + 3. Có bao nhiêu giá trị của tham số m để đồ thị (P ) cắt trục hoành
tại hai điểm phân biệt A , B sao cho tam giác I AB là tam giác đều với I là đỉnh của (P ). ĐS: 2.
BÀI 14. Cho hàm số f ( x) = ax2 + bx + c có đồ thị là parabol (P ) đỉnh I (1; 2). Biết rằng đường thẳng (d ) : y = 4
7
cắt (P ) tại hai điểm phân biệt A , B và tam giác I AB đều. Tính f (2). ĐS: f (2) = .
2
1
BÀI 15. Cho hai hàm số (P ) : y = f ( x) = x2 − x và (P ) : y = g( x) = ax2 − 4ax + b (a > 0) có các đỉnh lần lượt là I 1 và
4
I 2 . Gọi A , B lần lượt là giao điểm của đồ thị (P ) và trục hoành, Biết rằng 4 đỉnh A , B, I 1 , I 2 tạo thành tứ giác
lồi có diện tích bằng 10. Tính diện tích S của tam giác I AB với I là đỉnh của parabol (P ) : y = h( x) = f ( x) + g( x).
ĐS: S = 6.
BÀI 16 (Võ Quốc Bá Cẩn). Cho a, b, c ∈ [1; 2]. Chứng minh rằng
1 1 1
µ ¶

TOÁN CHUYÊN
(a + b + c) + + ≤ 10 (1)
a b c

BÀI 17. Cho x1 , x2 , . . . , xn ∈ [0; 1]. Chứng minh rằng

(1 + x1 + x3 + · · · + xn )2 ≥ 4 x12 + x22 + · · · + x2n


¡ ¢
(2)

BÀI 18 (VM0 2012). Cho các cấp số cộng (a n ), (b n ) và số nguyên dương m > 2. Xét m tam thức bậc hai P1 ( x),
P2 ( x), . . . , P m ( x) với P k ( x) = x2 + a k x + b k với k = 1, 2, . . . , m. Chứng minh rằng nếu hai tam thức P1 ( x), P m ( x) đều
không có nghiệm thì tất cả các tam thức còn lại cũng vô nghiệm.
BÀI 19 (VM0 1996). Cho a, b, c là các số không âm thỏa mãn ab + bc + ca + abc = 4. Chứng minh rằng ta có bất
đẳng thức
a + b + c ≥ ab + bc + ca (3)

BÀI 4. HÀM SỐ BẬC HAI-BÀI TẬP


Nguyễn Thành Nhân- Chuyên Hùng Vương- Bình Dương

A. CƠ SỞ LÝ THUYẾT
B. BÀI TẬP
BÀI 1. Cho parabol (P ) : y = ax2 + bx + c.
1 Tìm hàm số biết đỉnh I (2; −1) và đồ thị đi qua điểm M (4; 3).

2 Khảo sát, vẽ (P ).

3 Từ đồ thị (P ), suy ra đồ thị của các hàm số y = | x2 − 4 x + 3| (P1 ), y = x2 − 4| x|+ 3 (P2 ), y = ( x − 1)| x − 3| (P2 ).

4 Dùng đồ thị, biện luận theo m số nghiệm của các phương trình | x2 − 4 x + 3|+ m + 1 = 0; x2 − 4| x|− m + 4 = 0.

5 Cho đường thẳng ( d ) : y = mx + 3. Tìm m để d cắt (P ) tại hai điểm phân biệt A , B mà
9
+ SO AB = . ĐS: m = −1 hoặc m = −7.
1
3 3
+ x A + x B = 8. ĐS: m = −2.

6 Tìm m để đường thẳng d là tiếp tuyến của parabol (P ).

ÓNguyễn Thành Nhân 43 Chuyên Hùng Vương-BD


Các Bài Giảng 2023-2024 Đại Số 10 Chuyên

BÀI 2. Chứng minh rằng đường thẳng sau đây luôn tiếp xúc với một Parabol cố định
1
1 y = 2 mx − m2 + 4 m + 2 ( m ̸= 0). ĐS: y = x2 + 4 x + 6. 2 y = (4 m − 2) x − 4 m2 − 2 ( m ̸= ). ĐS: y = x2 − 2 x − 2.
2

BÀI 3. Chứng minh rằng các parabol sau đây luôn tiếp xúc với một đường thẳng cố định.

1 y = 2 x2 − 4(2 m − 1) x + 8 m2 − 3. ĐS: y = 4 x − 3. 2 y = mx2 − (4 m − 1) x + 4 m + 1 ( m ̸= 0). ĐS: y = x − 1.

BÀI 4. Tìm giá trị nhỏ nhất và giá trị lớn nhất nếu có của các biểu thức

1 f ( x) = ( x − 1)( x − 2)( x − 3)( x − 4).


p p p
2 f ( x) = x + 2 + 6 − x + 4 − x2 + 4 x + 12 + 2019. p
p p
Từ đó hãy tìm tất cả các giá trị của tham số m để phương trình x + 2 + 6 − x + 4 − x2 + 4 x + 12 = m có
nghiệm.

BÀI 5. Cho hàm số y = x2 + (2m + 1) x + m2 − 1 (Pm ).

1 Chứng minh rằng với mọi m thì (Pm ) luôn cắt đường thẳng y = x tại hai điểm phân biệt A , Bpvà
AB = const. ĐS: AB = 2 2.

2 Chứng minh rằng với mọi m thì (Pm ) luôn tiếp xúc với một đường thẳng cố định. Tìm đường thẳng đó.
LATEX:Nguyễn Thành Nhân

ĐS: y = x − 1.

3 Tìm những điểm trong mặt phẳng tọa độ mà đồ thị của hàm số (Pm ) không thể đi qua với mọi m. ĐS:
Nửa mặt phẳng − x + y + 1 < 0.

Vận dụng tính chất của hàm bậc hai để chứng minh các bất đẳng thức sau đây
BÀI 6 (Võ Quốc Bá Cẩn). Cho a, b, c ∈ [1; 2]. Chứng minh rằng

1 1 1
µ ¶
(a + b + c) + + ≤ 10 (1)
a b c

BÀI 7. Cho x1 , x2 , . . . , xn ∈ [0; 1]. Chứng minh rằng

(1 + x1 + x3 + · · · + xn )2 ≥ 4 x12 + x22 + · · · + x2n


¡ ¢
(2)

BÀI 8 (VM0 2012). Cho các cấp số cộng (a n ), ( b n ) và số nguyên dương m > 2. Xét m tam thức bậc hai P1 ( x),
P2 ( x), . . . , P m ( x) với P k ( x) = x2 + a k x + b k với k = 1, 2, . . . , m. Chứng minh rằng nếu hai tam thức P1 ( x), P m ( x) đều
không có nghiệm thì tất cả các tam thức còn lại cũng vô nghiệm.
BÀI 9 (VM0 1996). Cho a, b, c là các số không âm thỏa mãn ab + bc + ca + abc = 4. Chứng minh rằng ta có bất
đẳng thức
a + b + c ≥ ab + bc + ca (3)

BÀI TẬP HÀM SỐ BẬC HAI-TOÁN THỰC TẾ (TOÁN


THƯỜNG)
Nguyễn Thành Nhân- Chuyên Hùng Vương- Bình Dương

CÁC BÀI TOÁN


BÀI 1. Biện luận theo m số nghiệm phương trình | x2 − 2| x| − 4| = m.
BÀI 2. Khi một quả bóng được đá lên, nó sẽ đạt đến độ cao nào đó rồi rơi xuống. Biết rằng quỹ đạo của quả
bóng là một Parabol trong mặt phẳng với hệ tọa độ Oth, trong đó t là thời gian (tính bằng giây) kể từ khi
quả bóng được đá lên, h là độ cao (tính bằng mét) của quả bóng. Giả thiết rằng quả bóng được đá lên từ độ
cao 1,2 m. Sau đó 1 giây, nó đạt độ cao 8,5 m và 2 giây sau khi đá lên, nó đạt độ cao 6 m. Hỏi sau bao lâu thì
quả bóng sẽ chạm đất kể từ khi được đá lên (tính chính xác đến hàng phần trăm)?
BÀI 3.

ÓNguyễn Thành Nhân 44 Chuyên Hùng Vương-BD


Các Bài Giảng 2023-2024 Đại Số 10 Chuyên

Cho hàm số y = f ( x) = ax2 + bx + c có đồ thị như hình vẽ y


1 Tìm a, b, c.

2 Tìm giá trị lớn nhất và giá trị nhỏ nhất của hàm số đã
3
cho trên đoạn [0, 3].

2
O x
−1

BÀI 4. Tháp Bánh Ít là một trong số ít những quần thể kiến trúc, văn hóa Chăm còn sót lại ở Việt Nam,
được xây dựng vào khoảng cuối thế kỉ XI đến đầu thế kỉ XII nằm trên ngọn đồi tại thôn Đại Lộc, xã Phước
Hiệp, huyện Tuy Phước, tỉnh Bình Định. Theo dòng thời gian, tháp Bánh Ít đã mang trong mình những dấu
ấn lịch sử của Vương quốc Chăm Pa cổ đại.
Trong một lần đi dã ngoại các bạn học sinh trường Phổ thông Năng Khiếu đã thực hiện phép đo ngọn tháp
bằng cách đặt hai giác kế (công cụ dùng để đo góc) tại hai điểm A , B trên mặt đất cách nhau 12 m cùng
thẳng hàng với chân C của tháp. Chân của giác kế có chiều cao h = 1,3 m. Gọi D là đỉnh tháp, các điểm C1 ,
A 1 , B1 thẳng hàng (như hình vẽ). Các bạn nhận thấy D
á A 1 C 1 = 49◦ và DB
á ◦
1 C 1 = 35 . Hãy tính chiều cao CD
của ngọn tháp.

TOÁN CHUYÊN
D

49◦ A1
35◦
B1
C1

C A 12 m B

BÀI 5. Trong trận chung kết WC2022, L. Messi đã có cơ hội thực hiện cú sút phạt trực tiếp trước khung
thành đội Pháp. Các cầu thủ Pháp lập thành hàng rào chắn cách điểm đá phạt 9 m và cầu thủ cao nhất
trong hàng rào là 2 m. Giả định rằng quĩ đạo quả bóng sau khi Messi thực hiện cú sút là một Parabol (như
hình vẽ) và nó đạt được chiều cao cực đại là 3 m sau khi rời chân Messi 14 m. Hỏi cú đá phạt này của Messi
có đưa bóng đi qua điểm cao nhất của hàng rào hay không? Tại sao?

3 B

O 9 14 20

ÓNguyễn Thành Nhân 45 Chuyên Hùng Vương-BD


Các Bài Giảng 2023-2024 Đại Số 10 Chuyên

BÀI 6.
Cầu Ponte Luis ở Porto (Bồ Đào Nha) có hình dạng y
5
đường cong parabol có bề lõm quay xuống dưới để
giúp giảm lực mà cây cầu phải gánh. Giả sử ta lập
A
một hệ trục tọa độ Ox y sao cho một chân cầu đi qua 4
gốc tọa độ O như hình vẽ ( x, y tính bằng mét), chân
kia của cầu ở vị trí (6; 0). Biết một điềm A trên cầu 3
có tọa độ là (2; 4). Tìm hàm số bậc hai có đồ thị chứa
cung parabol nói trên.
2

O x
−1 1 2 3 4 5 6

−1

BÀI 7. Cho parabol (P ) : y = f ( x) = x2 − 2 x + m.

1 Chỉ ra đỉnh và các khoảng đồng biến, nghịch biến của parabol (P ).
LATEX:Nguyễn Thành Nhân

2 Tìm m để parabol (P ) cắt trục hoành tại hai điểm phân biệt có hoành độ x1 , x2 thỏa x12 + x22 = 10.

BÀI 8.
Khi một quả bóng được đá lên, nó sẽ bay theo quỹ đạo của một cung parabol h
trong mặt phẳng tọa độ Oth như hình bên, trong đó t là thời gian kể từ khi
quả bóng được đá lên (tính bằng giây), h là độ cao (tính bằng mét) của quả 8.6
bóng. Giả sử quả bóng được đá lên từ độ cao 1,1 m. Sau 1 giây nó đạt độ cao
8,6 m. Sau 2 giây, nó đạt độ cao 6 m. Hỏi độ cao lớn nhất mà quả bóng đạt
được là bao nhiêu? (làm tròn đến hàng phần chục) 6

2
1.1

O 1 2 3 t

BÀI 9. Cho parabol (P ) : y = x2 + ax + b. Tìm a, b biết (P ) có trục đối xứng là d : x = 2 và qua điểm A (0; 3).
(
x2 − 4 x + 3 khi x ≤ 3
BÀI 10. Cho hàm số f ( x) =
− x2 + 10 x − 21 khi x > 3.

1 Vẽ đồ thị hàm số f ( x).

2 Tìm tất cả các giá trị của tham số m sao cho phương trình f ( x) = m có 2 nghiệm phân biệt.

3
BÀI 11. Xác định parabol (P ) : y = ax2 + bx + 2 (a ̸= 0) biết (P ) có trục đối xứng x = và cắt trục hoành tại điểm
2
M có hoành độ bằng 1.

BÀI 12. Một gia đình sản xuất gạo, do điều kiện nhà xưởng nên mỗi đợt gia đình đó sản xuất được x kg gạo.
Biết rằng mỗi kí bán được với giá 350 − 5 x (nghìn đồng) và chi phí sản xuất x kg gạo là x2 + 5 x + 1000 (nghìn
đồng). Hỏi gia đình đó phải sản xuất được bao nhiêu kg gạo để đạt được lợi nhuận tối đa?

BÀI 13. Xét sự biến thiên của hàm số y = x2 − 4 x + 5 trên (−∞; 2).

BÀI 14. Cho hàm số y = ax2 + bx + 3 có đồ thị (P ). Tìm a, b biết (P ) có đỉnh I (2; −1).

BÀI 15.

ÓNguyễn Thành Nhân 46 Chuyên Hùng Vương-BD


Các Bài Giảng 2023-2024 Đại Số 10 Chuyên

Tại một khu hội chợ, người ta thiết kế cổng chào có hình parabol hướng bề y
lõm xuống dưới. giả sử lập một hệ trục toạ độ Ox y sao cho một chân cổng đi
qua gốc toạ độ O như hình vẽ ( x và y tính bằng mét). Chân kia của cổng ở vị
trí (4; 0). Biết một điểm M trên cổng có toạ độ (1; 3). Hỏi chiều cao của cổng (vị
3 M
trí cao nhất của cổng tới mặt đất) là bao nhiêu mét?

O 1 4 x

BÀI 16.
Cho hàm số f ( x) = ax2 + 2 x + c (a ̸= 0) có đồ thị (P ) như hình vẽ bên. y

1 Xác định dấu của a.

2 Xác định trục đối xứng của đồ thị (P ).


−3 −1 O 1 x
3 Tìm giao điểm của (P ) với hai trục tọa độ.

4 Lập bảng biến thiên của hàm số.

5 Tính giá trị T = f (2) − f (−4).


−3

−4

TOÁN CHUYÊN
BÀI 17. Tìm parabol (P ) : y = ax2 + bx + c, biết rằng (P ) đi qua ba điểm A (1; −1), B(2; 3), C (−1; −3).
BÀI 18. Cho parabol (P ) : y = x2 + 6 x + 8.

1 Xác định tọa độ đỉnh, trục đối xứng của parabol (P ), tọa độ giao điểm của parabol (P ) với trục hoành.

2 Lập bảng biến thiên, vẽ đồ thị hàm số y = x2 + 6 x + 8.

3 Dựa vào đồ thị trên tìm giá trị nhỏ nhất và giá trị lớn nhất của hàm số f ( x) = x2 + 6 x + 8 trên đoạn [−4; 0].

BÀI 19. Khi một quả bóng được đá lên, nó sẽ đạt đến độ cao nào đó rồi rơi xuống. Biết rằng quỹ đạo của quả
bóng là một Parabol trong mặt phẳng với hệ tọa độ Oth, trong đó t là thời gian (tính bằng giây) kể từ khi
quả bóng được đá lên, h là độ cao (tính bằng mét) của quả bóng. Giả thiết rằng quả bóng được đá lên từ độ
cao 1,2 m. Sau đó 1 giây, nó đạt độ cao 8,5 m và 2 giây sau khi đá lên, nó đạt độ cao 6 m. Hỏi sau bao lâu thì
quả bóng sẽ chạm đất kể từ khi được đá lên (tính chính xác đến hàng phần trăm)?
BÀI 20. Lập bảng biến thiên và vẽ đồ thị hàm số y = x2 − 4 x + 3. Tìm giá trị nhỏ nhất của hàm số và tập giá
trị của hàm số đã cho.
BÀI 21. Cho parabol (P ) : y = − x2 + 2 bx + c có điểm M (2; 10) là điểm có tung độ lớn nhất. Tìm phương trình
parabol.
BÀI 22.
Một chiếc cổng hình parabol bao gồm một cửa chính hình chữ nhật ở giữa G
và hai cánh cửa phụ hai bên như hình vẽ. Biết chiều cao cổng parabol là 4
F E
m còn kích thước cửa ở giữa là 3 × 4. Hãy tính khoảng cách giữa hai điểm
A và B.

A C D B

BÀI 23.
Thành phố X dự định xây dựng một trạm nước sạch để cung cấp cho haipkhu B
dân cư A và B. Trạm nước sạch đặt tại vị trí C trên bờ sông. Biết AB = 229
km, khoảng cách từ A và B đến bờ sông lần lượt là AM = 2 km, BN = 4 km
A
(hình vẽ). Gọi T là tổng độ dài đường ống từ trạm nước đến A và B. Tìm giá
trị nhỏ nhất của T .
C
M N

ÓNguyễn Thành Nhân 47 Chuyên Hùng Vương-BD


Các Bài Giảng 2023-2024 Đại Số 10 Chuyên

BÀI 24.
Cho hàm số y = ax2 + b| x| + c với a ̸= 0 có đồ thị như hình vẽ bên. y
Chỉ ra các khoảng đồng biến và nghịch biến của đồ thị trên các
khoảng sau: (−∞; −1), (−1; 0), (0; 1), (1; +∞).

O
1 x
−1

BÀI 25. Khảo sát sự biến thiên và vẽ đồ thị hàm số y = x2 − 3 x + 2.


Từ đó suy ra đồ thị y = x2 − 3| x| + 2.
BÀI 26.
Xác định parabol (P), có đồ thị như hình bên. y

1
LATEX:Nguyễn Thành Nhân

O x

−1

−3

BÀI 27. Khi du lịch đến thành phố Lui (Mĩ) ta sẽ thấy một cái cổng lớn dạng parabol bề lõm quay xuống
dưới. Đó là cổng Acxơ. Biết cổng dài 162 m, và đi từ một đầu bên đây của cổng 10 m thì sẽ thấy một cây đèn
cao 43 m gắn với cổng. Tính chiều cao của cổng? (khoảng cách từ điểm cao nhất của cổng đến mặt đất).
BÀI 28. Cho hàm số y = x2 − 4 x + 3, có đồ thị là (P ).
1 Lập bảng biến thiên và vẽ đồ thị (P ).

2 Nhận xét về sự biến thiên của hàm số trong khoảng (0; 3).

3 Tìm tập hợp giá trị x sao cho y ≤ 0.

4 Tìm các khoảng của tập xác định để đồ thị (P ) nằm hoàn toàn phía trên đường thẳng y = 8.

5 Tìm giá trị lớn nhất, giá trị nhỏ nhất của hàm số trên đoạn [−2; 1].
BÀI 29.
Cho đồ thị hàm số y = f ( x) như hình bên. Tìm tập xác định, tập giá trị, y
khoảng đồng biến, khoảng nghịch biến của hàm số có đồ thị trên. 1

−3 −2 −1 O 1 2 3 x

−1

−2

−3

−4

−5

BÀI 30. Tìm hàm số bậc hai biết đồ thị qua điểm M (−2; 5) và hàm số đạt giá trị nhỏ nhất bằng −3 tại x = 1.
BÀI 31. Khi nuôi cá thí nghiệm trong hồ, một nhà sinh học thấy rằng: Nếu trên mỗi đơn vị diện tích của mặt
hồ có n con cá thì trung bình mỗi con cá sau một vụ cân nặng P (n) = 360 − 10 n. Hỏi phải thả bao nhiêu con
cá trên một đơn vị diện tích để trọng lương cá sau một vụ thu được nhiều nhất?
BÀI 32. Xác định parabol y = ax2 + bx + c, biết rằng parabol có đỉnh S (2; −1) và cắt trục tung tại điểm có tung
độ bằng −3.
BÀI 33. Một quả bóng được ném vào không trung có chiều cao tính từ lúc bắt đầu ném ra được cho bởi công
thức h( t) = − t2 + 4 t + 5 (tính bằng mét), t là thời gian tính bằng giây ( t ≥ 0). Tính chiều cao lớn nhất quả bóng
đạt được và tính xem sau bao lâu quả bóng sẽ rơi xuống mặt đất?

ÓNguyễn Thành Nhân 48 Chuyên Hùng Vương-BD


Các Bài Giảng 2023-2024 Đại Số 10 Chuyên

BÀI 34. Một công ty dịch vụ du lịch đang tổ chức các tua du lịch mùa World Cup từ Việt Nam đi Qatar. Với
giá tua 10 ngày, công ty định giá 300 triệu thì sẽ có 400 người tham gia, tuy nhiên công ty dự tính rằng cứ
giảm giá đi 10 triệu thì sẽ có thêm 5 người tham gia. Hỏi công ty nên bán giá là bao nhiêu thì sẽ có doanh
thu lớn nhất
BÀI 35. Tìm parabol y = ax2 + bx + 3, biết rằng parabol đó

1 đi qua hai điểm A (2; 15) và B(−1; 0);

2 đi qua điểm P (−3; 9) và có trục đối xứng x = −1;

3 có đỉnh I (−2; 19).

BÀI 36. Một cây cầu treo có trọng lượng phân bố đều dọc theo chiều dài của nó. Cây cầu có trụ tháp đôi cao
75 m so với mặt của cây cầu và cách nhau 400 m. Các dây cáp có hình dạng đường parabol và được treo trên
các đỉnh tháp. Các dây cáp chạm mặt cầu ở tâm của cây cầu. Tìm chiều cao của dây cáp tại điểm cách tâm
của cây cầu 100 m (giả sử mặt của cây cầu là bằng phẳng).
BÀI 37. Cho đồ thị hàm số bậc hai như dưới đây
y
y

y = f ( x)

y = g ( x)

1
O 1
x O 1
1

TOÁN CHUYÊN
x

Với mỗi đồ thị, hãy:

1 Tìm tọa độ đỉnh của đồ thị.

2 Tìm khoảng đồng biến và khoảng nghịch biến của hàm số.

3 Tìm giá trị lớn nhất hoặc giá trị nhỏ nhất của hàm số.

4 Tìm tập xác định và tập giá trị của hàm số.

BÀI 38. Với mỗi hàm số bậc hai sau: y = f ( x) = − x2 − x + 1, y = g( x) = x2 − 8 x + 8. Hãy thực hiện các yêu cầu sau

1 Viết lại hàm số bậc hai dưới dạng y = a( x − h)2 + k.

2 Tìm giá trị lớn nhất hoặc nhỏ nhất của hàm số.

3 Vẽ đồ thị hàm số.

BÀI 39. Tìm tập xác định và tập giá trị của các hàm số bậc hai sau

1 f ( x) = − x2 + 4 x − 3. 2 f ( x) = x2 − 7 x + 12.

BÀI 40. Tìm parabol y = ax2 + bx + 2, biết rằng parabol đó

1 đi qua hai điểm M (1; 5), và N (−2; 8).

3
2 đi qua điểm A (3; −4) và có trục đối xứng x = − .
2

3 có đỉnh I (2; −2).

BÀI 41. Tìm phương trình parabol có đỉnh I (−1; 2) và đi qua điểm A (1; 6).

ÓNguyễn Thành Nhân 49 Chuyên Hùng Vương-BD


Các Bài Giảng 2023-2024 Đại Số 10 Chuyên

BÀI 42.
Xác định dấu của các hệ số a, b, c và dấu của biệt thức ∆ = b2 − 4ac y
của hàm số bậc hai y = ax2 + bx + c, biết đồ thị của nó có dạng như
Hình 6.16.

O 1
x
−1

Hình 6.16

BÀI 43. Bác Hùng dùng 200 m hàng rào dây thép gai để rào miếng đất đủ rộng thành một mảnh vườn hình
chữ nhật.
LATEX:Nguyễn Thành Nhân

1 Tìm công thức tính diện tích S ( x) của mảnh vườn hình chữ nhật rào được theo chiều rộng x (m) của
mảnh vườn đó.

2 Tìm kích thước của mảnh vườn hình chữ nhật có diện tích lớn nhất có thể rào được.

BÀI 44. Một quả bóng được ném lên trên theo phương thẳng đứng từ mặt đất với vận tốc ban đầu 14,7 m/s.
Khi bỏ qua sức cản của không khí, độ cao của quả bóng so với mặt đất (tính bằng mét) có thể mô tả bởi
phương trình

h( t) = −4,9 t2 + 14,7 t.

1 Sau khi ném bao nhiêu giây thì quả bóng đạt độ cao lớn nhất?

2 Tìm độ cao lớn nhất của quả bóng.

3 Sau khi ném bao nhiêu giây thì quả bóng rơi chạm đất?

BÀI 45. Một hòn đá được ném lên trên theo phương thẳng đứng. Khi bỏ qua sức cản không khí, chuyển dộng
của hòn đá tuân theo phương trình sau:

y = −4,9 t2 + mt + n,

với m, n là các hằng số. Ở đây t = 0 là thời điểm hòn đá được ném lên, y( t) là độ cao của hòn đá tại thời điểm
t (giây) sau khi ném và y = 0 ứng với với chạm đất.

1 Tìm phương trình chuyển động của hòn đá, biết rằng điểm ném cách mặt đất 1,5 m và thời gian để hòn
đá đạt độ cao lớn nhất là 1,2 giây sau khi ném.

2 Tìm độ cao của hòn đá sau 2 giây kể từ khi ném.

3 Sau bao lâu kể từ khi ném, hòn đá rơi xuống mặt đất (Kết quả làm tròn đến chữ số thập phân thứ hai)?

BÀI 46.
Cho hàm số y = ax2 + bx + c có đồ thị ở Hình 11. Xác định dấu a, b, c. y

O x

Hình 11
BÀI 47.

ÓNguyễn Thành Nhân 50 Chuyên Hùng Vương-BD


Các Bài Giảng 2023-2024 Đại Số 10 Chuyên

Trong một công trình, người ta xây dựng một cổng ra vào hình parabol
(minh hoạ ở Hình 13) sao cho khoảng cách giữa hai chân cổng BC là
9 m. Từ một điểm M trên thân cổng người ta đo được khoảng cách tới
mặt đất là MK = 1,6 m và khoảng cách từ K tới chân cổng gần nhất là
BK = 0,5 m. Tính chiều cao của cổng theo đơn vị mét (làm tròn kết quả
đến hàng phần mười).

Hình 13
BÀI 48. Một rạp chiếu phim có sức chứa 1 000 người. Với giá vé là 40 000 đồng, trung bình sẽ có khoảng 300
người đến rạp xem phim mỗi ngày. Để tăng số lượng vé bán ra, rạp chiếu phim đã khảo sát thị trường và
thấy rằng nếu giá vé cứ giảm 10 000 đồng thì sẽ có thêm 100 người đến rạp mỗi ngày.
1 Tìm công thức của hàm số R ( x) mô tả doanh thu từ tiền bán vé mỗi ngày của rạp chiếu phim khi giá
vé là x nghìn đồng.
2 Tìm mức giá vé để doanh thu từ tiền bán vé mỗi ngày của rạp là lớn nhất.

BÀI 49.
Tại một buổi khai trương, người ta làm một cổng chào có đường
viền trong của mặt cắt là đường parabol. Người ta đo khoảng
N G Q UÝ
cách giữa hai chân cổng là 4,5 m. Từ một điểm trên thân cổng MỪ KH
người ta đo được khoảng cách tới mặt đất (điểm H ) là 1,8 m và O Á
À

CH

CH
khoảng cách từ điểm H tới chân cổng gần nhất là 1 m. Hãy tính

TOÁN CHUYÊN
chiều cao của cổng chào đó (tính theo đường viền trong) theo
đơn vị mét và làm tròn kết quả đển hàng phần mười.

ÓNguyễn Thành Nhân 51 Chuyên Hùng Vương-BD


LATEX:Nguyễn Thành Nhân Các Bài Giảng 2023-2024 Đại Số 10 Chuyên

ÓNguyễn Thành Nhân 52 Chuyên Hùng Vương-BD


CHƯƠNG 4 CÁC CHUYÊN ĐỀ CHUYÊN TOÁN
BÀI 1. ÁNH XẠ
Nguyễn Thành Nhân- Chuyên Hùng Vương- Bình Dương

A. LÝ THUYẾT CẦN NẮM


1. Ánh xạ là gì?
Cùng với khái niệm tập hợp, ánh xạ là một khái niệm quan trọng của toán học. Nó có mặt trong hầu hết các
lĩnh vực toán học khác nhau. Khái niệm ánh xạ là một mở rộng tự nhiên của khái niệm hàm số. Hàm số thì
tập nguồn và tập đích đều là tập con của R, còn ánh xạ thì tập nguồn và tập đích có thể là những tập tùy ý.
Định nghĩa 1. Một ánh xạ f từ tập X đến tập Y là một quy tắc đặt tương ứng mỗi phần tử x ∈ X với một (và
chỉ một) phần tử của Y . Phần tử này được gọi là ảnh của x qua ánh xạ f và được ký hiệu là f ( x).
Tập X được gọi là tập xác định của f , tập Y được gọi là tập giá trị của f .
Ký hiệu:
f : X → Y hay x 7→ f ( x).
Khi X , Y ⊂ R thì ánh xạ f được gọi là hàm số xác định trên X .
• Cho a ∈ X và b ∈ Y . Nếu f (a) = b thì b được gọi là ảnh của a và a được gọi là nghịch ảnh của b.

TOÁN CHUYÊN
• Mỗi phần tử a của X đều có một ảnh duy nhất là f (a). Mỗi phần tử y của Y có thể có một hay nhiều
nghịch ảnh và cũng có thể không có nghịch ảnh nào.
• Tập f ( X ) = { y ∈ Y |∃ x ∈ X , y = f ( x)} gọi là tập ảnh của f . Nói cách khác f ( X ) là tập tất cả các các phần tử
của Y mà có nghịch ảnh.

Một số ví dụ về ánh xạ
• Ánh xạ f từ N đến N đặt tương ứng mỗi số nguyên n với lập phương của nó. Tức là f ( n) = n3 . Tập ảnh
của f chính là tập
{0, 1, 8, 27, . . . , n3 , . . .}.

• Tập X là các lớp học của trường chuyên Hùng Vương, Y là tập hợp các giáo viên của trường và f là quy
tắc đặt tương ứng mỗi lớp học một giáo viên chủ nhiệm của lớp đó. Khi đó ta có ánh xạ

f : X →Y.

• Xét tập M = { m; ; n p} và X là tập tất cả các tập con của M , đó là

X = {∅; { m}; { n}; { p}; { m; n}; { m; p}; { n; p}; { m; n; p}}.

Xét ánh xạ f là đặt tương ứng mỗi tập con của M với số phần tử của tập đó, chẳng hạn f (∅) = 0. Khi đó
ta có ánh xạ
f : X → N.
Tập ảnh là f ( X ) = {0; 1; 2; 3}.
• Cho X là tập tất cả các học sinh của trường chuyên Hùng Vương, Y là tập tất cả các lớp học của trường.
Ánh xạ f là quy tắc đặt tương ứng mỗi học sinh với lớp mà học sinh đó học. Khi đó ta có ánh xạ

f : X →Y.

Tập ảnh của f là tập các lớp học của trường chuyên Hùng Vương.
Nghịch ảnh của mỗi lớp học là mỗi học sinh của lớp đó. Do đó sĩ số lớp học là số nghịch ảnh của lớp học.
• Xét quy tắc sau đây

f :R → R
x 7→ y2 = 4 x.

Quy tắc trên không xác định một ánh xạ bởi vì chẳng hạn với x = 1 nhưng cho tới hai ảnh là y = ±2.

53
Các Bài Giảng 2023-2024 Đại Số 10 Chuyên

2. Đơn ánh, toàn ánh, song ánh.


Định nghĩa 2.

1 Ánh xạ f : X → Y được gọi là đơn ánh nếu với x1 , x2 ∈ X mà x1 ̸= x2 thì f ( x1 ) ̸= f ( x2 ). Tức là hai phần tử
phân biệt sẽ cho hai ảnh phân biệt.
Từ định nghĩa rõ ràng ánh xạ f là đơn ánh khi và chỉ khi với x1 , x2 ∈ X mà f ( x1 ) = f ( x2 ) thì ta phải có
x1 = x2 .

2 Ánh xạ f : X → Y được gọi là toàn ánh nếu với mỗi phần tử ∈ Y đều tồn tại một phần tử x ∈ X sao cho
f ( x) = y.
Như vậy f là toàn ánh nếu và chỉ nếu Y = f ( X ).

3 Ánh xạ f : X → Y được gọi là song ánh giữa X và Y nếu nó vừa là đơn ánh vừa là toàn ánh.
Như vậy f : X → Y là song ánh nếu và chỉ nếu với mỗi phần tử y ∈ Y , tồn tại và duy nhất một phần tử
x ∈ X sao cho f ( x) = y.

Một số ví dụ về đơn ánh, toàn ánh, song ánh


• Hàm số f : R → R được xác định f ( x) = ( x − 2)2 không phải là đơn ánh bởi vì f (0) = f (4) = 4. Ngoài ra f
cũng không phải là toàn ánh vì y = −1 không tồn tại x để f ( x) = −1.

• Hàm số f : R → R được xác định bởi f ( x) = x3 là một đơn ánh vì rõ ràng nếu x13 = x23 thì x1 = x2 . Ngoài ra
LATEX:Nguyễn Thành Nhân

p
f cũng là toàn ánh vì với mỗi y ∈ R và y = x3 thì tồn tại x = 3 y sao cho f ( x) = y. Như vậy f là song ánh.

• Hàm số f : Z → Z sao cho f ( n) = n + 2019 là đơn ánh vì

f ( n 1 ) = f ( n 2 ) ⇒ n 1 + 2019 = n 2 + 2019 ⇒ n 1 = n 2 .

Ngoài ra f cũng là toàn ánh vì với mỗi số b ∈ Z thì tồn tại số a = b − 2019 để f (a) = b. Như vậy f là song
ánh.

3. Ánh xạ ngược của một song ánh


Cho ánh xạ f : X → Y là một song ánh. Khi đó với mỗi y ∈ Y , tồn tại một và duy nhất một phần tử x ∈ X để
f ( x) = y. Phần tử duy nhất x ∈ X này được gọi là ảnh của phần tử y qua ánh xạ ngược của f . Từ đó ta có định
nghĩa
Định nghĩa 3. Ánh xạ ngược của f , ký hiệu là f −1 , là ánh xạ từ Y vào X gán cho mỗi phần tử y ∈ Y phần tử
duy nhất x ∈ X mà f ( x) = y. Như vậy
f −1 ( y) = x ⇔ y = f ( x).
Chú ý 7. Chỉ có song ánh mới có ánh xạ ngược.

p
VÍ DỤ 1. Ánh xạ f : R → R với y = f ( x) = x3 có ánh xạ ngược là f −1 : R → R với x = f −1 ( y) = 3 y.

4. Ánh xạ hợp
Cho g là ánh xạ từ tập A vào tập B và f là ánh xạ từ tập B vào tập C .
Nếu g(a) ∈ B với mỗi a ∈ A (tức là g(a) ⊂ B) thì ta có thể xác định một ánh xạ từ A đến C theo quy tắc sau:
Đặt tương ứng mỗi a ∈ A một phần tử f ( g(a)) ∈ C .
Ánh xạ này được gọi là ánh xạ hợp của ánh xạ f và g, ký hiệu là f ◦ g. Từ đó ta có định nghĩa
Định nghĩa 4. Nếu g : A → B và f : B → C và g( A ) ⊂ B thì ánh xạ hợp f ◦ g : A → C được xác định bởi

( f ◦ g)(a) = f ( g(a)).

Trong trường hợp f , g là các hàm số thì ánh xạ hợp của f và g được gọi là hàm số hợp. Hàm số g được gọi là
hàm số trung gian.
Chú ý 8. Khi điều kiện g( A ) ⊂ B không được thỏa mãn thì ta không xác định đươc hàm hợp f ◦ g từ A vào C .
Tuy nhiên nếu trong A có tập con A ′ mà f ( A ′ ) ⊂ B thì vẫn có thể xác định một ánh xạ hợp

f ◦ g : A′ → C

mà f ◦ g(a) = f ( g(a)).

ÓNguyễn Thành Nhân 54 Chuyên Hùng Vương-BD


Các Bài Giảng 2023-2024 Đại Số 10 Chuyên

VÍ DỤ 2. Cho A = {1; 2; 3}, B = {m; n; p}, C = { x; y; z}. Ta xét các ánh xạ sau đây g : A → B với g(1) = p,
g(2) = n, g(3) = m và ánh xạ f : B → C với f ( m) = y, f ( n) = z, f ( p) = x. Hãy xác định ánh xạ f ◦ g.

VÍ DỤ 3. Cho f : Z → Z và g : Z → Z với f (n) = 2019n + 2020 và g(n) = 2 n − 1. Hãy xác định f ◦ g, g ◦ f , f ◦ f ,


g ◦ g.

B. CÁC DẠNG TOÁN


1. Ứng dụng vào bài toán đếm
Trước khi đi vào ứng dung cụ thể, ta xét tình huống sau đây
Bài toán:
Có một đàn chim bồ câu đi tìm thức ăn và trở về chuồng vào lúc chiều tối. Gọi A là tập hợp các con bồ câu,
B là tập hợp các chuồng bồ câu. Xét ánh xạ f đặt tương ứng mỗi con chim bồ câu và chuồng mà nó bay vào.
Tùy theo f là đơn ánh (không là song ánh), toàn ánh (không là song ánh), song ánh mà ta có sự so sánh về
số chuồng với số chim bồ câu. Cụ thể
• Nếu f là đơn ánh (không là toàn ánh) thì mỗi chuồng chỉ chứa tối đa một con bồ câu và có những
chuồng không có bồ câu. Do đó
| A | < | B |.

• Nếu f là toàn ánh (không phải là đơn ánh) thì dẫn đến chuồng nào cũng có ít nhất một con bồ câu và
có một chuồng nào đó chứa ít nhất hai con bồ câu. Do đó

TOÁN CHUYÊN
| A | > | B |.

• Nếu f là song ánh thì mỗi chuồng có một và chỉ một con chim bồ câu. Do dó

| A | = | B |.

Từ đó ta có định lý
Định lí 1. Cho A và B là hai tập hữu hạn.
• Nếu có một đơn ánh f : A → B thì | A | ≤ |B|.

• Nếu có một toàn ánh f : A → B thì | A | ≥ |B|.

• Nếu có một song ánh f : A → B thì | A | = |B|.

BÀI 1. Có một nhóm người mà trong có cứ mỗi cặp không quen nhau có đúng hai người quen chung, còn mỗi
cặp quen nhau thì không có người quen chung. Chứng minh rằng số người quen của mỗi người là như nhau.

2. Phương trình hàm


BÀI 2 (Slovak 2003-2004). Tìm tất cả các hàm số f : R → R thỏa mãn

f ( x) + x f (1 − x) = x2 + 1, ∀ x ∈ R.

− x3 + 3 x2 − 2 x + 1
ĐS: f ( x) = .
x2 − x + 1
BÀI 3 (Kosovo-211). Tìm tất cả các hàm số f : R → R\{−1, 1} thỏa mãn
x−3 3+ x
µ ¶ µ ¶
f +f = x, ∀ x ̸∈ {−1; 1}.
x+1 1− x

x3 + 7 x
ĐS: f ( x) = .
2 − 2 x2
BÀI 4 (Thái Lan 2006). Tìm tất cả các hàm số f : R → R thỏa mãn điều kiện

f ( x2 + x + 3) + 2 f ( x2 − 3 x + 5) = 6 x2 − 10 x + 17, ∀ x ∈ R.

Hãy tìm f (85). ĐS: f (85) = 167.

ÓNguyễn Thành Nhân 55 Chuyên Hùng Vương-BD


Các Bài Giảng 2023-2024 Đại Số 10 Chuyên

1
µ ¶
BÀI 5. Giả sử f ( x) là một hàm số với mọi giá trị thực, xác định với x ̸= 0, sao cho f ( x) + 2 f = 3 x. Tìm tất cả
x
2 p
các nghiệm của phương trình f ( x) = f (− x). ĐS: f ( x) = − x, x = ± 2.
x
BÀI 6. Tìm tất cả các hàm số f : R → R thỏa mãn
f ( x + y) + f ( x − y) = f ( x) + 6 x y 3 f ( y) + 2019 x2 , ∀ x, y ∈ R.
p

ĐS: f ( x) = 2019 x2 .
BÀI 7. Tìm tất cả các hàm số f : R → R thỏa mãn
f ( x + y) − 2 f ( x − y) + f ( x) − 2 f ( y) = y − 2, ∀ x, y ∈ R.

ĐS: f ( x) = x + 1.
BÀI 8. Tìm tất cả các hàm số f : R → R thỏa mãn f (0) = 1 và
[ f ( x + y)]2019 = [ f ( x)]2019 + y2 + 2 x y, ∀ x, y ∈ R.
p
2019
ĐS: f ( x) = 1 + x2 .
BÀI 9 (Nguyễn Trọng Tuấn). Cho hàm số f : Z → Z thỏa mãn điều kiện f (1) = 0 và
f ( m + n) = f ( m) + f ( n) + 3(4 mn − 1), ∀ m, n ∈ Z.
LATEX:Nguyễn Thành Nhân

Tính f (19). ĐS: f (19) = 1998.


BÀI 10. Cho hàm số f : Z → Z thỏa mãn các điều kiện
1 f (0) ̸= 0 2 f (1) = 3

3 f ( x) f ( y) = f ( x + y) + f ( x − y), ∀ x, y ∈ Z.

Tính F (7). ĐS: f (7) = 843.


BÀI 11. Hàm số f R → R thỏa mãn
f ( x) + f ( y) = f ( x + y) − x y − 1, ∀ x, y ∈ R.

Nếu f (1) = 1. Hãy tìm n nguyên để f ( n) = n. ĐS: n = 1; n = −2.


BÀI 12 (VMO2000). Tìm tất cả các hàm số f : R → R thỏa mãn

BÀI 2. LUYỆN TẬP


Nguyễn Thành Nhân- Chuyên Hùng Vương- Bình Dương

A. CÁC KỸ THUẬT CƠ BẢN GIẢI PHƯƠNG TRÌNH HÀM


Để giải một phương trình hàm, ta thường sử dụng một trong các phương pháp hoặc phối hợp các phương
pháp điển hình sau
1 Phương pháp thế biến.

2 Phương pháp thêm biến.

3 Phương pháp quy nạp.


Phương pháp này thường áp dụng khi giải phương trình hàm trên các tập rời rạc N, N∗ , Z, Q hoặc trên
các tập con của các tập này.
4 Phương pháp sử dụng tính đơn ánh, toàn ánh, song ánh.

5 Sử dụng tính đơn điệu của hàm số.

6 Sử dụng điểm bất động.

7 Sử dụng sai phân trong dãy số.

8 Sử dụng tính liên tục của hàm số.

9 Sử dụng phương trình hàm cơ bản.

ÓNguyễn Thành Nhân 56 Chuyên Hùng Vương-BD


Các Bài Giảng 2023-2024 Đại Số 10 Chuyên

B. BÀI TẬP LUYỆN TẬP


1. Phương pháp thế biến
Định nghĩa 1. Hàm số f ( x) được gọi là cộng tính trên tập xác định trên D nếu với mọi x, y ∈ D thì x + y ∈ D và
f ( x + y) = f ( x) + f ( y).
Hàm số f ( x) được gọi là nhân tính trên tập xác định trên D nếu với mọi x, y ∈ D thì x.y ∈ D và f ( x.y) = f ( x) . f ( y).
BÀI 1 (Trần Minh Hiền). Tìm tất cả các hàm số f : R → R thỏa mãn

f ( f ( x) − y) = f ( x) − f ( y) + f ( x) f ( y) − x y, ∀ x, y ∈ R. (1)

BÀI 2 (Trần Minh Hiền). Tìm tất cả các hàm số f : R → R thỏa mãn

f ( x − y)2 = ( f ( x))2 − 2 x f ( y) + y2 , ∀ x, y ∈ R
¡ ¢
(5)
2 2
¡ ¡ ¢ ¡ ¢¢
BÀI 3 (VMO-2013). ¡ 2 Tìm tất¢ các hàm số f : R → R thỏa mãn f (0) = 0, f (1) = 2013 và ( x − y) f f ( x) − f f ( y) =
2
( f ( x) − f ( y)) f ( x) − f ( y) , ∀ x, y ∈ R .
BÀI
¡ 4 (DHĐBBB-2014). Tìm tất cả các hàm số f : R → R thỏa mãn
f x2 + y2 = x f ( x) + y f ( y) , ∀ x, y ∈ R.
¢
(1)
BÀI 5 (VMO 2002). Hãy¢ tìm tất cảc các hàm số f ( x) xác định trên tấp số thực R và thỏa mãn hệ thức
f ( y − f ( x)) = f x2002 − y − 2001.y. f ( x) , ∀ x, y ∈ R.
¡
(1)
BÀI 6 (Áo 1996). Tìm tất cả các hàm số f : R → R thỏa mãn điều kiện x2 f ( x) + f (1 − x) = 2 x − 24 , ∀ x ∈ R.
BÀI 7 (Hàn Quốc 2003). Tìm tất cả các hàm số f : R → R thỏa mãn:

f ( x − f ( y)) = f ( x) + x f ( y) + f ( f ( y)) , ∀ x, y ∈ R (4)

TOÁN CHUYÊN
BÀI 8 (Iran 1999). Xác định các hàm số f : R → R thỏa mãn

f ( f ( x) + y) = f x2 − y + 4 y f ( x), ∀ x, y ∈ R
¡ ¢

BÀI 9 (Belarus 1995). Tìm tất cả các hàm số f : R → R thỏa mãn

f ( f ( x + y)) = f ( x + y) + f ( x) f ( y )

BÀI 10 (VMO 2005). Hãy xác định tất cả các hàm số f : R → R thỏa mãn điều kiện

f ( f ( x − y)) = f ( x) f ( y) − f ( x) + f ( y) − x y, ∀ x, y ∈ R. (8)

BÀI 11 (AMM,E2176). Tìm tất cả các hàm số f : Q → Q thỏa mãn điều kiện
x+ y f ( x ) + f ( y)
µ ¶
f (2) = 2 và f = , ∀ x ̸= y.
x− y f ( x ) − f ( y)

BÀI 12. Tìm tất cả các hàm số f : Z → Z thỏa mãn điều kiện

f (2a) + 2 f ( b) = f ( f (a + b)) , ∀a, b ∈ Z.

2. Phương pháp thêm biến


Đôi khi, trong quá trình xử lí các bài toán phương trình hàm, ta có thể thêm một vài biến phụ vào để phép
thế trở nên linh hoạt hơn, từ đó phát hiện được nhiều tính chất thú vị của hàm giúp ích cho việc giải toán.
BÀI 1. Tìm tất cả các hàm số f : R → R thỏa mãn

f ( y + f ( x)) = f ( x) f ( y) + f ( f ( x)) + f ( y) − x y, ∀ x, y ∈ R

BÀI 2 (IMO shortlist, 2007). Tìm tất cả các hàm số f : R+ → R+ thỏa mãn

f ( x + f ( y)) = f ( x + y) + f ( y) , ∀ x, y ∈ R.

BÀI 3 (IMO shortlist, 2011). Tìm tất cả các hàm số f , g : R → R thỏa mãn:

f ( f ( x + y)) = f ( x) + (2 x + y) g( y), ∀ x, y ∈ R

ÓNguyễn Thành Nhân 57 Chuyên Hùng Vương-BD


Các Bài Giảng 2023-2024 Đại Số 10 Chuyên

BÀI 4 (IMO shortlist, 2011). Tìm tất cả các hàm số f , g : R → R thỏa mãn:

f ( x) x
µ ¶
f = , ∀ x, y > 0
y f ( x) + 1 x f ( y) + 1

BÀI 5 (VMO 2016). Tìm tất cả các số thực a để tồn tại hàm số f : R → R thỏa mãn:
i)
f (1) = 2016

ii)
f ( x + y + f ( y)) = f ( x) + a y, ∀ x, y ∈ R

BÀI 6 (VMO 2017). Tìm tất cả các hàm số f , : R → R thỏa mãn hệ thức:

f ( x f ( y) − f ( x)) = x y + 2 f ( y), ∀ y ∈ R

BÀI 7 (Việt Nam TST 2004). Tìm tất cả các giá trị của a, sao cho tồn tại duy nhất một hàm số f thỏa điều
kiện: f , : R → R thỏa mãn điều kiện:

f x2 + y + f ( y) = ( f ( x))2 + a y, ∀ x, y ∈ R
¡ ¢
LATEX:Nguyễn Thành Nhân

BÀI 8 (IMO shortlisted 1996). Chứng minh rằng tồn tại song ánh f : Z+ → Z+ sao cho

f (3 mn + m + n) = 4 f ( m) f ( n) + f ( m) + f ( n) ∀ m, n ∈ Z+ .

BÀI 9 (Áo 99- Việt Nam 2000). Tìm tất cả các hàm số f : R → R thỏa mãn điều kiện

x2 f ( x) + f (1 − x) = 2 x − x4 , ∀ x ∈ R.

1 − x2 , nếu x ̸= a, x ̸= b




ĐS: f ( x) = c ∈ R, nếu x = a ( c là hằng số tùy ý). với a, b là nghiệm của phương trình x2 − x − 1 = 0.

2a − a4 − a2 c, nếu x = b

BÀI 10 (Iran 99). Tìm tất cả các hàm số f : R → R thỏa mãn điều kiện

f ( f ( x) + y) = f ( x2 − y) + 4 y f ( x), ∀ x, y ∈ R.

ĐS: f ( x) ≡ 0.
BÀI 11 (Balarus 1995). Tìm tất cả các hàm số f : R → R thỏa mãn điều kiện

f ( f ( x + y)) = f ( x + y) + f ( x) f ( y) − x y, ∀ x, y ∈ R.

ĐS: f ( x) = x, ∀ x ∈ R.

3. Phương pháp quy nạp


Khi hàm số xác định trên N hoặc Z, để chứng minh các tính chất của nó, chúng ta thường dùng phương pháp
quy nạp.
Trong nhiều bài toán cũng phải xác định từng bước N → Z → Q → R. Riêng bước từ Q → R ta cần thêm tính
liên tục của hàm số.
Ý tưởng của phương pháp lày là ta dự đoán được công thức của hàm f (n) rồi chứng minh minh bằng quy
nạp.
Phương µ pháp này ta cần tính f (0), f (1) rồi dựa vào đó tính f (n) với n ∈ N. Sau đó tính f (n) với n ∈ Z. Tiếp đến
1

tính f , từ đó suy ra f (r ) với r ∈ Q. Phương pháp này thường sử dụng khi tìm hàm trên các tập rời rạc N,
n
Z, Q.

VÍ DỤ 1. Xác định hàm số f (n) : N → N thỏa mãn

f (1) > 0; f ( m2 + n2 ) = f 2 ( m) + f 2 ( n), ∀ m, n ∈ N.

ÓNguyễn Thành Nhân 58 Chuyên Hùng Vương-BD


Các Bài Giảng 2023-2024 Đại Số 10 Chuyên

VÍ DỤ 2. Xác định hàm số f (n) : Z → Z thỏa mãn



 f ( f ( n)) = n
 (1)
f ( f ( n + 2) + 2) = n, ∀ n ∈ Z (2)

f (0) = 1 (3)

VÍ DỤ 3. Xác định hàm số f (n) : N → N thỏa mãn

f ( f ( n)) + f ( n) = 2 n + 3, ∀ n ∈ N.

VÍ DỤ 4. Tìm tất cả hàm số f : N∗ → N∗ sao cho:

f ( m2 + n2 ) = f 2 ( m) + f 2 ( n) với mọi m, n ∈ N∗ ; f (1) = 1.

VÍ DỤ 5 (DHĐBBB-2010). Tìm tất cả các hàm số f : Q∗+ → Q∗+ (Q∗+ là tập hợp các số hữu tỉ dương) thỏa
mãn
f ( x y)
f ( x) + f ( y) + 2 x y f ( x y) = , ∀ x, y ∈ Q∗+ (1).
f ( x + y)

VÍ DỤ 6. Xác định hàm số liên tục f : (0; +∞) → (0; +∞) thỏa mãn đồng thời các điều kiện sau:

1 f (2 x) = 2 f ( x), (2.1).

TOÁN CHUYÊN
2 f ( f ( x))3 e f ( x) − 1 = x2 (e x − 1) f ( x), ∀ x > 0,
¡ ¡ ¢¢
(2.2).

3 f (e − 1) = (e − 1) f (1), (2.3).

4 f ( k) là số nguyên dương với mọi số nguyên k. (2.4).

VÍ DỤ 7. Gọi Q+ là tập các số hữu tỉ không âm, tìm tất các các hàm f : Q+ → Q+ thỏa mãn các điều
kiện sau

1) f ( x + 1) = f ( x) + 1, ∀ x. (3.1)

2) f ( x2 ) = f 2 ( x), ∀ x. (3.2)

VÍ DỤ 8. Tìm tất cả hàm số f : Z → Z sao cho


f ( x3 + y3 + z3 ) = f 3 ( x) + f 3 ( y) + f 3 ( z) với mọi x, y, z nguyên. (I)
Ý tưởng của lời giải là thiết lập các đẳng thức mà hai vế là tổng của 3 lập phương, sau đó sử dụng
đẳng thức này để tính toán một vài giá trị của hàm số, từ đó chứng minh f ( x) ≡ ax bằng quy nạp.

VÍ DỤ 9. Xác định hàm số f (n) : N → R thoả mãn

f (0) = β; f ( n + 1) = 2 f 2 ( n) − 1, ∀ n ∈ N.

VÍ DỤ 10. Xác định hàm số f (n) : N → R thoả mãn

b2 − 2 b
f ( n + 1) = a f 2 ( n) + b f ( n) + 1, ∀ n ∈ N, a ̸= 0; c = .
4a

VÍ DỤ 11. Tìm tất cả hàm số f : N → N sao cho

f ( x3 + y3 + z3 ) = f 3 ( x) + f 3 ( y) + f 3 ( z), ∀ x, y, z ∈ N. (II)

Biết rằng f (1) = 1, f (4) = 4, f (5) = 5.

ÓNguyễn Thành Nhân 59 Chuyên Hùng Vương-BD


Các Bài Giảng 2023-2024 Đại Số 10 Chuyên

VÍ DỤ 12. Tìm tất cả hàm số f : N → N thỏa mãn đồng thời hai điều kiện

(i) 2 f (m2 + n2 ) = f 2 (m) + f 2 (n), ∀ m, n ∈ N;

(ii) ∀m, n ∈ N sao cho m ≥ n thì f (m2 ) ≥ f (n2 ).

VÍ DỤ 13 (IMO 1982). Cho f : N → N thỏa mãn với mọi số tự nhiên m, n thì


"
0
f ( m + n) − f ( m) − f ( n) = ; f (2) = 0, f (3) > 0, f (9999) = 3333.
1

Tính f (1982).

VÍ DỤ 14. Tìm tất cả các hàm số f : N∗ → N∗ thỏa mãn đồng thời ba điều kiện

(i) f (2) = 2.
(ii) f (mn) = f (m) f (n), ∀m, n ∈ N∗ , (m, n) = 1.
(iii) f (m) < f (n), ∀m < n.
LATEX:Nguyễn Thành Nhân

VÍ DỤ 15. Tìm tất cả các hàm số f : Z → Z thỏa mãn đồng thời hai điều kiện
(i) f ( f (n)) = 4n + 3, ∀n ∈ Z.

(ii) f ( f (n) − n) = 2n + 3, ∀n ∈ Z.

BÀI 12. Tìm tất cả các hàm số f : N → N thỏa mãn điều kiện

f ( m + n2 + p3 ) = f ( m) + f 2 ( n) + f 3 ( p),

với mọi m, n, p ∈ N.
BÀI 13. Tìm tất cả các hàm số f : N → N thỏa mãn điều kiện

f ( m4 + 5 n4 + 10 p4 ) = f 4 ( m) + 5 f 4 ( n) + 10 f 4 ( p),

với mọi m, n, p ∈ N.
BÀI 14. Tìm tất cả các hàm số f : N → N thỏa mãn điều kiện

f ( f 2 ( m) + f 2 ( n)) = m2 + n2 ,

với mọi m, n ∈ N.
BÀI 15. Tìm tất cả các hàm số f : N → N thỏa mãn các điều kiện sau
(i) f (1) = 1.
1
(ii) f ( m + n) + f (m − n) = ( f (2 m) + f (2 n)), ∀m, n ∈ N.
2
BÀI 16. Tìm tất cả các hàm số f : N → N thỏa mãn các điều kiện sau
(i) f (1) = 2, f (2) = 4.
(ii) f ( f (m) + f (n)) = f ( f ( m)) + f ( n), ∀m, n ∈ N.
BÀI 17. Tồn tại hay không hàm số f : N → N thỏa mãn các điều kiện sau
(i) f (1) = 2.
(ii) f ( f (n)) = f (n) + n, ∀n ∈ N.
(iii) f (n) < f (n + 1)∀n ∈ N.
BÀI 18. Tìm tất cả các hàm f : Q → Q thỏa mãn các điều kiện

f (1) = 2, f ( x y) = f ( x) f ( y) − f ( x + y) + 1, ∀ x, y ∈ Q.

ÓNguyễn Thành Nhân 60 Chuyên Hùng Vương-BD


Các Bài Giảng 2023-2024 Đại Số 10 Chuyên

BÀI 19. Tìm tất cả các hàm số f : N → N thỏa mãn các điều kiện

f (1) > 0; f ( m2 + n2 ) = f 2 ( m) + f 2 ( n), ∀ m, n ∈ N.

ĐS: f ( x) = x, ∀ x ∈ N.
BÀI 20 (Nguyễn Trọng Tuấn). Tìm tất cả các hàm số f : N → N thỏa mãn các điều kiện

f ( m + f ( n)) = f ( m) + n, ∀ m, n ∈ N.

ĐS: f (n) = n, ∀n ∈ N.
BÀI 21 (Nguyễn Trọng Tuấn). Tìm tất cả các hàm số f : N → N thỏa mãn

f ( mn + 1) = m f ( n) + 2, ∀ m, n ∈ N.

ĐS: f (n) = 2n, ∀n ∈ N.

4. Phương pháp sử dụng tính đơn ánh, toàn ánh, song ánh
1 Nếu f : R → R là đơn ánh thì từ f ( x) = f ( y) ta ra suy được x = y.

2 Nếu f : R → R là toàn ánh thì với mỗi y ∈ R, tồn tại x ∈ R sao cho f ( x) = y.

3 Nếu f : R → R là song ánh thì ta sử dụng cả hai đặc trưng trên.

4 Nếu f : R → R là một hàm tăng thực sự (hoặc giảm thực sự) thì f là đơn ánh.

• Nếu một hàm số mà đơn ánh thì ta rất hay dùng tác động f vào cả hai vế.

TOÁN CHUYÊN
• Nếu f là một toàn ánh ta hay dùng: Tồn tại một số b sao cho f ( b) = 0, sau đó tìm b. Nếu quan hệ hàm
là hàm bậc nhất của biến ở vế phải thì ta hay dùng quan hệ này.

Lưu ý thêm
✓ Nếu f : R → R là một toàn ánh thì với mọi y ∈ Y , luôn tồn tại x ∈ X sao cho y = f ( x), tức là phương trình
y = f ( x) (theo ẩn x) luôn luôn có nghiệm.

✓ Nếu f là một hàm số đơn ánh thì rất hay sử dụng kỹ thuật tác động f vào hai vế phương trình hoặc sử
dụng tính chất f (φ( x)) = f (ϕ( x)) ⇒ φ( x) = ϕ( x).
✓ Nếu f là toàn ánh thì ta thường sử dụng tính chất tồn tại một số b sao cho f (b) = 0, sau đó tìm b.
✓ Nếu quan hệ hàm là hàm bậc nhất của biến ở vế phải thì có thể nghĩ đến tính đơn ánh, toàn ánh.
✓ Nếu f : R → R là toàn ánh và f ( x) = ϕ( x), ∀ x ∈ T , trong đó T là tập giá trị của hàm f thì f ( x) = ϕ( x), ∀ x ∈ R.
✓ Nếu một vế có chứa f ( x) và về còn lại chứa biến x ở bên ngoài thì thông thường f là đơn ánh.
✓ Nếu f : R → R là đơn ánh thì từ f ( x) = f ( y) suy ra x = y.
✓ Nếu f : R → R và f ( f ( x)) = ax + b (a ̸= 0) thì f là song ánh.

BÀI 22 (Bà Rịa-Vũng Tàu TST 2014). Tìm tất cả các hàm f : R → R sao cho

f ( x y + f ( x)) + f ( x − y f ( x)) = 2 x, ∀ x, y ∈ R (8)

ĐS: f ( x) = x, ∀ x ∈ R.
BÀI 23 (Trần Minh Hiền). Tìm tất cả các hàm số f : Q → Q thỏa mãn

f ( f ( x) + y) = x + f ( y), ∀ x, y ∈ Q (9)

ĐS: f ( x) = x; f ( x) = − x, ∀ x ∈ Q.
BÀI 24. Chứng minh không tồn tại hàm số f : R → R và thỏa mãn hệ thức

f ( x − 2018 f ( y)) = y − 2019 f ( x) ∀ x, y ∈ R (1)


1
BÀI 25. Cho hàm số f : R → R thỏa mãn điều kiện f x2 + f ( y) = 4 y + f 2 ( x) (1) với mọi x, y ∈ R. Chứng minh
¡ ¢
2
rằng f ( x) là hàm lẻ trên R.

ÓNguyễn Thành Nhân 61 Chuyên Hùng Vương-BD


Các Bài Giảng 2023-2024 Đại Số 10 Chuyên

BÀI 26. Tìm tất cả các hàm số f xác định trên tập R, lấy giá trị trong R và thỏa mãn phương trình

f ( x + y + f ( y)) = f ( f ( x)) + 2 y

với mọi số thực x, y.


BÀI 27. Tìm tất cả f : R+ → R thỏa mãn đồng thời hai điều kiện sau
1 f ( f ( x) + y) = x + f ( y) (1).

2 ∀ x ∈ R+ , ∃ y ∈ R+ sao cho f ( y) = x (2).

BÀI 28. Tìm tất cả các hàm số f : R → R và thỏa mãn hệ thức

f ( x + y) + f x2 + f ( y) = f 2 ( x) + f ( x) + f ( y) + y ∀ x, y ∈ R (1)
¡ ¢

BÀI 29. Tìm tất cả hàm số f : R → R thỏa mãn điều kiện

f ( f ( x) + y) = 2 x + f ( f ( y) − x)

với mọi x, y ∈ R.
BÀI 30. Tìm tất cả các hàm số f : R → R và thỏa mãn hệ thức

f ( x f ( x + y)) = f ( y f ( x)) + x2 ∀ x, y ∈ R (1)


LATEX:Nguyễn Thành Nhân

BÀI 31. Tìm tất cả các hàm số f : R → R thỏa mãn hệ thức

f x2 + f ( x y) = x f ( x + y) ∀ x, y ∈ R (1)
¡ ¢

Nhận xét. Có một số sai sót thường gặp


a a
µ ¶ µ ¶
1 Trong cách 2 học sinh lý luận vì f (− x2 ) = f − x2 − 2 với mọi x ̸= 0 mà f (0) = 0 nên f (− x2 ) = f − x2 − 2 ∀ x ∈
x x
R.
a a
µ ¶ µ ¶
2 Vì f − 3 = 0 (với x ̸= 0) mà f (0) = 0 nên f − 3 = 0 ∀ x ∈ R.
x x

3 Từ x f ( x) = − x f (− x) ∀ x ∈ R suy ra f ( x) = − f (− x) ∀ ∈ R. Có một số học sinh còn sử dụng tính chất x f ( x) =


− x f (− x)∀ x ∈ R để suy ra f (0) = 0; “nếu f là hàm số lẻ thì f đơn ánh”; “Nếu f không là hàm hằng thì chỉ
có hữu hạn điểm x sao cho f ( x) = 0”. Chúng ta có thể lấy hàm f ( x) = sin x để làm phản ví dụ cho các lập
luận trên.
BÀI 32 (Turkey TST 2014). Tìm tất cả hàm số f : R → R thỏa mãn điều kiện

f f ( y) + x2 + 1 + 2 x = y + f 2 ( x + 1) (1)
¡ ¢

với mọi x, y ∈ R.
BÀI 33. Tìm tất cả các hàm số f : R → R thỏa mãn đẳng thức

f ( x f ( y) + f ( x)) = 2 f ( x) + x y, ∀ x, y ∈ R. (1)

BÀI 34. (Morocco 2011) Tìm tất cả các hàm số f : R → R thỏa mãn điều kiện

( x − 2 f ( y) + f ( y + 2 f ( x))) = f ( x + y f ( x)), ∀ x, y ∈ R. (1)

BÀI 35. (IMO 2015) Tìm tất cả các hàm số f : R → R thỏa mãn

f ( x + f ( x + y)) + f ( x y) = x + f ( x + y) + y f ( x), ∀ x, y ∈ R. (1)

Bài tập tương tự


BÀI 1. (TST 2004 Việt Nam) Tìm tất cả các giá trị của a sao cho tồn tại duy nhất một hàm số f : R → R,
thỏa mãn ¡ 2 2
f x + y + f ( y) = f ( x) + a y, ∀ x, y ∈ R.
¢

BÀI 2. (VMO 2016) Tìm tất cả các giá trị của a sao cho tồn tại hàm số f : R → R thỏa mãn
1 f (1) = 2016

2 f ( x + y + f ( y)) = f ( x) + a y, ∀ x, y ∈ R.

ÓNguyễn Thành Nhân 62 Chuyên Hùng Vương-BD


Các Bài Giảng 2023-2024 Đại Số 10 Chuyên

BÀI 3. (VMO 2017) Tìm tất cả các hàm số f : R → R, thỏa mãn

f ( x f ( y) − f ( x)) = 2 f ( x) + x y, ∀ x, y ∈ R.

BÀI 4. Tìm tất cả các hàm f : R → R thỏa mãn điều kiện

f ( f ( x) + 2 f ( y)) = f ( x) + y + f ( y) , ∀ x, y ∈ R.

BÀI 5. (Bugaria 2003) Tìm tất cả các hàm số f : R → R thỏa mãn

f x2 + y + f ( y) = 2 y + f 2 ( x) , ∀ x, y ∈ R.
¡ ¢

BÀI 6. (Chọn đội tuyển PTNK TP Hồ Chí Minh 2013) Tìm tất cả các hàm số f : R → R thỏa mãn

f x3 + y + f ( y) = 2 y + x2 f ( x), ∀ x, y ∈ R.
¡ ¢

BÀI 7. (Moldava TST 2015) Tìm tất cả các hàm f : N → N thỏa mãn

f ( m f ( n)) = n + f (2015 m) , ∀ m, n ∈ N.

BÀI 8. (Turkey TST 2015) Tìm tất cả các hàm f : R → R sao cho

f x2 + 4 y2 f ( y) = f ( x − y) + y2 ( f ( x + y) + f ( y)) , ∀ x, y ∈ R.
¡ ¢ ¡ ¢

Sử dụng tính chất ánh xạ của hàm số giải một số bài tập phương trình hàm trên N, Z.

TOÁN CHUYÊN
BÀI 1. Tìm tất cả các hàm f : N∗ → N∗ thỏa mãn đẳng thức f ( f (m) + f ( n)) = m + n, với mọi m, n ∈ N∗ .
BÀI 2. Tìm tất cả các song ánh f , g, h : N∗ → N∗ thỏa mãn

f 3 ( n) + g3 ( n) + h3 ( n) = 3 ng ( n) h ( n) , ∀ n ∈ N∗ . (1)

BÀI 3. (TST Việt Nam 2014) Tìm tất cả các hàm f : Z → Z thỏa mãn

f (2 m + f ( m) + f ( m) f ( n)) = n f ( m) + m, ∀ m, n ∈ Z. (1)

BÀI 4. (Đề thi HSG Quốc gia 2012) Tìm tất cả các hàm số f xác định trên tập số thực R, lấy giá trị trong
R và thỏa mãn đồng thời các điều kiện sau

1 f là toàn ánh từ R đến R.

2 f là hàm số tăng trên R.

3 f ( f ( x)) = f ( x) + 12 x với mọi số thực x.

BÀI 5. Tìm tất cả các toàn ánh f : (0; +∞) → (0; +∞) thỏa mãn

2 x f ( f ( x)) = f ( x) ( x + f ( f ( x))) , ∀ x > 0. (5)

Kết hợp ánh xạ trong các bài toán khác


BÀI 1 (IMO Shortlist 2005). Tìm tất cả các hàm f : R+ → R+ sao cho

f ( x) f ( y) = 2 f ( x + y f ( x)) , ∀ x, y ∈ R+ . (1)

BÀI 2 (IMO Shortlist 2007). Tìm tất cả hàm số f : R+ → R+ sao cho

f ( x + f ( y)) = f ( x + y) + f ( y) , ∀ x, y ∈ R+ . (1)

BÀI 3 (IMO 2017). Tìm tất cả các hàm số f : R → R thỏa mãn

f ( f ( x) f ( y)) + f ( x + y) = f ( x y) , ∀ x, y ∈ R. (1)

BÀI 4 (Iran TST 2008). Cho k là một số nguyên dương. Tìm tất cả các hàm số f : N∗ → N∗ thỏa mãn điều kiện

f ( m) + f ( n) | ( m + n) k , ∀ m, n ∈ N∗ .

ÓNguyễn Thành Nhân 63 Chuyên Hùng Vương-BD


Các Bài Giảng 2023-2024 Đại Số 10 Chuyên

Bài tập tương tự


BÀI 1. Tìm tất cả các hàm f : R → R thỏa mãn

f ( x (1 + y)) = f ( x) (1 + f ( y)) .

BÀI 2. Tìm tất cả các hàm f : R → R thỏa mãn

f ( x f ( y) + f ( x)) = 2 f ( x) + x y.

BÀI 3. Tìm tất cả các hàm f : (0; +∞) → (0; +∞) thỏa mãn

x f ( x f ( y)) = f ( f ( y)) , ∀ x, y ∈ (0; +∞) .

BÀI 4. Tìm tất cả các hàm f : (0; +∞) → (0; +∞) thỏa mãn

f ( x + y) + f ( x y) = x + y + x y, ∀ x, y ∈ (0; +∞) .

BÀI 5. Tìm tất cả các hàm f : R → R thỏa mãn

f ( f ( x − y)) = f ( x) − f ( y) + f ( x) f ( y) , ∀ x, y ∈ R.

BÀI 6. Tìm tất cả các hàm f : R → R thỏa mãn


LATEX:Nguyễn Thành Nhân

f ( f ( x) − y) = f ( x) − f ( y) + f ( x) f ( y) − x y, ∀ x, y ∈ R.

BÀI 7. Tìm tất cả các hàm f : R → R thỏa mãn

f ( f ( x ) + y) = f ( x + y) + x f ( y) − x y − x + 1 , ∀ x, y ∈ R.

BÀI 8. Tìm tất cả các hàm f : R → R thỏa mãn

f ( f ( x + y)) = f ( x + y) + f ( x) f ( y) − x y, ∀ x, y ∈ R.

BÀI 9. Tìm tất cả các hàm f : R → R thỏa mãn

f x2 + x y + f ( y) = f 2 ( x) + x f ( y) + y, ∀ x, y ∈ R.
¡ ¢

BÀI 10. Tìm tất cả các hàm f : R → R thỏa mãn

( f ( x) + f ( z)) ( f ( y) + f ( t)) = f ( x y − zt) + f ( xt + yz) , ∀ x, y, z, t ∈ R.

BÀI 11. Tìm tất cả các hàm f : R → R thỏa mãn

f ( y − f ( x)) = x2 018 − y − 2017 y f ( x) , ∀ x, y ∈ R.


¡ ¢

BÀI 12. Tìm tất cả các hàm số f : R → R sao cho

f ( x) f ( y) = x f ( f ( y − x)) + x f (2 x) + f ( x2 ), ∀ x, y ∈ R.

BÀI 13. Cho hàm số f : R → R sao cho

( y + 1) f ( x) + f ( x f ( y) + f ( x + y)) = y ∀ x, y ∈ R.

1 Chứng minh rằng f (0) ̸= 1.

2 Tìm tất cả các hàm số f ( x) thỏa mãn điều kiện trên.

BÀI 14. Tìm tất cả các hàm số f : N∗ → N∗ thỏa mãn điều kiện

( f (a) + b) f (a + f ( b)) = (a + f ( b))2 , ∀a, b ∈ N∗ .

BÀI 15. Cho hàm số f ( x) liên tục trên [0; 1] thỏa mãn f (0) = 0, f (1) = 1 và
2x + y
µ ¶
6f = 5 f ( x) + f ( y), x ≥ y; x, y ∈ [0; 1] .
3

8
µ ¶
Hãy tính f .
23

ÓNguyễn Thành Nhân 64 Chuyên Hùng Vương-BD


Các Bài Giảng 2023-2024 Đại Số 10 Chuyên

BÀI 16. Tìm tất cả các hàm số f : R → R sao cho thỏa mãn
(
f (1) > 0
f ( x y − 1) + 2 f ( x) f ( y) = 3 x y − 1, ∀ x, y ∈ R.

BÀI 17. Tìm tất cả các hàm số f : R → R sao cho thỏa mãn

f ( x + y f ( x)) = x f ( y) + f ( x) , ∀ x, y ∈ R.

BÀI 18. Tìm tất cả các hàm số f : R → R sao cho

f ( x y) + f ( x − y) + f ( x + y + 1) = x y + 2 x + 1, ∀ x, y ∈ R.

BÀI 19. Tìm tất cả các hàm số đơn điệu trên R thỏa mãn

f x3 + f ( y) = f 3 ( x) + y, ∀ x, y ∈ R.
¡ ¢

BÀI 20. Tìm tất cả các hàm số f : R → R sao cho

f ( f ( x) + f ( y)) = f x2 + 2 x2 f ( y) + f 2 ( y) , ∀ x, y ∈ R.
¡ ¢

BÀI 21 (Indian 2015). Tìm tất cả các hàm f : R → R sao cho

f x 2 + y f ( x ) = x f ( x + y) , ∀ x, y ∈ R.
¡ ¢

BÀI 22 (Zhautykov Olympiad 2015). Tìm tất cả các hàm f : R → R sao cho

f x3 + y3 + x y = x2 f ( x) + y2 f ( y) + f ( x y) , ∀ x, y ∈ R.
¡ ¢

TOÁN CHUYÊN
BÀI 23 (Baltic Way 2014). Tìm tất cả các hàm f : R → R thỏa mãn ∀ x, y ∈ R ta có

f ( f ( y)) + f ( x − y) = f ( x f ( y) − x) .

BÀI 24 (Albania TST 2014). Tìm tất cả các hàm f : R → R thỏa mãn

f ( x) f ( y) = f ( x + y) + x y, ∀ x, y ∈ R.

BÀI 25 (European Girl’s Mathemmatical Olympiad 2014).


Tìm tất cả các hàm f : R → R thỏa mãn với mọi x, y ∈ R ta có

f y2 + 2 x f ( y) + f 2 ( x) = ( y + f ( x)) ( x + f ( y)) .
¡ ¢

BÀI 26 (Romanian Distric Olympiad 2014). Tìm tất cả các hàm f : N → N sao cho với mọi m, n ∈ N, ta có

f ( m + n) − 1 | f ( m) + f ( n) .

BÀI 27 (Korea 2014). Tìm tất cả các hàm f : R → R thỏa mãn với mọi x, y ∈ R, ta có

f ( x f ( x) + f ( x) f ( y) + y − 1) = f ( x f ( x) + x y) + y − 1.

BÀI 28 (Middle European Mathematical Olympiad 2014).


Tìm tất cả các hàm f : R → R thỏa mãn

x f ( y) + f ( x f ( y)) − x f ( f ( y) − f ( x y) = 2 x + f ( y) − f ( x + y).

BÀI 29 (Canada 2008). Tìm tất cả các hàm số f : Q → Q thỏa mãn đẳng thức:

f (2 f ( x) + f ( y)) = 2 x + y, ∀ x, y ∈ Q.

BÀI 30 (Mở rộng Canada 2008). Tìm tất cả các hàm số f : Q+ → Q+ thỏa mãn đẳng thức

f (2 f ( x) + f ( y)) = 2 x + y, x, y ∈ Q+ .

BÀI 31. Tìm tất cả các hàm số f : R → R thỏa mãn

f ( x f ( y) + x) = x y + f ( x), ∀ x, y ∈ R (12)

ĐS: f ( x) = x, f ( x) = − x trên R.

ÓNguyễn Thành Nhân 65 Chuyên Hùng Vương-BD


Các Bài Giảng 2023-2024 Đại Số 10 Chuyên

BÀI 32 (IMO Shortlist). Tìm tất cả các hàm số f : N → N thỏa mãn

f ( f ( n) + f ( m)) = m + n, ∀ m, n ∈ N (14)

ĐS: f (n) = n, ∀nN.


BÀI 33. Tìm tất cả các hàm số f : R → R thỏa mãn

f ( x + f ( y)) = x + f ( y) + x f ( y), ∀ x, y ∈ R.

ĐS: f ( x) ≡ −1, ∀ x ∈ R.
BÀI 34 (IMO Shortlist 2002). Tìm tất cả các hàm số f : R → R thỏa mãn

f ( f ( x) + y) = 2 x + f ( f ( y) − x), ∀ x, y ∈ R.

ĐS: f ( x) = x − a với a là hằng số.


BÀI 35. Tìm tất cả các hàm f : R → R tăng thực sự thỏa mãn

f ( f ( x) + y) = f ( x + y) + 1, ∀ x, y ∈ R.

ĐS: f ( x) = x + 1, ∀ x ∈ R.
BÀI 36. Tìm tất cả các hàm f : R → R tăng thực sự thỏa mãn
LATEX:Nguyễn Thành Nhân

f ( x f ( y)) = y f (2 x), ∀ x, y ∈ R.

ĐS: f ( x) = 2 x, ∀ x ∈ R.

BÀI 3. ÔN TẬP CHƯƠNG I


Nguyễn Thành Nhân- Chuyên Hùng Vương- Bình Dương

A. CƠ SỞ LÝ THUYẾT

1. Phương pháp sử dụng điểm bất động


Định nghĩa
Cho ánh xạ f : X → R, ( X ⊂ R). Điểm x0 ∈ X gọi là một điểm bất động của f nếu f ( x0 ) = x0 .
Tập hợp các điểm bất động của f được kí hiệu là F ix( f ).

VÍ DỤ 1.
1 Hàm số f (( x) = x2 − x + 1 có một điểm bất động, F ix( f ) = {1}.

2 Hàm số f ( x) = x3 có ba điểm bất động, F ix( f ) = {−1; 0; 1}.

3 Hàm số f ( x) = x + 2 không có điểm bất động, F ix( f ) = ∅.

Các định lí
Định lí 1. Mọi hàm số liên tục f : [a; b] → [a; b] có ít nhất một điểm bất động.
Chứng minh 1. Xét hàm số g : [a; b] → [a; b] được xác định bởi g( x) = f ( x) − x. Do f liên tục nên g là hàm liên
tục. Ta có f (a), f (b) ∈ [a; b] nên f (a) − a ≤ 0 và f (b) − b ≥ 0 hay g(a) g(b) = [ f (a) − a][ f (b) − b] ≤ 0. Theo định lí
Bozano-Cauchy thì luôn tồn tại ít nhất một điểm c ∈ [a; b] sao cho g( c) = 0, hay phương trình f ( x) − x = 0 có ít
nhất một nghiệm. Do đó tồn tại ít nhất một điểm x0 sao cho f ( x0 ) = x0 .
Định lí 2.
1 Nếu hàm số f : [a; b] → [a; b] có đạo hàm trên [a; b] và thỏa mãn | f ′ ( x)| < 1, ∀ x ∈ [a; b] thì khi đó f có duy
nhất một điểm bất động trên đoạn [a; b].
2 Cho số thực k với k ∈ (0; 1) và f : [a; b] → [a; b] là hàm số thỏa mãn | f ( x) − f ( y)| ≤ k.| x − y|, ∀ x ∈ [a; b]. Khi đó
f có duy nhất một điểm bất động.

ÓNguyễn Thành Nhân 66 Chuyên Hùng Vương-BD


Các Bài Giảng 2023-2024 Đại Số 10 Chuyên

Chứng minh 2.

1 Xét hàm số g : [a; b] → [a; b] được xác định bởi g( x) = f ( x) − x. Hiển nhiên g( x) là hàm liên tục. Theo định
lí (1), tồn tại ít nhất một điểm x0 ∈ [a; b] sao cho g( x0 ) = 0, mặt khác g′ ( x) = f ′ ( x) − 1 < 0, ∀ x ∈ [a; b], nên x0
là duy nhất.

2 Nhận xét f ( x) là một hàm liên tục trên đoạn [a; b]. Thật vậy, lấy dãy số tùy ý { xn } ⊂ [a; b] sao cho
lim xn = x0 . Ta có x0 ∈ [a; b] và | f ( xn ) − f ( x0 )| ≤ k.| xn − x0 | với mọi n ∈ N∗ . Do lim xn = x0 nên lim f ( xn ) = f ( x0 )
. Vậy f liên tục tục trên đoạn [a; b].
Theo định lí (1) mọi hàm số liên tục f : [a; b] → [a; b] tồn tại ít nhất một điểm bất động x0 . Giả sử x1 ∈ [a; b]
cũng là một điểm bất động của f . Ta có | f ( x1 ) − f ( x0 )| ≤ k.| x1 − x0 |, nhưng lại có f ( x1 ) = x1 , f ( x0 ) = x0 nên
| x1 − x0 | ≤ k.| x1 − x0 | , suy ra x1 = x0 .
Vậy f có duy nhất một điểm bất động.

Định lí 3.

1 Nếu f là một hàm giảm thật sự trên tập số thực X ⊂ R thì f không có nhiều hơn một điểm bất động trên
X.
f ( x)
2 Nếu hàm là một hàm đơn điệu thật sự trên tâp số thực X ⊂ R thì f không có nhiều hơn một điểm
x
bất động trên X .

Chứng minh 3.

1 Xét hàm số g : X → R được xác định bởi g( x) = f ( x) − x. Do f giảm thật sự nên g là một hàm giảm thật
sự trên X . Do đó, nếu tập giá trị g( X ) không chứa giá trị 0 thì hàm số f không có điểm bất động, nếu
g( X ) có chứa giá trị 0 thì hàm số f có đúng một điểm bất động.

TOÁN CHUYÊN
f ( x)
2 Tương tự, hàm đơn điệu thật sự trên tập số thực X ⊂ R. Do đó, nếu tập giá trị g( X ) không chứa giá
x
trị 1 thì hàm số f không có điểm bất động, nếu g( X ) có chứa giá trị 1 thì hàm số f có đúng một điểm
bất động.

Định lí 4. Cho F (u) là hàm một biến thực, h( x, y, z, t) là hàm cho trước của bốn biến x, y, z, t các định trên tập
X × X × R × R ( trong đó X ⊂ R ). Nếu hàm F ( u) có duy nhất một điểm bất động u 0 thì mọi nghiệm của phương
trình
F ( h( x, y, f ( x), f ( y))) = h( x, y, f ( x), f ( y)), ∀ x, y ∈ X (4.1)
( Trong đó f là hàm một biến cần tìm xác định trên X ) phải thỏa mãn phương trình: h( x, x, f ( x), f ( x)) = u0 .
Chứng minh 4. Giả sử f ( x) là một hàm thỏa mãn (4.1), khi đó đặt x = y ∈ R ta được

F ( h( x, x, f ( x), f ( x))) = h( x, x, f ( x), f ( x)), ∀ x ∈ X

Điều này chứng tỏ h( x, x, f ( x), f ( x)) là một điểm bất động của F với mọi x ∈ X . Nhưng hàm F (u) có duy nhất
một điểm bất động u0 . Do đó ta có:
h( x, x, f ( x), f ( x)) = u 0
Nhận xét: Khi ứng dụng định lí này để giải các phương trình dạng (4.1), ta cần chứng minh rằng phương
trình F ( u) = u có duy nhất một nghiệm u0 trong một miền nào đó chứa miền giá trị của g.
Định lí 5. Cho F (u) là hàm số liên tục của một biến thực, h( x, y, z, t) là hàm liên tục cho trước của bốn biến x,
y, z, t các định trên tập X × X × R × R ( trong đó X ⊂ R ). Nếu tập điểm bất động của hàm F ( u) là F ix(F ) là tập
không quá đếm được thì mọi nghiệm của phương trình

F ( h( x, y, f ( x), f ( y))) = h( x, y, f ( x), f ( y)), ∀ x, y ∈ X (4.2)

( Trong đó f là hàm liên tục một biến cần tìm xác định trên X ) phải thỏa mãn phương trình h( x, x, f ( x), f ( x)) =
u 0 với u 0 là điểm bất động nào đó thuộc F ix(F ).
Chứng minh 5. Giả sử f là một hàm liên tục thỏa mãn (4.2), khi đó đặt x = y ∈ R ta được

F ( h( x, x, f ( x), f ( x))) = h( x, x, f ( x), f ( x)), ∀ x ∈ X

Điều này chứng tỏ h( x, x, f ( x), f ( x)) thuộc tập F ix(F ), ∀ x ∈ X . Do h và f là các hàm liên tục nên g( x) =
h( x, x, f ( x), f ( x)) là một hàm liên tục trên X , hơn nữa X là một khoảng nên g( X ) phải là một tập con khác
rỗng của R. Nếu g( X ) có nhiều hơn một điểm thì g( X ) là một khoảng có độ đo dương và do đó g( X ) không
đếm được điều này mâu thuẫn với việc F ix(F ) là tập không quá đếm được, nên g( X ) không thể có nhiều hơn
một điểm. Vậy định lí đã được chứng minh.

ÓNguyễn Thành Nhân 67 Chuyên Hùng Vương-BD


Các Bài Giảng 2023-2024 Đại Số 10 Chuyên

Các bài toán


BÀI 1. Tìm tất cả các hàm f : R → R thỏa mãn:

i) f ( f ( x) + y) = x f ( y) + f ( f ( x) + f ( y)), ∀ x, y ∈ R

ii) f có một điểm bất động.

BÀI 2. Tìm tất cả các hàm f : R → R, thỏa mãn các điều kiện sau:

i) f là một hàm giảm thật sự trên R.

ii) f ( x + f ( y)) = f ( x) + y, ∀ x, y ∈ R.

BÀI 3 (IMO 1983). Tìm hàm số f : R → R thỏa mãn hai điều kiện sau:

lim f ( x) = 0 và f ( x f ( y)) = y f ( x), ∀ x, y ∈ R+ .


x→∞

BÀI 4 (IMO 1994). Tìm tất cả các hàm số f : (−1; +∞) → (−1; +∞) thỏa mãn đồng thời các điều kiện sau đây:

i) f ( x + f ( y) + x f ( y)) = y + f ( x) + y f ( x) ∀ x ∈ (−1; +∞).


f ( x)
ii) là hàm tăng thực sự trên các khoảng (−1; 0), (0; +∞).
LATEX:Nguyễn Thành Nhân

BÀI 5. Tìm tất cả các hàm số f xác định và liên tục trên R thỏa mãn phương trình sau:

( f ( x)( x + 2 y) + f ( y))3 = f ( x) + f ( y)( y + 2 x) ∀ x, y ∈ R.

BÀI 6. Tìm tất cả các hàm số f liên tục và xác định trên R thỏa mãn phương trình sau:

π sin y2 f ( x) + f ( y) = 2 x2 f ( y) + f ( x) ∀ x, y ∈ R.
¡ ¢ ¡ ¢

BÀI 7 (IMO 1996). Tìm tất cả các hàm số f : N → N sao cho:

f ( m + f ( n)) = f ( f ( m)) + f ( n) ∀ m, n ∈ N.

BÀI 8. Cho hàm số f : N → R sao cho tồn tại x0 ∈ N thỏa mãn tính các chất sau:

( i ) f ( x0 ) = x0 ( x0 là một điểm bất động của f ).

( ii ) f ( x0 + n) = x0 + f ( n) với mọi số nguyên không âm n.

m m
· ¸ µ · ¸ ¶
Chứng minh rằng f (m) = x0 + f m − x0 , ∀ m ∈ N∗ .
x0 x0
(trong đó [ x] là phần nguyên của x và không vượt quá x).
BÀI 9. Tìm tất cả các hàm số f : N → [1; +∞] thỏa mãn tính các chất sau:

( i ) f (3) = 8.

( ii ) f ( n) = f (3) f ( n − 3), ∀ n ∈ N.

BÀI 10. (AMM, E984) Tìm tất cả các hàm f : R → R sao cho f ( f ( x)) = x2 − 2, ∀ x ∈ R.

2. Lớp hàm liên tục (phần tham khảo thêm đối với lớp 10)

Mối quan hệ giữa giới hạn hàm và giới hạn dãy


Các bài toán phương trình hàm được giải bằng cách sử dụng một định lý quen thuộc của giải tích, thể hiện
mối quan hệ giữa giới hạn hàm và giới hạn dãy.
Định lí 6. Cho hàm số f xác định trên khoảng (a, b), x0 ∈ (a, b) tuỳ ý. Khi đó:

1 Điều kiện cần và đủ để lim f ( x) = A là với mọi dãy số (a n ) hội tụ về x0 ta đều có lim f (a n ) = A .
x→ x0 n→+∞

2 Điều kiện cần và đủ để f ( x) liên tục tại x0 là với mọi dãy số (a n ) hội tụ về x0 ta đều có lim f (a n ) = f ( x0 ).
n→+∞

ÓNguyễn Thành Nhân 68 Chuyên Hùng Vương-BD


Các Bài Giảng 2023-2024 Đại Số 10 Chuyên

Hàm liên tục cộng tính


Ta chú ý rằng tập hữu tỷ Q là trù mật trong tập số thực R, vì vậy khi giải các bài toán xác định hàm số f liên
tục trên tập R, bài toán sẽ coi như được giải xong nếu ta xác định được tất cả các giá trị f ( q), ∀ q ∈ Q. Trong
các bài toán ở mục này ta sẽ chỉ ra tính cộng tính của các hàm liên tục, sau đó áp dụng định lí dưới đây.
Định lí 7. Nếu f : R → R là một hàm liên tục, cộng tính:

f ( x + y) = f ( x) + f ( y), (1)

∀ x, y ∈ R thì f ( x) = ax với a là một hằng số tùy ý.


Chứng
( minh 6. Giả sử f : R → R là một nghiệm hàm. Lần lượt thay x = y = 0 và y = − x vào (1) suy ra
f (0) =0
.
f (− x ) = − f ( x )
Tiếp tục thay y = x và y = nx (với n ∈ N∗ ) tùy ý vào (1) suy ra
(
f (2 x) =2 f ( x)
f (( n + 1) x) = f ( x) + f ( nx).

Vì vậy bằng phương pháp quy nạp ta chứng minh được

f ( nx) = n f ( x), ∀ n ∈ N, ∀ x ∈ R. (2)


m
Hơn nữa do f ( x) là hàm lẻ ( f (− x) = − f ( x)) nên ta có (2) đúng với mọi n ∈ Z. Lại có ∀ q ∈ Q, ta viết q = ; m ∈
n
Z, n ∈ N∗ , ta được
m 1
 ³ ´ µ ¶
 f ( qx) = f x = mf x

TOÁN CHUYÊN

n n
1
µ ¶


 f ( x) = n f x .
n
m
Từ đó suy ra f ( qx) =f ( x) = q f ( x) , ∀ x ∈ R.
n
Cuối cùng, với mọi số thực r tùy ý ta viết r = lim q n , với { q n } là một dãy số hữu tỉ. Khi đó, f (rx) =
n→+∞
lim f ( q n x) = lim q n f ( x) = r f ( x) , ∀ x ∈ R.
n→+∞ n→+∞
Vậy f ( x) = x f (1) = ax, f (1) = a = const, x ∈ R. Thử lại ta có f ( x) = ax liên tục trên R và f ( x + y) = a( x + y) =
ax + a y = f ( x) + f ( y).

Định lý giới hạn kẹp


Định lí 8. Cho ba dãy số {a n } , {b n } , { c n }. Nếu a n ≤ b n ≤ c n , ∀n ∈ N∗ và lim a n = lim c n = L thì lim b n = L.
n→+∞ n→+∞ n→+∞

Hệ quả 1. Nếu |a n | ≤ b n và lim b n = 0 thì lim a n = 0.


n→+∞ n→+∞

Hàm liên tục đơn điệu


Định lí 9. Giả sử f là một hàm xác định và liên tục trên [a; b]. Khi đó với mọi số thực M nằm giữa f (a) và
f ( b) đều tồn tại ít nhất một số c ∈ (a; b) sao cho f ( c) = M .
Định lí 10. Giả sử f là một hàm xác định và liên tục trên (a; b). Khi đó nếu f là đơn ánh trên (a; b) thì f là
đơn điệu.
Chú ý 9. Định lý 5 vẫn đúng nếu ta thay khoảng (a; b) bởi đoạn [a; b] hoặc tập số thực R.

Một số bài tập áp dụng


BÀI 1. Tìm tất cả các hàm số liên tục f : R → R thỏa mãn đẳng thức

f ( x + y) = f ( x) + f ( y) − f ( x) f ( y) , ∀ x, y ∈ R. (1)

BÀI 2. Tìm tất cả các hàm số liên tục f : R → R thỏa mãn đồng thời các điều kiện sau:
i) f (1) = 2019
ii) f ( x + y) = 2019 x f ( y) + 2019 y f ( x) , ∀ x, y ∈ R. (1)

ÓNguyễn Thành Nhân 69 Chuyên Hùng Vương-BD


Các Bài Giảng 2023-2024 Đại Số 10 Chuyên

BÀI 3. Tìm tất cả các hàm số liên tục f : R → R thỏa mãn điều kiện

f ( x + y) + f ( x − y) = 2 [ f ( x) + f ( y)] , ∀ x, y ∈ R. (1)

Nhận xét 5. Trong 3 bài toán trên ta có cách giải chung như sau

✓ Thiết lập công thức cho hàm trong tập số tự nhiên.

✓ Thiết lập công thức cho hàm trong tập số nguyên.

✓ Thiết lập công thức cho hàm trong tập số hữu tỉ.
Từ đó chuyển qua giới hạn để thiết lập công thức tương tự cho hàm trong tập số thực. Chú ý rằng tính
liên tục không có tác dụng đối với các phương trình hàm trên tập số hữu tỉ. Tuy nhiên nếu biết chắc
chắn là hàm liên tục, ta có thể thiết lập công thức cho hàm trong tập số hữu tỉ và suy ra công thức
tương tự trong tập số thực.

BÀI 4. Tìm tất cả các hàm số liên tục f : R → R thỏa mãn

( x + y) f ( x + y) = x f ( x) + y f ( y) + 2 x y , ∀ x, y ∈ R (1)
BÀI 5. Tìm tất cả các hàm số liên tục f : R → R thỏa mãn:

f ( f ( x)) = f ( x) + 2 x, ∀ x ∈ R
LATEX:Nguyễn Thành Nhân

BÀI 6. Tìm tất cả các hàm số liên tục f : [0; +∞) → [0; +∞) thỏa mãn đẳng thức

x x+1
µ ¶ µ ¶
2 f ( x) = f + f , ∀ x ∈ [0; +∞) . (1)
x2 + x + 1 2

BÀI 7. Tìm tất cả các hàm số liên tục f : R → R thỏa mãn đẳng thức

f ( x + f ( y + z)) + f ( y + f ( z + x)) + f ( z + f ( x + y)) = 0, ∀ x, y, z ∈ R (1)

BÀI 8. Cho hàm số g : R → R là một hàm liên tục sao cho lim ( g ( x) − x) = ∞ và sao cho tập { x, g ( x) = x} là một
x→+∞
tập hữu hạn khác rỗng. Chứng minh rằng nếu f : R → R là liên tục và f ◦ g = f thì f là một hàm hằng.
BÀI 9. Cho hàm số liên tục f : [−1; 1] → R thỏa mãn

f 2 x2 − 1 = 2 x f ( x), ∀ x ∈ [−1; 1].


¡ ¢

Chứng minh rằng f ≡ 0.


Nhận xét 6. Từ bài 4 đến bài 9 ta đã sử dụng dãy số vào giải phương trình hàm. Trong quá trình sử dụng
dãy số vào giải phương trình hàm ta lưu ý một số vấn đề sau

✓ Trong nhiều trường hợp, ta cần tìm công thức tổng quát của hàm số, khi đó một trong các hướng đi mà
ta nghĩ đến là thiết lập một bất đẳng thức dạng a n ≤ f ( x) ≤ b n , ở đây (a n ), (b n ) là hai dãy được chọn sao
cho bất đẳng thức trên đúng với mọi n (ứng với mỗi x cố định). Lúc này, nếu lim a n = lim b n = L( x) thì
bằng cách chuyển sang giới hạn, ta sẽ tìm được công thức tổng quát của f ( x) và L( x).

✓ Nếu cần suy xét một tính chất nào đó của f ( x), ta có thể thiết lập một đẳng thức dạng A ( f ) ≥ a n B( f ),
trong đó A ( f ), B( f ) là hai biểu thức của x và f ( x), còn (a n ) là dãy được chọn sao cho bất đẳng thức trên
đúng với mọi n (ứng với mỗi x cố định). Lúc này, dựa trên sự hội tụ của a n ta có thể đưa ra nhiều kết
luận cho A ( f ) và B( f ), từ đó suy ra tính chất của f ( x).

BÀI 10 (1D0K2). TÌm tất cả các hàm số liên tục f : R → R thỏa mãn

{ f ( x + y)} = { f ( x) + f ( y)} , ∀ x, y ∈ R. (1)

(trong đó [ t] là số nguyên lớn nhất không vượt quá t và { t} = t − [ t]).


BÀI 11 (1D0K2). Tìm tất cả các hàm số liên tục f : R → R thỏa mãn

f ( x y) + f ( x + y) = f ( x y + x) + f ( y), ∀ x, y ∈ R. (1)

BÀI 12 (1D0K2). Tìm tất cả các hàm số liên tục f : R → R thỏa mãn

f ( x − y) f ( y − z) f ( z − x) + 8 = 0, ∀ x, y, z ∈ R. (1)

ÓNguyễn Thành Nhân 70 Chuyên Hùng Vương-BD


Các Bài Giảng 2023-2024 Đại Số 10 Chuyên

Nhận xét 7. Từ bài tập 10 đến bài tập 12 ta đã sử dụng phương trình hàm Cauchy vào giải phương trình
hàm.
BÀI 13. Tìm tất cả các hàm số liên tục f : [0; 1] → R thỏa mãn
f ( x) ≥ 2 x f x2 , ∀ x ∈ [0; 1].
¡ ¢
(1)
Nhận xét 8. Trong bài toán trên ta đã sử dụng định lý giới hạn kẹp để giải.
BÀI 14. Tìm tất cả các hàm số liên tục f : R → R thỏa mãn, với mọi số thực a, b sao cho a < b, luôn tồn tại
c ∈ (a; b) sao cho f ( c) ≥ max { f (a); f ( b)}.
Chú ý 10. Trong bài toán trên ta đã sử dụng phương pháp phần tử cực biên để giải bài toán.
BÀI 15. Xác định hàm số f : R → R liên tục trên R và thỏa mãn
f ( f ( x)) = f ( x) + 6 x ∀ x ∈ R. (1)

BÀI 16. Xác định hàm số f : R → R liên tục trên R và thỏa mãn
f ( x + 9 f ( y)) = 4 y + f ( x), ∀ x, y ∈ R. (1)
1
µ ¶
BÀI 17. Cho a, b là hai số thực trong tập 0; và hàm số f : R → R là một hàm liên tục thỏa mãn f ( f ( x)) =
2
a f ( x) + bx, ∀ x ∈ R. Chứng minh rằng tồn tại một hằng số thực c sao cho f ( x) = cx.
Nhận xét 9. Trong một số bài toán trên ta đã áp dụng các định lí về hàm liên tục đơn điệu để giải bài tập.

Bài tập tự luyện


BÀI 1. Tìm tất cả các hàm số f : R → R liên tục trên R và thỏa mãn

TOÁN CHUYÊN
³x´ x
f ( x) − f = , ∀ x ∈ R.
3 3
BÀI 2. Tìm tất cả các hàm số f : [0, 1] → [0, 1] liên tục trên đoạn [0, 1] và thỏa mãn
³x´ µ
x+1

2 f ( x) = f +f , ∀ x ∈ [0, 1].
2 2

BÀI 3. Tìm tất cả các hàm số f : R → R liên tục trên R và thỏa mãn
f ( x + y) = f ( x) + f ( y) + 2ax y, (a = const) ∀ x, y ∈ R.

BÀI 4. Tìm tất cả các hàm số f : R → R liên tục trên R và thỏa mãn
f ( f ( f ( x + y + x y))) = f ( x) + f ( y) + f ( x y) ∀ x, y ∈ R.

BÀI 5. Tìm tất cả các hàm số f : R → R liên tục trên R và thỏa mãn
f ( x + y) + f ( x y ) = f ( y) + f ( x y + x ) ∀ x, y ∈ R.

BÀI 6. Tìm tất cả các hàm số f : R → R liên tục trên R và thỏa mãn
f ( x + y − f ( y)) = f ( x) + f ( y − f ( y)) ∀ x, y ∈ R.

BÀI 7. Tìm tất cả các hàm số liên tục f : [0, +∞) → [0, +∞) thỏa mãn
x 1+ x
µ ¶ µ ¶
2 f ( x) = f + f , vói mọi x ∈ [0; +∞).
1 + x + x2 2

BÀI 8. Tìm tất cả các hàm số f : R → R liên tục trên R và thỏa mãn
q p
f ( x4 + y4 ) = f x2 − y2 + f ( 2 x y) ∀ x, y ∈ R.
¡ ¢

BÀI 9. Tìm tất cả các hàm số f : R → R liên tục tại 0 và thỏa mãn
f ( x + y)[ f ( x) + 1][ f ( y) + 1] = [ f ( x) + f ( y) + 2 f ( x) f ( y)][ f ( x + y) + 1] ∀ x, y ∈ R.

BÀI 10. Tìm tất cả các hàm số f : R → R liên tục trên R và thỏa mãn
f ( f ( f ( x + y + x y))) = f ( x) + f ( y) + f ( x y) ∀ x, y ∈ R.

BÀI 11. Tìm tất cả các hàm số f : R → R liên tục trên R và thỏa mãn
f ( x + f ( y + f ( z))) = f ( x) + f ( f ( y)) + f ( f ( f ( z))) ∀ x, y ∈ R.

ÓNguyễn Thành Nhân 71 Chuyên Hùng Vương-BD


Các Bài Giảng 2023-2024 Đại Số 10 Chuyên

BÀI 4. KĨ THUẬT THẾ GIÁ TRỊ ĐẶC GIẢI PHƯƠNG TRÌNH HÀM
Nguyễn Thành Nhân- Chuyên Hùng Vương- Bình Dương

A. LÝ THUYẾT CẦN NẮM


Phương pháp thế giá trị đặc biệt
Định nghĩa 1. Hàm số f ( x) được gọi là cộng tính trên tập xác định trên D nếu với mọi x, y ∈ D thì x + y ∈ D và
f ( x + y) = f ( x) + f ( y).
Hàm số f ( x) được gọi là nhân tính trên tập xác định trên D nếu với mọi x, y ∈ D thì x.y ∈ D và f ( x.y) = f ( x) . f ( y).
Trong khi giải phương trình hàm, chúng ta thường sử dụng phép thế đặc biệt sau đây để thu thập thông tin

x = 0; y = 0; x = y = 0; x = a; (a ∈ R); x = y = a; ; y = − x; y = f ( x); . . .

BÀI 1 (Trần Minh Hiền). Tìm tất cả các hàm số f : R → R thỏa mãn

f ( f ( x) − y) = f ( x) − f ( y) + f ( x) f ( y) − x y, ∀ x, y ∈ R. (1)
LATEX:Nguyễn Thành Nhân

BÀI 2 (Trần Minh Hiền). Tìm tất cả các hàm số f : R → R thỏa mãn

f ( x − y)2 = ( f ( x))2 − 2 x f ( y) + y2 , ∀ x, y ∈ R
¡ ¢
(5)

các hàm số f : R → R thỏa mãn f (0) = 0, f (1) = 2013 và ( x − y) f f 2 ( x) − f f 2 ( y) =


¡ ¡ ¢ ¡ ¢¢
BÀI 3 (VMO-2013). Tìm tất
( f ( x) − f ( y)) f ( x) − f 2 ( y) , ∀ x, y ∈ R .
¡ 2 ¢

BÀI
¡ 4 (DHĐBBB-2014). Tìm tất cả các hàm số f : R → R thỏa mãn
f x2 + y2 = x f ( x) + y f ( y) , ∀ x, y ∈ R.
¢
(1)
BÀI 5 (VMO 2002). Hãy¢ tìm tất cảc các hàm số f ( x) xác định trên tấp số thực R và thỏa mãn hệ thức
f ( y − f ( x)) = f x2002 − y − 2001.y. f ( x) , ∀ x, y ∈ R.
¡
(1)
2 4
BÀI 6 (Áo 1996). Tìm tất cả các hàm số f : R → R thỏa mãn điều kiện x f ( x) + f (1 − x) = 2 x − 2 , ∀ x ∈ R.
BÀI 7 (Hàn Quốc 2003). Tìm tất cả các hàm số f : R → R thỏa mãn:

f ( x − f ( y)) = f ( x) + x f ( y) + f ( f ( y)) , ∀ x, y ∈ R (4)

BÀI 8 (Iran 1999). Xác định các hàm số f : R → R thỏa mãn

f ( f ( x) + y) = f x2 − y + 4 y f ( x), ∀ x, y ∈ R
¡ ¢

BÀI 9 (Belarus 1995). Tìm tất cả các hàm số f : R → R thỏa mãn

f ( f ( x + y)) = f ( x + y) + f ( x) f ( y )

BÀI 10 (VMO 2005). Hãy xác định tất cả các hàm số f : R → R thỏa mãn điều kiện

f ( f ( x − y)) = f ( x) f ( y) − f ( x) + f ( y) − x y, ∀ x, y ∈ R. (8)

BÀI 11 (AMM,E2176). Tìm tất cả các hàm số f : Q → Q thỏa mãn điều kiện
x+ y f ( x) + f ( y)
µ ¶
f (2) = 2 và f = , ∀ x ̸= y.
x− y f ( x) − f ( y)

BÀI 12. Tìm tất cả các hàm số f : Z → Z thỏa mãn điều kiện

f (2a) + 2 f ( b) = f ( f (a + b)) , ∀a, b ∈ Z.

BÀI 5. GIẢI PHƯƠNG TRÌNH HÀM BẰNG PHƯƠNG PHÁP


QUY NẠP
Nguyễn Thành Nhân- Chuyên Hùng Vương- Bình Dương

ÓNguyễn Thành Nhân 72 Chuyên Hùng Vương-BD


Các Bài Giảng 2023-2024 Đại Số 10 Chuyên

1. Phương pháp quy nạp


Khi hàm số xác định trên N hoặc Z, để chứng minh các tính chất của nó, chúng ta thường dùng phương pháp
quy nạp.
Trong nhiều bài toán cũng phải xác định từng bước N → Z → Q → R. Riêng bước từ Q → R ta cần thêm tính
liên tục của hàm số.
Ý tưởng của phương pháp lày là ta dự đoán được công thức của hàm f (n) rồi chứng minh minh bằng quy
nạp.
Phương µ pháp này ta cần tính f (0), f (1) rồi dựa vào đó tính f (n) với n ∈ N. Sau đó tính f (n) với n ∈ Z. Tiếp đến
1

tính f , từ đó suy ra f (r ) với r ∈ Q. Phương pháp này thường sử dụng khi tìm hàm trên các tập rời rạc N,
n
Z, Q.

VÍ DỤ 1. Xác định hàm số f : N → N thỏa mãn

f (1) > 0; f ( m2 + n2 ) = f 2 ( m) + f 2 ( n), ∀ m, n ∈ N.

VÍ DỤ 2. Xác định hàm số f : Z → Z thỏa mãn



 f ( f ( n)) = n
 (1)
f ( f ( n + 2) + 2) = n, ∀ n ∈ Z (2)

f (0) = 1 (3)

VÍ DỤ 3. Xác định hàm số f : N → N thỏa mãn

TOÁN CHUYÊN
f ( f ( n)) + f ( n) = 2 n + 3, ∀ n ∈ N.

VÍ DỤ 4. Tìm tất cả hàm số f : N∗ → N∗ sao cho:

f ( m2 + n2 ) = f 2 ( m) + f 2 ( n) với mọi m, n ∈ N∗ ; f (1) = 1.

VÍ DỤ 5 (DHĐBBB-2010). Tìm tất cả các hàm số f : Q∗+ → Q∗+ (Q∗+ là tập hợp các số hữu tỉ dương) thỏa
mãn
f ( x y)
f ( x) + f ( y) + 2 x y f ( x y) = , ∀ x, y ∈ Q∗+ (1).
f ( x + y)

VÍ DỤ 6. Xác định hàm số liên tục f : (0; +∞) → (0; +∞) thỏa mãn đồng thời các điều kiện sau:

1 f (2 x) = 2 f ( x), (2.1).

2 f ( f ( x))3 e f ( x) − 1 = x2 (e x − 1) f ( x), ∀ x > 0,


¡ ¡ ¢¢
(2.2).

3 f (e − 1) = (e − 1) f (1), (2.3).

4 f ( k) là số nguyên dương với mọi số nguyên k. (2.4).

VÍ DỤ 7. Gọi Q+ là tập các số hữu tỉ không âm, tìm tất các các hàm f : Q+ → Q+ thỏa mãn các điều
kiện sau
1) f ( x + 1) = f ( x) + 1, ∀ x. (3.1)

2) f ( x2 ) = f 2 ( x), ∀ x. (3.2)

VÍ DỤ 8. Tìm tất cả hàm số f : Z → Z sao cho


f ( x3 + y3 + z3 ) = f 3 ( x) + f 3 ( y) + f 3 ( z) với mọi x, y, z nguyên. (I)
Ý tưởng của lời giải là thiết lập các đẳng thức mà hai vế là tổng của 3 lập phương, sau đó sử dụng
đẳng thức này để tính toán một vài giá trị của hàm số, từ đó chứng minh f ( x) ≡ ax bằng quy nạp.

ÓNguyễn Thành Nhân 73 Chuyên Hùng Vương-BD


Các Bài Giảng 2023-2024 Đại Số 10 Chuyên

VÍ DỤ 9. Xác định hàm số f (n) : N → R thoả mãn

f (0) = β; f ( n + 1) = 2 f 2 ( n) − 1, ∀ n ∈ N.

VÍ DỤ 10. Xác định hàm số f (n) : N → R thoả mãn

b2 − 2 b
f ( n + 1) = a f 2 ( n) + b f ( n) + 1, ∀ n ∈ N, a ̸= 0; c = .
4a

VÍ DỤ 11. Tìm tất cả hàm số f : N → N sao cho

f ( x3 + y3 + z3 ) = f 3 ( x) + f 3 ( y) + f 3 ( z), ∀ x, y, z ∈ N. (II)

Biết rằng f (1) = 1, f (4) = 4, f (5) = 5.

VÍ DỤ 12. Tìm tất cả hàm số f : N → N thỏa mãn đồng thời hai điều kiện
(i) 2 f (m2 + n2 ) = f 2 (m) + f 2 (n), ∀ m, n ∈ N;
LATEX:Nguyễn Thành Nhân

(ii) ∀m, n ∈ N sao cho m ≥ n thì f (m2 ) ≥ f (n2 ).

VÍ DỤ 13 (IMO 1982). Cho f : N → N thỏa mãn với mọi số tự nhiên m, n thì


"
0
f ( m + n) − f ( m) − f ( n) = ; f (2) = 0, f (3) > 0, f (9999) = 3333.
1

Tính f (1982).

VÍ DỤ 14. Tìm tất cả các hàm số f : N∗ → N∗ thỏa mãn đồng thời ba điều kiện
(i) f (2) = 2.
(ii) f (mn) = f (m) f (n), ∀m, n ∈ N∗ , (m, n) = 1.

(iii) f (m) < f (n), ∀m < n.

VÍ DỤ 15. Tìm tất cả các hàm số f : Z → Z thỏa mãn đồng thời hai điều kiện
(i) f ( f (n)) = 4n + 3, ∀n ∈ Z.
(ii) f ( f (n) − n) = 2n + 3, ∀n ∈ Z.

2. Bài tập
BÀI 1. Tìm tất cả các hàm số f : N → N thỏa mãn điều kiện

f ( m + n2 + p3 ) = f ( m) + f 2 ( n) + f 3 ( p),

với mọi m, n, p ∈ N.
BÀI 2. Tìm tất cả các hàm số f : N → N thỏa mãn điều kiện

f ( m4 + 5 n4 + 10 p4 ) = f 4 ( m) + 5 f 4 ( n) + 10 f 4 ( p),

với mọi m, n, p ∈ N.
BÀI 3. Tìm tất cả các hàm số f : N → N thỏa mãn điều kiện

f ( f 2 ( m) + f 2 ( n)) = m2 + n2 ,

với mọi m, n ∈ N.

ÓNguyễn Thành Nhân 74 Chuyên Hùng Vương-BD


Các Bài Giảng 2023-2024 Đại Số 10 Chuyên

BÀI 4. Tìm tất cả các hàm số f : N → N thỏa mãn các điều kiện sau
(i) f (1) = 1.
1
(ii) f (m + n) + f (m − n) = ( f (2m) + f (2n)), ∀m, n ∈ N.
2
BÀI 5. Tìm tất cả các hàm số f : N → N thỏa mãn các điều kiện sau
(i) f (1) = 2, f (2) = 4.
(ii) f ( f (m) + f (n)) = f ( f (m)) + f (n), ∀m, n ∈ N.
BÀI 6. Tồn tại hay không hàm số f : N → N thỏa mãn các điều kiện sau
(i) f (1) = 2.
(ii) f ( f (n)) = f (n) + n, ∀n ∈ N.
(iii) f (n) < f (n + 1)∀n ∈ N.
BÀI 7. Tìm tất cả các hàm f : Q → Q thỏa mãn các điều kiện

f (1) = 2, f ( x y) = f ( x) f ( y) − f ( x + y) + 1, ∀ x, y ∈ Q.

BÀI 8. Tìm tất cả các hàm số f : N → N thỏa mãn các điều kiện

f (1) > 0; f ( m2 + n2 ) = f 2 ( m) + f 2 ( n), ∀ m, n ∈ N.

ĐS: f ( x) = x, ∀ x ∈ N.
BÀI 9 (Nguyễn Trọng Tuấn). Tìm tất cả các hàm số f : N → N thỏa mãn các điều kiện

TOÁN CHUYÊN
f ( m + f ( n)) = f ( m) + n, ∀ m, n ∈ N.

ĐS: f (n) = n, ∀n ∈ N.
BÀI 10 (Nguyễn Trọng Tuấn). Tìm tất cả các hàm số f : N → N thỏa mãn

f ( mn + 1) = m f ( n) + 2, ∀ m, n ∈ N.

ĐS: f (n) = 2n, ∀n ∈ N.

BÀI 6. GIẢI PHƯƠNG TRÌNH HÀM BẰNG PHƯƠNG PHÁP SỬ


DỤNG TÍNH ĐƠN ÁNH-TOÀN ÁNH-SONG ÁNH
Nguyễn Thành Nhân- Chuyên Hùng Vương- Bình Dương

Sử dụng tính chất ánh xạ của hàm số giải một số bài tập phương trình hàm trên N, Z.
BÀI 1. Tìm tất cả các hàm f : N∗ → N∗ thỏa mãn đẳng thức f ( f (m) + f (n)) = m + n, với mọi m, n ∈ N∗ .
BÀI 2. Tìm tất cả các song ánh f , g, h : N∗ → N∗ thỏa mãn

f 3 ( n) + g3 ( n) + h3 ( n) = 3 ng ( n) h ( n) , ∀ n ∈ N∗ . (1)

BÀI 3. (TST Việt Nam 2014) Tìm tất cả các hàm f : Z → Z thỏa mãn

f (2 m + f ( m) + f ( m) f ( n)) = n f ( m) + m, ∀ m, n ∈ Z. (1)

BÀI 4. (Đề thi HSG Quốc gia 2012) Tìm tất cả các hàm số f xác định trên tập số thực R, lấy giá trị trong
R và thỏa mãn đồng thời các điều kiện sau

1 f là toàn ánh từ R đến R.

2 f là hàm số tăng trên R.

3 f ( f ( x)) = f ( x) + 12 x với mọi số thực x.

BÀI 5. Tìm tất cả các toàn ánh f : (0; +∞) → (0; +∞) thỏa mãn

2 x f ( f ( x)) = f ( x) ( x + f ( f ( x))) , ∀ x > 0. (5)

ÓNguyễn Thành Nhân 75 Chuyên Hùng Vương-BD


Các Bài Giảng 2023-2024 Đại Số 10 Chuyên

Kết hợp ánh xạ trong các bài toán khác


BÀI 1 (IMO Shortlist 2005). Tìm tất cả các hàm f : R+ → R+ sao cho

f ( x) f ( y) = 2 f ( x + y f ( x)) , ∀ x, y ∈ R+ . (1)

BÀI 2 (IMO Shortlist 2007). Tìm tất cả hàm số f : R+ → R+ sao cho

f ( x + f ( y)) = f ( x + y) + f ( y) , ∀ x, y ∈ R+ . (1)

BÀI 3 (IMO 2017). Tìm tất cả các hàm số f : R → R thỏa mãn

f ( f ( x) f ( y)) + f ( x + y) = f ( x y) , ∀ x, y ∈ R. (1)

BÀI 4 (Iran TST 2008). Cho k là một số nguyên dương. Tìm tất cả các hàm số f : N∗ → N∗ thỏa mãn điều kiện

f ( m) + f ( n) | ( m + n) k , ∀ m, n ∈ N∗ .

Bài tập tương tự


BÀI 1. Tìm tất cả các hàm f : R → R thỏa mãn

f ( x (1 + y)) = f ( x) (1 + f ( y)) .
LATEX:Nguyễn Thành Nhân

BÀI 2. Tìm tất cả các hàm f : R → R thỏa mãn

f ( x f ( y) + f ( x)) = 2 f ( x) + x y.

BÀI 3. Tìm tất cả các hàm f : (0; +∞) → (0; +∞) thỏa mãn

x f ( x f ( y)) = f ( f ( y)) , ∀ x, y ∈ (0; +∞) .

BÀI 4. Tìm tất cả các hàm f : (0; +∞) → (0; +∞) thỏa mãn

f ( x + y) + f ( x y) = x + y + x y, ∀ x, y ∈ (0; +∞) .

BÀI 5. Tìm tất cả các hàm f : R → R thỏa mãn

f ( f ( x − y)) = f ( x) − f ( y) + f ( x) f ( y) , ∀ x, y ∈ R.

BÀI 6. Tìm tất cả các hàm f : R → R thỏa mãn

f ( f ( x) − y) = f ( x) − f ( y) + f ( x) f ( y) − x y, ∀ x, y ∈ R.

BÀI 7. Tìm tất cả các hàm f : R → R thỏa mãn

f ( f ( x) + y) = f ( x + y) + x f ( y) − x y − x + 1, ∀ x, y ∈ R.

BÀI 8. Tìm tất cả các hàm f : R → R thỏa mãn

f ( f ( x + y)) = f ( x + y) + f ( x) f ( y) − x y, ∀ x, y ∈ R.

BÀI 9. Tìm tất cả các hàm f : R → R thỏa mãn

f x2 + x y + f ( y) = f 2 ( x) + x f ( y) + y, ∀ x, y ∈ R.
¡ ¢

BÀI 10. Tìm tất cả các hàm f : R → R thỏa mãn

( f ( x) + f ( z)) ( f ( y) + f ( t)) = f ( x y − zt) + f ( xt + yz) , ∀ x, y, z, t ∈ R.

BÀI 11. Tìm tất cả các hàm f : R → R thỏa mãn

f ( y − f ( x)) = x2 018 − y − 2017 y f ( x) , ∀ x, y ∈ R.


¡ ¢

BÀI 12. Tìm tất cả các hàm số f : R → R sao cho

f ( x) f ( y) = x f ( f ( y − x)) + x f (2 x) + f ( x2 ), ∀ x, y ∈ R.

ÓNguyễn Thành Nhân 76 Chuyên Hùng Vương-BD


Các Bài Giảng 2023-2024 Đại Số 10 Chuyên

BÀI 13. Cho hàm số f : R → R sao cho

( y + 1) f ( x) + f ( x f ( y) + f ( x + y)) = y ∀ x, y ∈ R.

1 Chứng minh rằng f (0) ̸= 1.

2 Tìm tất cả các hàm số f ( x) thỏa mãn điều kiện trên.

BÀI 14. Tìm tất cả các hàm số f : N∗ → N∗ thỏa mãn điều kiện

( f (a) + b) f (a + f ( b)) = (a + f ( b))2 , ∀a, b ∈ N∗ .

BÀI 15. Cho hàm số f ( x) liên tục trên [0; 1] thỏa mãn f (0) = 0, f (1) = 1 và
2x + y
µ ¶
6f = 5 f ( x) + f ( y), x ≥ y; x, y ∈ [0; 1] .
3

8
µ ¶
Hãy tính f .
23
BÀI 16. Tìm tất cả các hàm số f : R → R sao cho thỏa mãn
(
f (1) > 0
f ( x y − 1) + 2 f ( x) f ( y) = 3 x y − 1, ∀ x, y ∈ R.

BÀI 17. Tìm tất cả các hàm số f : R → R sao cho thỏa mãn

f ( x + y f ( x)) = x f ( y) + f ( x) , ∀ x, y ∈ R.

TOÁN CHUYÊN
BÀI 18. Tìm tất cả các hàm số f : R → R sao cho

f ( x y) + f ( x − y) + f ( x + y + 1) = x y + 2 x + 1, ∀ x, y ∈ R.

BÀI 19. Tìm tất cả các hàm số đơn điệu trên R thỏa mãn

f x3 + f ( y) = f 3 ( x) + y, ∀ x, y ∈ R.
¡ ¢

BÀI 20. Tìm tất cả các hàm số f : R → R sao cho

f ( f ( x) + f ( y)) = f x2 + 2 x2 f ( y) + f 2 ( y) , ∀ x, y ∈ R.
¡ ¢

BÀI 21 (Indian 2015). Tìm tất cả các hàm f : R → R sao cho

f x 2 + y f ( x ) = x f ( x + y) , ∀ x, y ∈ R.
¡ ¢

BÀI 22 (Zhautykov Olympiad 2015). Tìm tất cả các hàm f : R → R sao cho

f x3 + y3 + x y = x2 f ( x) + y2 f ( y) + f ( x y) , ∀ x, y ∈ R.
¡ ¢

BÀI 23 (Baltic Way 2014). Tìm tất cả các hàm f : R → R thỏa mãn ∀ x, y ∈ R ta có

f ( f ( y)) + f ( x − y) = f ( x f ( y) − x) .

BÀI 24 (Albania TST 2014). Tìm tất cả các hàm f : R → R thỏa mãn

f ( x) f ( y) = f ( x + y) + x y, ∀ x, y ∈ R.

BÀI 25 (European Girl’s Mathemmatical Olympiad 2014).


Tìm tất cả các hàm f : R → R thỏa mãn với mọi x, y ∈ R ta có

f y2 + 2 x f ( y) + f 2 ( x) = ( y + f ( x)) ( x + f ( y)) .
¡ ¢

BÀI 26 (Romanian Distric Olympiad 2014). Tìm tất cả các hàm f : N → N sao cho với mọi m, n ∈ N, ta có

f ( m + n) − 1 | f ( m) + f ( n) .

BÀI 27 (Korea 2014). Tìm tất cả các hàm f : R → R thỏa mãn với mọi x, y ∈ R, ta có

f ( x f ( x) + f ( x) f ( y) + y − 1) = f ( x f ( x) + x y) + y − 1.

ÓNguyễn Thành Nhân 77 Chuyên Hùng Vương-BD


Các Bài Giảng 2023-2024 Đại Số 10 Chuyên

BÀI 28 (Middle European Mathematical Olympiad 2014).


Tìm tất cả các hàm f : R → R thỏa mãn

x f ( y) + f ( x f ( y)) − x f ( f ( y) − f ( x y) = 2 x + f ( y) − f ( x + y).

BÀI 29 (Canada 2008). Tìm tất cả các hàm số f : Q → Q thỏa mãn đẳng thức:

f (2 f ( x) + f ( y)) = 2 x + y, ∀ x, y ∈ Q.

BÀI 30 (Mở rộng Canada 2008). Tìm tất cả các hàm số f : Q+ → Q+ thỏa mãn đẳng thức

f (2 f ( x) + f ( y)) = 2 x + y, x, y ∈ Q+ .

BÀI 31. Tìm tất cả các hàm số f : R → R thỏa mãn

f ( x f ( y) + x) = x y + f ( x), ∀ x, y ∈ R (12)

ĐS: f ( x) = x, f ( x) = − x trên R.
BÀI 32 (IMO Shortlist). Tìm tất cả các hàm số f : N → N thỏa mãn

f ( f ( n) + f ( m)) = m + n, ∀ m, n ∈ N (14)

ĐS: f (n) = n, ∀nN.


LATEX:Nguyễn Thành Nhân

BÀI 33. Tìm tất cả các hàm số f : R → R thỏa mãn

f ( x + f ( y)) = x + f ( y) + x f ( y), ∀ x, y ∈ R.

ĐS: f ( x) ≡ −1, ∀ x ∈ R.
BÀI 34 (IMO Shortlist 2002). Tìm tất cả các hàm số f : R → R thỏa mãn

f ( f ( x) + y) = 2 x + f ( f ( y) − x), ∀ x, y ∈ R.

ĐS: f ( x) = x − a với a là hằng số.


BÀI 35. Tìm tất cả các hàm f : R → R tăng thực sự thỏa mãn

f ( f ( x) + y) = f ( x + y) + 1, ∀ x, y ∈ R.

ĐS: f ( x) = x + 1, ∀ x ∈ R.
BÀI 36. Tìm tất cả các hàm f : R → R tăng thực sự thỏa mãn

f ( x f ( y)) = y f (2 x), ∀ x, y ∈ R.

ĐS: f ( x) = 2 x, ∀ x ∈ R.

ÓNguyễn Thành Nhân 78 Chuyên Hùng Vương-BD


CHƯƠNG 5 BẤT ĐẲNG THỨC

BÀI 1. BẤT ĐẲNG THỨC CAUCHY


Nguyễn Thành Nhân- Chuyên Hùng Vương- Bình Dương

A. CƠ SỞ LÝ THUYẾT
Bất đẳng thức Cauchy là tên thường gọi của bất đẳng thức giữa trung bình cộng (AM) và trung bình nhân
(GM). Bất đẳng thức cho ta đánh giá giữa trung bình cộng và trung bình nhân của n (n ≥ 2) số thực không
âm.

1. Bất đẳng thức Cauchy cho trường hợp hai biến


Định lí 1 (Cauchy). Cho hai số thực không âm a 1 và a 2 . Khi đó
a1 + a2 p
≥ a1 a2 (1)
2

Dấu đẳng thức xảy ra khi và chỉ khi a 1 = a 2 .


Chứng minh:

TOÁN CHUYÊN
p p
Biến đổi đưa về hằng đẳng thức ( a 1 − a 2 )2 ≥ 0.
Chú ý 11. Từ bất đẳng thức (1), ta thu được các kết quả sau đây mà thường sử dụng khi áp dụng bất đẳng
thức Cauchy.
p
1 a1 + a2 ≥ 2 a1 a2 . 2 a21 + a22 ≥ 2a 1 a 2 .

a21 + a22 ³ a + a ´2
1 2
3 a1 a2 ≤ . 4 a1 a2 ≤ .
2 2

Dấu đẳng thức xảy ra trong các bất đẳng thức trên khi và chỉ khi a 1 = a 2 .

2. Bất đẳng thức Cauchy cho trường hợp ba biến


Định lí 2 (Cauchy). Cho ba số thực không âm a 1 , a 2 , a 3 . Khi đó
a1 + a2 + a3 p
≥ 3 a1 a2 a3 (2)
3

Dấu đẳng thức xảy ra khi và chỉ khi a 1 = a 2 = a 3 .


Chứng minh:
Bất đẳng thức cần chứng minh tương đương
p
3
p
P = a1 + a2 + a3 + a1 a2 a3 ≥ 4 3 a1 a2 a3 .

Ta có
p p p p
q q
4
P ≥ 2 a1 a2 + 2 a1 3 a1 a2 a3 ≥ 4 a1 a2 a3 3 a1 a2 a3 = 4 3 a1 a2 a3 .

Ta có điều cần chứng minh.


Chú ý 12. Từ bất đẳng thức (2), ta thu được các kết quả sau đây mà thường sử dụng khi áp dụng bất đẳng
thức Cauchy.
p
1 a1 + a2 + a3 ≥ 3 3 a1 a2 a3 . 2 a31 + a32 + a33 ≥ 3a 1 a 2 a 3 .

a31 + a32 + a33 ³ a + a + a ´3


1 2 3
3 a1 a2 a3 ≤ . 4 a1 a2 a3 ≤ .
3 3

Dấu đẳng thức xảy ra trong các bất đẳng thức trên khi và chỉ khi a 1 = a 2 = a 3 .

79
Các Bài Giảng 2023-2024 Đại Số 10 Chuyên

3. Bất đẳng thức Cauchy cho trường hợp tổng quát


Định lí 3 (Cauchy). Cho n (n ≥ 2) số thực không âm a 1 , a 2 , . . . , a n . Khi đó
a1 + a2 + · · · + a n p
≥ n a1 a2 · · · a n (3)
n

Dấu đẳng thức xảy ra khi và chỉ khi a 1 = a 2 = · · · = a n .


Chứng minh:
Phép chứng minh cho trường hợp tổng quát được thực hiện bằng nhiều cách khác nhau. Có ít nhất 48 cách
chứng minh cho bất đẳng thức (3), trong đó có phương pháp quy nạp Cauchy, có lẽ chính vì thế mà bất đẳng
thức giữa trung bình cộng và trung bình nhân thường được gọi dưới tên là bất đẳng thức Cauchy. Học sinh
có thể tìm hiểu ở các cuốn sách chuyên khảo về bất đẳng thức. Do phạm vi bài học nên các phép chứng minh
đó không được trình bày ở đây.
Chú ý 13. Từ bất đẳng thức (3), ta thu được các kết quả sau đây mà thường sử dụng khi áp dụng bất đẳng
thức Cauchy.
p
1 a1 + a2 + · · · + a n ≥ n n a1 a2 · · · a n . 2 a 1n + a 2n + · · · + a nn ≥ na 1 a 2 · · · a n .

a 1n + a 2n + · · · + a nn ³ a + a + · · · + a ´n
1 2 n
3 a1 a2 · · · a n ≤ . 4 a1 a2 · · · a n ≤ .
n n
LATEX:Nguyễn Thành Nhân

Dấu đẳng thức xảy ra trong các bất đẳng thức trên khi và chỉ khi a 1 = a 2 = · · · = a n .

4. Các kết quả thu được từ bất đẳng thức Cauchy


Các kết quả sau đây nhiều khi được sử dụng như là một công đoạn nhỏ của một bài toán chứng minh bất
đẳng thức. Chúng ta thường gọi đó là các bất đẳng thức phụ.

Các kết quả


1 1
µ

1 Với a, b dương, ta có (a + b) + ≥ 4.
a b

1 1 1
µ ¶
2 Với a, b, c dương, ta có (a + b + c) + + ≥ 9.
a b c

1 1 1
µ ¶
3 Với a 1 , a 2 , . . ., a n dương, ta có (a 1 + a 2 + · · · + a n ) + +···+ ≥ n2 .
a1 a2 an

4 Với a, b, c không âm, ta có ab + bc + ca ≤ a2 + b2 + c2 .

5 Với a, b, c không âm, ta có 3(ab + bc + ca) ≤ (a + b + c)2 ≤ 3(a2 + b2 + c2 ).

a b c 3
6 Với a, b, c dương ta có + + ≥ (Bất đẳng thức Nesbit).
b+c c+a a+b 2

7 Với a, b, c là ba số thực dương, ta có

(a + b − c)( b + c − a)( c + a − b) ≤ abc.

5. Một số kỹ thuật căn bản trong áp dụng bất đẳng thức Cauchy

a) Kỹ thuật chọn điểm rơi


Đây là kỹ thuật quan trọng nhất mà học sinh cần trang bị. Kỹ thuật chọn điểm rơi là kỹ thuật nhìn dấu
đẳng thức xảy ra. Khi thấy được dấu đẳng thức xảy ra tại đâu, ta mới có sự điều chỉnh các hệ số một cách
phù hợp, tránh được những sai lầm.

Điểm rơi trong bất đẳng thức đối xứng

ÓNguyễn Thành Nhân 80 Chuyên Hùng Vương-BD


Các Bài Giảng 2023-2024 Đại Số 10 Chuyên

3
VÍ DỤ 1. Cho các số thực dương x, y, z thỏa a + y + z = . Tìm giá trị nhỏ nhất của biểu thức
2
1 1 1
P = x+ y+ z+ + + .
x y z

15 1
ĐS: min P = tại x = y = z = .
2 2

a2 b2 c2
VÍ DỤ 2. Cho a, b, c dương thỏa a + b + c = 2. Chứng minh rằng + + ≥ 1.
b+c c+a a+b

VÍ DỤ 3. Cho a ≥ 10, b ≥ 100, c ≥ 1000. Tìm giá trị nhỏ nhất của
1 1 1
P = a+ +b+ +c+ .
a b c

Điểm rơi trong bất đẳng thức không đối xứng

VÍ DỤ 4. Cho các số thực dương a, b, c thỏa mãn a + 2b + 3 c ≥ 20. Chứng minh rằng
3 9 4
a+b+c+ + + ≥ 13.

TOÁN CHUYÊN
a 2b c

VÍ DỤ 5. Cho a, b, c dương. Chứng minh rằng

2 c3 1 1 1
µ ¶
30a + 3 b2 + + 36 + + ≥ 84.
9 ab bc ca

Điểm rơi giả định


Rất nhiều bài chứng minh bất đẳng thức, ta chưa thể nhìn ra được điểm rơi. Lúc đó, ta sử dụng điểm rơi giả
định để sử dụng trong lập luận.

VÍ DỤ 6. Cho các số thực x, y, z thỏa mãn x y + yz + zx = 1. Tìm giá trị nhỏ nhất của biểu thức

P = x 2 + y2 + 2 z 2 .

b) Kỹ thuật ghép đối xứng


Kỹ thuật ghép đối xứng cũng là một kỹ thuật quan trọng không thể thiếu khi sử dụng bất đẳng thức Cauchy
để chứng minh. Ta có thể tóm tắt ý tưởng thông qua một số dạng hay gặp như sau
1 Dạng 1: Chứng minh X + Y + Z ≥ A + B + C . Ta đi chứng minh các bất đẳng thức

X + Y ≥ 2 A ; Y + Z ≥ 2B; Z + X ≥ 2C.

Cộng vế theo vế thì có ngay bất đẳng thức cần chứng minh. Tổng quát hơn nữa, ta đi chứng minh

mX + nY + pZ ≥ ( m + n + p) A ; mY + nZ + pX ≥ ( m + n + p)B; mZ + nX + pY ≥ ( m + n + p)C.

Cộng vế theo vế ta có điều cần chứng minh.


2 Chứng minh X Y Z ≥ ABC với X , Y , Z ≥ 0. Ta thường đi chứng minh

X Y ≥ A 2 ; Y Z ≥ B2 ; Y Z ≥ C 2 .

Nhân vế theo vế ta được điều cần chứng minh.

ÓNguyễn Thành Nhân 81 Chuyên Hùng Vương-BD


Các Bài Giảng 2023-2024 Đại Số 10 Chuyên

VÍ DỤ 7. Cho a, b, c là ba số thực dương. Chứng minh rằng


ab bc ca
+ + ≥ a + b + c.
c a b

VÍ DỤ 8. Cho ba số không âm a, b, c. Chứng minh rằng


p
4
p
4
p
4
a+b+c ≥ ab3 + bc3 + ca3 .

VÍ DỤ 9. Cho các số thực dương a, b, c. Chứng minh rằng

abc ≥ ( b + c − a)( c + a − b)(a + b − c).

1 1 1 1
VÍ DỤ 10. Cho ba số thực dương x, y, z thỏa mãn + + ≥ 2. Chứng minh rằng x yz ≤ .
1+ x 1+ y 1+ z 8
LATEX:Nguyễn Thành Nhân

c) Kỹ thuật Cauchy ngược dấu


Kỹ thuật này được sáng tạo bởi tác giả Phạm Kim Hùng- HCV IMO 2005. Kỹ thuật này áp dụng khi ta áp
dụng trực tiếp bất đẳng thức Cauchy thì dấu bị ngược.

VÍ DỤ 11. Cho các số thực dương a, b, c thỏa mãn a + b + c = 3. Chứng minh rằng
a b c 3
2
+ 2
+ 2
≥ .
1+b 1+ c 1+a 2

VÍ DỤ 12. Cho bốn số thực dương a, b, c, d có tổng bằng 4. Chứng minh rằng
a b c d
+ + + ≥ 2.
1 + b 2 1 + c 2 1 + d 2 1 + a2

VÍ DỤ 13. Cho bốn số thực dương a, b, c, d có tổng bằng 4. Chứng minh rằng
a b c d
+ + + ≥ 2.
1 + b 2 c 1 + c 2 d 1 + d 2 a 1 + a2 b

VÍ DỤ 14. Chứng minh rằng với bốn số thực dương a, b, c, d ta luôn có

a3 b3 c3 d3 a+b+c+d
2 2
+ 2 2
+ 2 2
+ 2 2
≥ .
a +b b +c c +d d +a 2

VÍ DỤ 15. Cho a, b, c ≥ 0 và a + b + c = 3. Chứng minh rằng

a2 b2 c2
2
+ 2
+ ≥ 1.
a + 2b b + 2c c + 2 a2

VÍ DỤ 16. Cho a, b, c ≥ 0 và a + b + c = 3. Chứng minh rằng

a2 b2 c2
+ + ≥ 1.
a + 2 b 3 b + 2 c 3 c + 2 a3

ÓNguyễn Thành Nhân 82 Chuyên Hùng Vương-BD


Các Bài Giảng 2023-2024 Đại Số 10 Chuyên

VÍ DỤ 17. Chứng minh rằng với mọi số dương a, b, c có tổng bằng 3 thì
a+1 b+1 c+1
2
+ 2 + 2 ≥ 3.
b +1 c +1 a +1

VÍ DỤ 18. Chứng minh rằng với mọi số dương a, b, c, d có tổng bằng 4 thì
a+1 b+1 c+1 d +1
+ + + ≥ 4.
b 2 + 1 c 2 + 1 d 2 + 1 a2 + 1

VÍ DỤ 19. Chứng minh rằng với mọi số dương a, b, c, d có tổng bằng 4 thì
1 1 1 1
+ + + ≥ 2.
a2 + 1 b 2 + 1 c 2 + 1 d 2 + 1

BÀI 2. BẤT ĐẲNG THỨC CAUCHY-BÀI TẬP


Nguyễn Thành Nhân- Chuyên Hùng Vương- Bình Dương

A. BÀI TẬP

TOÁN CHUYÊN
Sau khi được học một số kỹ thuật căn bản để xử lý bất đẳng thức Cauchy, trong bài học này, các bài tập sẽ
được đặt một cách tùy ý. Việc học sinh quyết định lựa chọn phương pháp nào phù hợp để giải là rất quan
trọng. Đó là một kỹ năng cần thiết phải rèn luyện.
BÀI 1 (Nghệ An-Vòng 1 (2015-2016)). Với các số thực dương có tổng bằng 3. Chứng minh rằng
a+1 b+1 c+1
2
+ 2 + 2 ≥ 3.
b +1 c +1 a +1

BÀI 2 (Bình Định TST 2016). Cho a, b, c là độ dài ba cạnh của một tam giác. Tìm giá trị nhỏ nhất của biểu
thức
4a 9b 16 c
A= + + .
b+c−a c+a−b a+b−c
BÀI 3. Cho a, b, c là các số dương thỏa mãn a2 + b2 + c2 = 1. Tìm giá trị lớn nhất của biểu thức A = (1 + 2a)(1 +
2 bc).
( x + y)2 ( x + y)2
BÀI 4. Cho x, y là hai số thực dương. Tìm giá trị nhỏ nhất của biểu thức A = + .
x 2 + y2 xy
BÀI 5. 1 Cho a, b là hai số dương. Chứng minh rằng:
1 1 4
(i) + ≥ .
a b a+b
p 1
(ii) a2 − ab + 3b2 + 1 ≥ (a + 5b + 2).
4
1 1 1
2 Cho các số thực dương a, b, c thỏa mãn + + ≤ 3. Tìm giá trị lớn nhất của biểu thức:
a b c
1 1 1
P=p +p +p .
a2 − ab + 3 b2 + 1 b2 − bc + 3 c2 + 1 c2 − ca + 3a2 + 1

BÀI 6. Cho a, b, c là các số thực dương. Tìm giá trị nhỏ nhất của biểu thức

3(ab + bc + ca) (a + b + c)3


P= + .
a2 + b 2 + c 2 abc
1 1 1
BÀI 7. Cho các số dương x, y, z thoả mãn + + ≥ 2020. Tìm giá trị nhỏ nhất của biểu thức
x+ y y+ z z+ x
p p p
y2 + 2 x2 z 2 + 2 y2 x2 + 2 z 2
P= + + .
xy yz zx

ÓNguyễn Thành Nhân 83 Chuyên Hùng Vương-BD


Các Bài Giảng 2023-2024 Đại Số 10 Chuyên

BÀI 8. Cho a, b, c là các số thực dương thỏa mãn abc = 8. Chứng minh

a b c 1 2
+ + ≤ (a + b2 + c2 ).
ca + 4 ab + 4 bc + 4 16

BÀI 9. Với a, b, c là những số thực dương thỏa mãn a + b + c = 3. Chứng minh rằng

a(a + bc)2 b( b + ca)2 c( c + ab)2


+ + ≥ 4.
b(ab + 2 c ) c( bc + 2a ) a( ca + 2 b2 )
2 2

BÀI 10. Cho ba số dương x, y, z thỏa mãn x y + yz + zx = 5. Chứng minh


p
x y 3z 2 6
p +p +p ≤ .
x2 + 5 y2 + 5 6( z2 + 5) 3

Đẳng thức xảy ra khi và chỉ khi nào?


BÀI 11. Cho các phương trình x2 + ax + 3 = 0 và x2 + bx + 5 = 0, với a, b là tham số.

1 Chứng minh nếu ab ≥ 16 thì trong hai phương trình trên có ít nhất một phương trình có nghiệm.

2 Giả sử hai phương trình trên có nghiệm chung x0 . Tìm a, b sao cho |a| + | b| có giá trị nhỏ nhất.
LATEX:Nguyễn Thành Nhân

BÀI 12. Cho x, y là các số thực dương và x5 − y3 ≥ 2 x. Chứng minh rằng x3 ≥ 2 y.


1 1 1
BÀI 13. Cho các số a, b, c > 0 thỏa mãn a + b + c + ab + bc + ca = 6abc. Chứng minh 2
+ 2 + 2 Ê 3.
a b c
BÀI 14. Cho a, b, c > 0 thỏa mãn a + b + c = 1, chứng minh rằng

a b c
p + p + p ≤ 1.
a + a + bc b + b + ca c + c + ab
Đẳng thức xảy ra khi nào?
p p p
BÀI p
15. Xét các sốpdương a, b, cpthỏa mãn a+1+ b+1+ c + 1 = 6. Tìm giá trị nhỏ nhất của biểu thức
P= a2 + ab + b2 + b2 + bc + c2 + c2 + ca + a2 .
BÀI 16. Cho các số thực không âm a, b, c thỏa mãn điều kiện a + b + c = 1.
1
Chứng minh a3 + b3 + c3 ≤ + a4 + b 4 + c 4 .
8
BÀI r
17. Cho ar
, b, c là barsố thực dương thỏa mãn điều kiện a + b + c = 3. Tìm giá trị nhỏ nhất của biểu thức
a+b b+c c+a
P= + + .
c + ab a + bc b + ca
BÀI 18. Cho các số thực x, y, z thỏa mãn điều kiện x y + yz + zx + 4 x2 + y2 + z2 = 15. Chứng minh rằng x2 + y2 +
¡ ¢

z2 ≥ 3.
BÀI 19. Cho x, y, z > 0. Chứng minh bất đẳng thức
p s p
xy 1 2 yz
p +p p + p ≥ 2 (1).
1 + yz x y + yz 1+ xy

BÀI 20. Cho x > 0, y > 0 và x y = 4. Tìm giá trị nhỏ nhất của biểu thức

x3 y3
Q= + .
4( y + 2) 4( x + 2)

BÀI 21. Cho các số dương a, b, c. Chứng minh rằng

8(a2 + b2 + c2 ) 27(a + b)( b + c)( c + a)


+ ≥ 16.
ab + bc + ca (a + b + c)3

BÀI 3. BẤT ĐẲNG THỨC CAUCHY-SCHWARZ


Nguyễn Thành Nhân- Chuyên Hùng Vương- Bình Dương

ÓNguyễn Thành Nhân 84 Chuyên Hùng Vương-BD


Các Bài Giảng 2023-2024 Đại Số 10 Chuyên

A. CƠ SỞ LÝ THUYẾT
Bất đẳng thức Cauchy-Schwarz còn biết đến dưới tên gọi là bất đẳng thức Schwarz, bất đẳng thức Cauchy,
hoặc cái tên khá dài là bất đẳng thức Cauchy-Bunyakovski-Schwarz, đặt theo tên gọi của Cauchy, Bun-
yakovski, Schwarz là một bất đẳng thức rất thông dụng tronbg toán sơ cấp. Ở Việt Nam, bất đẳng thức này
thường được gọi dưới tên là Bunyakovski hoặc gọi theo tên dài dưới thứ tự đảo ngược đó là Bunyakovski-
Cauchy-Schawrz và hay ký hiệu là BCS. Trong các bài giảng, chúng ta thống nhất với tên gọi là Cauchy-
Schwarz. Một đẳng thức thường được áp dụng trong các chứng minh sử dụng bất đẳng thức Cauchy-Schwarz
đó là đẳng thức Lagrange.

1. Đẳng thức Lagrange


Với các số thực a 1 , a 2 , . . . , a n và b1 , b2 , . . . , b n , ta có
à ! à ! à !2
n n n ¢2
a2i b2i
X X X X ¡
· = ai bi + ai b j − a j bi . (1)
i =1 i =1 i =1 1≤ i < j ≤ n

Vài trường hợp đơn giản


a21 + a22 b21 + b22 = (a 1 b 1 + a 2 b 2 )2 + (a 1 b 2 − a 2 b 1 )2 .
¡ ¢¡ ¢
1

2 a21 + a22 + a23 b21 + b22 + b23 = (a 1 b 1 + a 2 b 2 + a 3 b 3 )2 + (a 1 b 2 − a 2 b 1 )2 + (a 1 b 3 − a 3 b 1 )2 + (a 2 b 3 − a 3 b 2 )2 .


¡ ¢¡ ¢

2. Bất đẳng thức Cauchy-Schwarz


Định lí 1 (Cauchy-Schwarz). Cho 2n (n ≥ 2) số thực tùy ý a 1 , a 2 , . . . , a n và b1 , b2 , . . . , b n . Ta có

TOÁN CHUYÊN
(a 1 b 1 + a 2 b 2 + · · · + a n b n )2 ≤ a21 + a22 + · · · + a2n · b21 + b22 + · · · + b2n .
¡ ¢ ¡ ¢
(2)
Hay viết gọn
à !2 à ! à !
n n n
a2i b2i
X X X
ai bi ≤ · (3)
i =1 i =1 i =1

Dấu đẳng thức xảy ra khi và chỉ khi hai bộ số (a 1 , a 2 , . . . , a n ) và (b1 , b2 , . . . , b n ) tỉ lệ. Có nghĩa là tồn tại t ∈ R sao
cho a i = tb i , ∀ i = 1; n.
Chứng minh:
Chúng ta có thể chứng minh bất đẳng thức Cauchy-Schwarz bằng cách xét dấu tam thức bậc hai, đẳng
thức Lagrange, Cauchy, quy nạp. Sau đây chúng ta dùng đẳng thức Lagange để chứng minh. Ta có
à ! à ! à !2
n n n ¢2
a2i b2i
X X X X ¡
· − ai bi = ai b j − a j bi ≥ 0.
i =1 i =1 i =1 1≤ i < j ≤ n

Dấu đẳng thức xảy ra khi và chỉ khi


a i b j − a j b i = 0 ⇔ a i = tb i , t ∈ R, i = 1, n.

Bất đẳng thức được chứng minh.

3. Bất đẳng thức Cauchy-Schwarz dạng Engle


Chúng ta xét một hệ quả của bất đẳng thức Cauchy-Schwarz được biết đến dưới tên gọi bất đẳng thức
Cauchy-Schwarz dạng Engle.

Bất đẳng thức Cauchy-Schwarz dạng Engle


Định lí 2. Cho n số thực tùy ý a 1 , a 2 , . . . , a n và n số thực dương b1 , b2 , . . . , b n . Khi đó
a21 a22 a2n (a 1 + a 2 + · · · + a n )2
+ +···+ ≥ . (4)
b1 b2 bn b1 + b2 + · · · + b n
Dấu đẳng thức xảy ra khi và chỉ khi
a1 a2 an
= = ··· = .
b1 b2 bn
Chú ý 14. Bất đẳng thức Cauchy-Schwarz dạng Engle được phát biểu khi mẫu là các số thực dương. Chúng
ta cũng thường gọi với tên gọi là bất đẳng thức cộng mẫu.

ÓNguyễn Thành Nhân 85 Chuyên Hùng Vương-BD


Các Bài Giảng 2023-2024 Đại Số 10 Chuyên

B. BÀI TẬP
Kỹ thuật thông thường
BÀI 1. Cho a, b, c là các số thực dương và a + b + c ≥ 2020. Chứng minh rằng

a2 b2 c2
+ + ≥ 1010.
b+c c+a a+b

BÀI 2 (Czech-Slovak 1999). Cho a, b, c là các số thực dương. Chứng minh rằng

a b c
+ + ≥ 1.
b + 2 c c + 2a a + 2 b

BÀI 3 (Bình Dương-Vòng 1-2019). Cho a, b, c là các số thực thỏa mãn ab + bc + ca = 1. Tìm giá trị lớn nhất của
biểu thức
1 1 1
T= + + .
a2 + 1 b 2 + 1 c 2 + 1
BÀI 4. Cho a, b, c dương và a + b + c = 3. chứng minh rằng

a b c
+ + ≥ 1.
a + 2 bc b + 2 ca c + 2ab
LATEX:Nguyễn Thành Nhân

BÀI 5. Cho a, b, c dương. Chứng minh rằng

a3 b3 c3 a2 + b 2 + c 2
+ + ≥ .
a + 2 b b + 2 c c + 2a 3

1 1 1
BÀI 6 (Iran 1999). Cho x, y, z > 1 và + + = 2. Chứng minh rằng
x y z
p p p p
x+ y+ z ≥ x−1+ y−1+ z − 1.

BÀI 7 (Võ Quốc Bá Cẩn). Cho bốn số thực dương a, b, c, d thỏa mãn

1 1 1 1
a+b+c+d = + + + .
a b c d

Chứng minh rằng


s s s s
a2 + 1 b2 + 1 c2 + 1 d2 + 1
a+b+c+d ≥ + + + .
2 2 2 2

Sử dụng đẳng thức Lagrange


Đẳng thức Lagrange sử dụng hiệu quả khi nào? Ta thấy rằng bất đẳng thức Cauchy-Schwarz chỉ áp dụng
được khi một vế nào đó có dạng tích của hai nhân tử. Vậy khi gặp các bài toán mà có từ tích ba nhân tử trở
lên thì chưa thể áp dụng bất đẳng thức Cauchy-Schwarz để chứng minh. Ta quan sát đẳng thức Lagrange
đơn giản sau
a21 + a22 b21 + b22 = (a 1 b 1 + a 2 b 2 )2 + (a 1 b 2 − a 2 b 1 )2 .
¡ ¢¡ ¢

Ta thấy rằng bên trái là tích hai nhân tử, sang phải chỉ còn một nhân tử. Vậy đẳng thức Lagrange cho phép
ta làm giảm các nhân tử trong tích. Ta cứ giảm giần cho tới khi còn lại hai nhân tử để có thể áp dụng được
bất đẳng thức Cauchy-Schwarz. Thật là một ứng dụng rất tốt.
BÀI 8 (Võ Quốc Bá Cẩn). Cho các số thực a, b, c tùy ý. Chứng minh rằng
· ¸2
2(a2 + b2 )( b2 + c2 )( c2 + a2 ) ≥
X
ab(a + b) − 2abc .

BÀI 9 (Gabriel Dospinescu). Chứng minh rằng với mọi số thực a, b, c ta có

(a2 + ab + b2 ) ≥ 3(a2 b + b2 c + c2 a)(ab2 + bc2 + ca2 ).


Y

ÓNguyễn Thành Nhân 86 Chuyên Hùng Vương-BD


Các Bài Giảng 2023-2024 Đại Số 10 Chuyên

1. Qua các đề thi


BÀI 10 (Bình Định TST 2016). Cho a, b, c là độ dài ba cạnh của một tam giác. Tìm giá trị nhỏ nhất của biểu
thức
4a 9b 16 c
A= + + .
b+c−a c+a−b a+b−c

BÀI 11 (Nghệ An-Vòng 1 (2015-2016)). Với các số thực dương có tổng bằng 3. Chứng minh rằng

a+1 b+1 c+1


2
+ 2 + 2 ≥ 3.
b +1 c +1 a +1

BÀI 12 (Phú Thọ TST 2016). Cho a, b, c là ba số thực phân biệt đôi một. Chứng minh rằng

1 1 1 9
· ¸
2 2 2
(a + b + c ) + + ≥
(a − b)2 ( b − c)2 ( c − a)2 2

BÀI 13 (Vũng Tàu-2016). Cho a, b, c là ba số thực dương thỏa mãn 4 c + 2 b ≥ a(b2 + c2 ). Tìm giá trị nhỏ nhất
của biểu thức
3 4 5
S= + + .
b+c−a a+c−b a+b−c

BÀI 14 (Trà Vinh-2016). Với a, b, c là các số thực dương. Chứng minh rằng

7 5 4 4 1 3
µ ¶
+ + ≥4 + + .
a b c a+b b+c c+a

TOÁN CHUYÊN
p
3 3
BÀI 15 (Hà Nội TST). Với x, y, z là ba số thực dương thỏa mãn 0 < x, y, z < và x y + yz + zx = . Tìm giá trị
2 4
nhỏ nhất của biểu thức
4x 4y 4z
P= + + .
3 − 4 x2 3 − 4 y2 3 − 4 z2

BÀI 16 (Hải Phòng TST). Với x, y, z là ba số thực dương có tổng bằng 3. Chứng minh rằng
x y z
p + p + p ≤ 1.
x+ 3 x + yz y+ 3 y + zx z+ 3z + x y

BÀI 17 (Quảng Ngãi TST). Với x, y, z là ba số thực dương có tổng bằng 12. Tìm giá trị lớn nhất của biểu thức

2 x + y + z − 15 x + 2 y + z − 15 x + y + 2 z − 15
M= + + .
x y z

BÀI 18 (Nam Định TST). Cho a, b, c là ba số thực không âm và thỏa mãn không có hai số nào đồng thời bằng
0 và a + b + c = 1. Tìm giá trị nhỏ nhất của biểu thức

2 2
P= + + ( c + 2)(3 + a + b)
(a + b)( b + c) ( c + a)(a + b)

BÀI 4. BẤT ĐẲNG THỨC HOLDER VÀ BẤT ĐẲNG THỨC ABEL


Nguyễn Thành Nhân- Chuyên Hùng Vương- Bình Dương

A. BẤT ĐẲNG THỨC HOLDER


Bất đẳng thức Holder là một bất đẳng thức rất mạnh và có nhiều ứng dụng. Việc làm quen và thuần thục
bất đẳng thức Holder là rất cần thiết đối với học sinh chuyên toán. Vì tuy rằng có một phát biểu hơi khó
nhìn, nhưng bất đẳng thức Holder hoàn toàn dễ hiểu và ứng dụng hiệu quả với nhiều bài toán. Thêm nữa
các kỹ năng xử lý bất đẳng thức Holder gắn liền với bất đẳng thức AM-GM. Để dễ dàng tiếp nhận bất đẳng
thức Holder, chúng ta đi từ trường hợp đơn giản đến tổng quát. Quy tắc viết trong bất đẳng thức Holder là
cộng theo hàng-nhân theo cột.

ÓNguyễn Thành Nhân 87 Chuyên Hùng Vương-BD


Các Bài Giảng 2023-2024 Đại Số 10 Chuyên

1. Bất đẳng thức Holder cho hai cặp số dương


Định lí 1 (Holder). Cho hai bộ số dương (a 11 ; a 12 ) và (a 21 ; a 22 ). Khi đó
¡p p ¢2
(a 11 + a 12 ) · (a 21 + a 22 ) ≥ a 11 a 21 + a 12 a 22 (1)
Dấu đẳng thức xảy ra khi và chỉ khi hai bộ (a 11 ; a 12 ) và (a 21 ; a 22 ) tỉ lệ.
Chứng minh:
Áp dụng bất đẳng thức Cauchy ta có
a 11 a 12 a 11 a 12
r
+ ≥2 · ;
a 11 + a 12 a 11 + a 12 a 11 + a 12 a 11 + a 12
a 21 a 22 a 21 a 22
r
+ ≥2 · .
a 21 + a 22 a 22 + a 22 a 21 + a 22 a 22 + a 22
Cộng vế theo vế hai bất đẳng thức trên ta được điều cần chứng minh.

2. Bất đẳng thức Holder cho hai bộ ba số dương


Định lí 2 (Holder). Cho hai bộ số dương (a 11 ; a 12 ; a 13 ); (a 21 ; a 22 ; a 23 ) và (a 31 ; a 32 ; a 33 ). Khi đó
¡p
3
p
3
p
3
¢3
(a 11 + a 12 + a 13 ) · (a 21 + a 22 + a 23 ) · (a 31 + a 32 + a 33 ) ≥ a 11 a 21 a 31 + a 12 a 22 a 32 + a 13 a 23 a 33 (2)
LATEX:Nguyễn Thành Nhân

Dấu đẳng thức xảy ra khi và chỉ khi ba bộ (a 11 ; a 12 ; a 13 ); (a 21 ; a 22 ; a 23 ) và (a 31 ; a 32 ; a 33 ) tỉ lệ.


Chứng minh:
Tương tự như bộ hai số.

3. Bất đẳng thức Holder cho m bộ, mỗi bộ n số dương


Định lí 3 (Holder). Cho m bộ, mỗi bộ n số dương (a 11 ; a 12 ; . . . ; a 1n ); (a 21 ; a 22 ; . . . ; a 2n ) ; . . . ; (a m1 ; a m2 ; . . . ; a mn ). Khi
đó Ã ! Ã s !m
m X
Y n n
X m
Y
m
a i, j ≥ ai j (3)
i =1 j =1 j =1 i =1

Dấu đẳng thức xảy ra khi và chỉ khi m bộ tỉ lệ.

4. Một số kết quả


Hệ quả 1. Bất đẳng thức Cauchy-Schwarz là hệ quả của bất đẳng thức Holder.
Hệ quả 2. Với (a; b; c), ( x; y; z) và (m; n; p) là các số thực dương, khi đó
(a3 + b3 + c3 )( x3 + y3 + z3 )( m3 + n3 + p3 ) ≥ (axm + b yn + cz p)3 (4)
Chứng minh
Áp dụng bất đẳng thức AM-GM, ta có
a3 x3 m3 3axm
+ + ≥ ,
a3 + b3 + c3 x3 + y3 + z3 m3 + n3 + p3 (a3 + b3 + c3 )( x3 + y3 + z3 )( m3 + n3 + p3 )
tương tự cho các bộ khác. Sau đó cộng vế theo vế thì ta được điều cần chứng minh.
Chú ý rằng phép chứng minh này áp dụng để chứng minh bất đẳng thức Holder tổng quát.
Hệ quả 3. Cho n số dương a 1 , a 2 , . . ., a n ta có
¡ p ¢n
(1 + a 1 )(1 + a 2 ) · · · (1 + a n ) ≥ 1 + n a 1 a 2 · · · a n (5)
Chứng minh
Áp dụng bất đẳng thức AM-GM, ta có
1 1 1 n
+ +···+ ≥ p ,
1 + a1 1 + a2 1 + an n
(1 + a 1 )(1 + a 2 ) · · · (1 + a n )
và p
a1 a2 an n n a1 a2 · · · a n
+ +···+ ≥ p .
1 + a1 1 + a2 1 + an n
(1 + a 1 )(1 + a 2 ) · · · (1 + a n )
Cộng vế theo vế thì ta được điều phải chứng minh.
Sau đây ta xét một phiên bản khác của bất đẳng thức Holder mà trường hợp riêng của nó là bất đẳng thức
cộng mẫu.

ÓNguyễn Thành Nhân 88 Chuyên Hùng Vương-BD


Các Bài Giảng 2023-2024 Đại Số 10 Chuyên

Hệ quả 4. Với a 1 , x2 , . . . , xn ; y1 , y2 , . . . , yn là các số thực dương và p là một số nguyên dương. Khi đó


p+1 p+1 p+1
x1 x2 xn ( x1 + x2 + · · · + xn ) p+1
p + p +···+ p ≥ .
y1 y2 yn ( y1 + y2 + · · · + yn ) p

B. BÀI TẬP
BÀI 1 (Dự tuyển 10-2023-Bình Dương). Cho các số thực x, y, z > 0. Chứng minh rằng

x3 y3 z3 x2 y2 z2
+ + ≥ + + .
y3 z3 x3 y2 z2 x2

BÀI 2. Với mọi số thực dương a, b, c, chứng minh rằng ta có

a b c
p +p +p ≥ 1.
a2 + 8 bc b2 + 8 ca c2 + 8ab

BÀI 3. Chứng minh rằng nếu a, b, c là các số thực dương và có tích bằng 1 thì ta có các bất đẳng thức sau

a b c
p +p +p ≥ 1,
7+b+ c 7+ c+a 7+a+b


a b c
p +p +p ≥ 1.
7 + b2 + c2 7 + c 2 + a2 7 + a2 + b 2

TOÁN CHUYÊN
BÀI 4. Chứng minh rằng với mọi a, b, c dương ta có

a3 b 3 c 3 a2 b 2 c 2
+ + ≥ + + .
b 2 c 2 a2 b c a

BÀI 5. Chứng minh rằng với mọi a, b, c dương và k ∈ N∗ ta có

a k+1 b k+1 c k+1 ak bk ck


+ + ≥ + + .
bk ck ak b k−1 c k−1 a k−1

BÀI 6 (Singapore MO). Chứng minh rằng với mọi a, b, c dương, ta có

3(a3 + b3 + c3 )2 ≥ (a2 + b2 + c2 )3 .

BÀI 7 (Tạp chí Crux). Với các số thực dương a 1 , a 2 , . . ., a n có tổng bằng 1. Hãy tìm giá trị nhỏ nhất của biểu
thức
a1 a2 an
p +p +···+ p .
1 − a1 1 − a2 1 + an
p
BÀI 8 (IMO Shortlist 2011). Gọi a, b, c là các số thực dương sao cho min (a + b, b + c, c + a) > 2 và a2 + b2 + c2 = 3.
Chứng minh rằng
a b c 3
+ + ≥ .
( b + c − a)2 ( c + a − b)2 (a + b − c)2 (abc)2

BÀI 9. Cho a, b, c là các số thực không âm có tổng bằng 1. Chứng minh rằng

a b c
p
3
+p
3
+p
3
≥ 1.
a + 2b b + 2c c + 2a

BÀI TẬP TỔNG HỢP

C. KHAI TRIỂN ABEL VÀ ỨNG DỤNG


Ở bài trước chúng ta đã biết đẳng thức Lagrange và ứng dụng của nó vào trong các bài toán áp dụng bất
đẳng thức Cauchy-Schwarz để chứng minh bất đẳng thức. Trong bài này, chúng ta được làm quen với khai
triển Abel hay đẳng thức Abel.

ÓNguyễn Thành Nhân 89 Chuyên Hùng Vương-BD


Các Bài Giảng 2023-2024 Đại Số 10 Chuyên

1. Khai triển Abel


k
X
Định lí 4 (Abel). Cho hai bộ số a 1 , a 2 , . . . , a n và b1 , b2 , . . . , b n . Đặt c k = b i ( k = 1; n). Khi đó
i =1

a 1 b 1 + a 2 b 2 + · · · + a n b n = (a 1 − a 2 ) c 1 + (a 2 − a 3 ) c 2 + · · · + (a n−1 − a n ) c n−1 + a n c n . (1)

Ta xét vài trường hợp đặc biệt


• a 1 b 1 + a 2 b 2 = ( a 1 − a 2 ) b 1 + a 2 ( b 1 + b 2 ).

• a 1 b 1 + a 2 b 2 + a 3 ab 3 = (a 1 − a 2 ) b 1 + (a 2 − a 3 )( b 1 + b 2 ) + a 3 ( b 1 + b 2 + b 3 ).

2. Bất đẳng thức Abel


Bất đẳng thức sau đây được chứng minh dựa trên việc sử dụng khai triển Abel.
Định lí 5 (Abel). Cho hai dãy a 1 , a 2 , . . . , a n và b1 , b2 , . . . , b n trong đó dãy thứ nhất là một dãy giảm (a 1 ≥ a 2 ≥
k
X
. . . ≥ a n ). Nếu đặt c k = b i ( k = 1; n) và M = max c k , m = min c k . Khi đó
i =1

n
X
ma 1 ≤ a i b i ≤ Ma 1 . (2)
i =1
LATEX:Nguyễn Thành Nhân

Bất đẳng thức trên được gọi là bất đẳng thức Abel.
Chứng minh
Áp dụng khai triển Abel, ta có
n
X
a i b1 = (a 1 − a 2 ) c 1 + (a 2 − a 3 ) c 2 + · · · + (a n−1 − a n ) c n−1 + a n c n
i =1
≤ M (a 1 − a 2 ) + M (a 2 − a 3 ) + · · · + M (a n−1 − a n ) + Ma n = Ma 1 .

Lập luận tương tự ta cũng có


n
X
a i b i ≥ ma 1 .
i =1

Bất đẳng thức được chứng minh.

3. Ứng dụng khai triển Abel


BÀI 10. Cho hai dãy số thực a 1 , a 2 , . . . , a n và b1 , b2 , . . . , b n thỏa mãn các điều kiện
a , a , . . . , an ≥ 0

 1 2


b1 ≥ b2 ≥ . . . ≥ b n ≥ 0





a ≥ b

1 1


 a1 a2 ≥ b1 b2

...






a1 a2 . . . a n ≥ b1 b2 . . . b n .

Chứng minh bất đẳng thức a 1 + a 2 + · · · + a n ≥ b1 + b2 + · · · + b n .


BÀI 11. Cho các số thực dương a, b, c sao cho 1 ≤ b ≤ a ≤ 3, ab ≤ 6, ab ≤ 6 c. Chứng minh rằng

a + b − c ≤ 4.

BÀI 12. Cho a, b, c thỏa mãn 0 < a ≤ b ≤ c ≤ 3, bc ≤ 6, abc ≤ 6. Chứng minh rằng

a + b + c ≤ 6.

BÀI 13. Cho a, b, c là các số thực thỏa mãn 0 < a ≤ b ≤ c ≤ 3, 2b + c ≤ 2, 3a + 2 b + c ≤ 3. Tìm giá trị nhỏ nhất của
biểu thức
P = a2 + b 2 + c 2 .
BÀI 14. Cho hai dãy số dương a 1 , a 2 , . . ., a n và b1 , b2 , . . ., b n thỏa mãn b1 ≤ b2 ≤ . . . ≤ b n và với mọi k = 1; n thì
a21 + a22 + · · · + a2k ≤ b21 + b22 + · · · + b2k . Chứng minh rằng

a1 + a2 + · · · + a n ≤ b1 + b2 + · · · + b n .

ÓNguyễn Thành Nhân 90 Chuyên Hùng Vương-BD


Các Bài Giảng 2023-2024 Đại Số 10 Chuyên

c a b a b c
BÀI 15. Giả sử 0 ≤ x ≤ y ≤ z và a, b, c ≥ 0 thỏa mãn ≤ 1, + ≤ 2, + + ≤ 3. Chứng minh rằng
z x y x y z
p p p p p p
a + b + c ≤ x + y + z.

BÀI 16 (USA 94). Cho các số thực dương x1 , x2 , . . . , xn thỏa mãn


p
x1 + x2 + · · · + x k ≥ k, ∀ k ≤ n.

Chứng minh rằng


1 1 1 1
µ ¶
x12 + x22 + · · · + x2n ≥ 1+ + +···++ .
4 2 3 n

BÀI 17 (Nguyễn Thành Nhân). Cho hai dãy hữu hạn các số dương (a k )nk=1 và (b k )nk=1 trong đó ( b k )nk=1 là dãy
giảm và b k ≤ 1, ∀k = 1; n. Biết rằng a 1 ≥ b1 , a 1 a 2 ≥ b2 , . . . , a 1 a 2 . . . a n ≥ b n . Tìm giá trị nhỏ nhất của biểu thức

a21 + a22 + · · · + a2n


T= .
( b 1 + b − 2 + · · · + b n )2

BÀI 5. BẤT ĐẲNG THỨC HOÁN VỊ- BẤT ĐẲNG THỨC


CHEBYSHEV
Nguyễn Thành Nhân- Chuyên Hùng Vương- Bình Dương

TOÁN CHUYÊN
A. CƠ SỞ LÝ THUYẾT

1. Bất đẳng thức hoán vị


Định lí 1. Cho hai dãy số thực được sắp thứ tự cùng chiều là
( (
a1 ≤ a2 ≤ . . . ≤ a n a1 ≥ a2 ≥ . . . ≥ a n
hoặc
b1 ≤ b2 ≤ . . . ≤ b n b1 ≥ b2 ≥ . . . ≥ b n .

Gọi ( t1 , t2 , . . . , t n ) là một hoán vị bất kỳ của bộ (b1 , b2 , . . . , b n ). Đặt

S = a 1 b 1 + a 2 b 2 + · · · + a n b n ; F = a 1 b n + a 2 b n−1 + · · · + a n b 1 ; S = a 1 t 1 + a 2 t 2 + · · · + a n t n .

Khi đó
F≤T≤S (1)
Dấu đẳng thức xảy ra khi và chỉ khi một trong hai dãy là dãy dừng, nghĩa là
a 1 = a 2 = · · · = a n hoặc b 1 = b 2 = · · · = b n .
Chú ý 15. Thực ra trong (1) ta có tất cả n! bất đẳng thức khác nhau.

2. Bất đẳng thức Chebyshev


Một trường hợp đặc biệt của bất đẳng thức Chebyshev cho trường hợp ba biến đó là
Định lí 2 (Chebyshev).

1 Với hai dãy số a, b, c và A , B, C đơn điệu cùng chiều, ta có

3(aA + bB + cC ) ≥ (a + b + c)( A + B + C ) (4)

2 Với hai dãy số a, b, c và A , B, C đơn điệu ngược chiều, ta có

3(aA + bB + cC ) ≤ (a + b + c)( A + B + C ) (5)

Chứng minh

ÓNguyễn Thành Nhân 91 Chuyên Hùng Vương-BD


Các Bài Giảng 2023-2024 Đại Số 10 Chuyên

1 Với hain dãy cùng chiều:


Ta xuất phát từ những điều hiển nhiên
(
a≤b
⇒ (a − b)( A − B) ≥ 0 ⇔ aA + bB ≥ aB + bA. (1)
A≤B

Do đó nếu a ≤ b ≤ c và A ≤ B ≤ C thì tương tự ta có được


bB + cC ≥ bC + cB (2); cC + aA ≥ cA + aC. (3)
Cộng vế theo vế (1), (2) và (3) ta được
2(aA + bB + cC ) ≥ a(B + C ) + b(C + A ) + c( A + B). (4)
Cộng vào hai vế của (4) cùng với biểu thức aA + bB + cC ta có
3(aA + bB + cC ) ≥ (a + b + c)( A + B + C ).
Ta có điều cần chứng minh cho trường hợp thứ nhất.
2 Trường hợp hai dãy đợn điệu ngược chiều ta chứng minh tương tự.
Bất đẳng thức Chebyshev tổng quát.
Định lí 3 (Chebyshev). Cho hai dãy các số thực a 1 , a 2 , . . ., a n và b1 , b2 , . . ., b n . Khi đó
1 Nếu hai dãy là đơn điệu cùng chiều thì
LATEX:Nguyễn Thành Nhân

n(a 1 b 1 + a 2 b 2 + · · · + a n b n ) ≥ (a 1 + a 2 + · · · + a n )( b 1 + b 2 + · · · + b n ) (2)

2 Nếu hai dãy là đơn điệu ngược chiều thì


n(a 1 b 1 + a 2 b 2 + · · · + a n b n ) ≤ (a 1 + a 2 + · · · + a n )( b 1 + b 2 + · · · + b n ) (3)

VÍ DỤ 1. Chứng minh rằng, trong tam giác ABC , ta có


aA + bB + cC π
≥ .
a+b+c 3

VÍ DỤ 2. Chứng minh rằng, trong tam giác nhọn ABC , ta có


sin A + sin B + sin C 1
≤ .
cos A + cos B + cos C 3 cot A cot B cot C

B. BÀI TẬP

I-ÁP DỤNG BẤT ĐẲNG THỨC HOÁN VỊ


(
a1 ≥ a2 ≥ . . . ≥ a n
BÀI 1 (IMO 1975). Cho . Gọi ( c 1 , c 2 , . . . , c n ) là một hoán vị tùy ý của (b1 , b2 , . . . , b n ). Chứng
b1 ≥ b2 ≥ . . . ≥ b n
minh rằng
n n
(a i − b i )2 ≤ (a i − c i )2 .
X X
i =1 i =1

BÀI 2. Cho a, b, c là các số thực dương. Chứng minh rằng


b 2 + c 2 c 2 + a2 a2 + b 2 a3 b 3 c 3
a+b+c ≤ + + ≤ + + .
2a 2b 2c bc ca ab
BÀI 3. Cho các số thực dương a, b, c. Chứng minh rằng
2 a3 − b 3 − c 3 2 b 3 − c 3 − a3 2 c 3 − a3 − b 3
+ + ≥ 0.
b2 + c2 c 2 + a2 a2 + b 2
BÀI 4 (IMO 1964). Cho a, b, c là độ dài ba cạnh của một tam giác. Chứng minh rằng
a2 ( b + c − a) + b2 ( c + a − b) + c2 (a + b − c) ≤ 3abc.
BÀI 5 (IMO 1978). Cho a 1 , a 2 , . . ., a n là n số nguyên dương phân biệt. Chứng minh rằng
a1 a2 an 1 1 1
+ +···+ 2 > + +···+ .
12 22 n 1 2 n

ÓNguyễn Thành Nhân 92 Chuyên Hùng VƯƠng-BD


Các Bài Giảng 2023-2024 Đại Số 10 Chuyên

II-ÁP DỤNG BẤT ĐẲNG THỨC CHEBYSHEV


BÀI 6. Cho các số thực dương a 1 , a 2 , . . ., a n có tổng bằng 1. Chứng minh rằng
a1 a2 an n
+ +···+ ≥ .
2 − a1 2 − a2 2 − a n 2n − 1

BÀI 7. Cho các số thực dương a, b, c, d có tổng bình phương bằng 4. Chứng minh rằng
a2 b2 c2 d2 4
+ + + ≥ .
b+c+d c+d+a d+a+b a+b+c 3
BÀI 8 (China MO96). Cho các số thực dương a 1 , a 2 , . . ., a n có tổng bằng 1. Chứng minh rằng
p p p
a1 a2 an a1 + a2 + · · · + a n
p +p +···+ p ≥ p .
1 − a1 1 − a2 1 − an n−1
BÀI 9 (CBGLT-T2). Chứng minh rằng, với tam giác ABC bất kì, ta có
1
1 a cos A + b cos B + c cos C ≤ (a + b + c). 2 18S ≤ (a + b + c) ( h a + h b + h c ).
2
BÀI 10. Chứng minh rằng, với tam giác ABC nhọn, ta có
p
1 sin 2 A + sin 2B + sin 2C ≤ sin A + sin B + sin C . 2 sin 2 A + sin 2B + sin 2C ≤ 3 (cos A + cos B + cos C ).

BÀI 11. Chứng minh rằng, nếu a, b, c > 0 và với mọi số nguyên dương n, ta có
a n+1 + b n+1 + c n+1 p
3
≥ abc.
an + bn + cn
BÀI 12. Chứng minh rằng, nếu a, b, c > 0 và với mọi số tự nhiên n, ta có

TOÁN CHUYÊN
a n+1 b n+1 c n+1 a n + b n + c n
+ + ≥ .
b+c c+a a+b 2

BÀI 6. BẤT ĐẲNG THỨC SCHUR


Nguyễn Thành Nhân- Chuyên Hùng Vương- Bình Dương

A. CƠ SỞ LÝ THUYẾT

1. Bất đẳng thức Schur dạng thông thường


Định lí 1. Cho các số thực dương a, b, c và số k ≥ 0. Khi đó ta có bất đẳng thức
a k (a − b)(a − c) + b k ( b − c)( b − a) + c k ( c − a)( c − b) ≥ 0 (1)
Dấu đẳng thức xảy ra khi và chỉ khi a = b = c hoặc a = b, c = 0 và các hoán vị.
Chứng minh
Không mất tính tổng quát, ta giả sử a ≥ b ≥ c. Khi đó
a k (a − b)(a − c) + b k ( b − c)( b − a) + c k ( c − a)( c − b) = c k (a − c)( b − c) + (a − b)[a k (a − c) − b k ( b − c)] ≥ 0.

Chú ý 16. Hai trường hợp đặc biệt thường gặp là k = 1, k = 2.


• Khi k = 1, ta gọi là bất đẳng thức Schur 3

a3 + b3 + c3 + 3abc ≥ ab(a + b) + bc( b + c) + ca( c + a) (2)

• Khi k = 2, ta gọi là bất đẳng thức Schur 4

a4 + b4 + c4 + abc(a + b + c) ≥ ab(a2 + b2 ) + bc( b2 + c2 ) + ca( c2 + a2 ) (3)


Khi k = 2 ta còn cách viết khác
a4 + b4 + c4 + abc(a + b + c) ≥ a3 ( b + c) + b3 ( c + a) + c3 (a + b) (4)

Chú ý 17. Bất đẳng thức Schur vẫn đúng trong trường hợp k ≤ 0. Trong trường hợp này, dấu đẳng thức xảy
ra khi và chỉ khi a = b = c.

ÓNguyễn Thành Nhân 93 Chuyên Hùng Vương-BD


Các Bài Giảng 2023-2024 Đại Số 10 Chuyên

2. Bất đẳng thức Schur dạng mở rộng


Định lí 2. Cho các số thực dương a, b, c, x, y, z sao cho (a; b; c) và ( x; y; z) là các bộ đơn điệu. Khi đó

x(a − b)(a − c) + y( b − c)( b − a) + z( c − a)( c − b) ≥ 0 (5)

Dấu đẳng thức xảy ra khi và chỉ khi a = b = c.


Chứng minh
Không mất tính tổng quát, ta giả sử a ≥ b ≥ c. Ta xét hai trường

• Nếu x ≥ y ≥ z. Khi đó

( c − a( c − b) ≥ 0 ⇒ z( c − a)( c − b) ≥ 0.

x(a − c) − y( b − c) ≥ x( b − c) − y( b − c) = ( x − y)( b − c) ≥ 0.
Từ đó, nhân hai vế với (a − b) ≥ 0, ta đươc

(a − b)[ x(a − c) − y( b − c)] ≥ 0 ⇔ x(a − b)(a − c) + y( b − c)( b − a) ≥ 0.

Cộng vế theo vế hai bất đẳng thức trên ta có điều cần chứng minh.

• Nếu x ≤ y ≤ z. Khi đó
LATEX:Nguyễn Thành Nhân

(a − b)(a − c) ≥ 0 ⇒ x(a − b)(a − c) ≥ 0.

z(a − c) − y(a − b) ≥ z(a − b) − y(a − b) = ( z − y)(a − b) ≥ 0.


Nhân hai vế với b − c ≥ 0, ta được

( b − c)[ z(a − c) − y(a − b)] ≥ 0 ⇔ z( c − a)( c − b) + y( b − a)( b − c) ≥ 0.

Cộng vế theo theo vế hai bất đẳng thức trên ta có điều cần chứng minh.

3. Kỹ thuật p, q, r trong bất đẳng thức Schur


Khi làm các bài toán về áp dụng bất đẳng thức Schur, chúng ta thường gặp tình huống sử dụng đến các bất
đẳng thức qua các biểu thức đối xứng sơ cấp. Lúc đó với ba biến a, b, c, ta đặt

p = a + b + c; q = ab + bc + ca; r = abc.

Chúng ta xây dựng hệ thống bất đẳng thức phụ liên quan đến p, q, r . Chúng ta gọi đó là kĩ thuật p, q, r .
Sau đây là một số bất đẳng thức cơ bản về mối quan hệ giữa p, q, r .

1 Từ bất đẳng thức Schur, với k = 0, k = 1, k = 2, ta lần lượt có

p2 ≥ 3 q; p3 − 4 pq + 9 r ≥ 0; p4 − 5 p2 q + 4 q2 + 6 pr ≥ 0.

2 Bằng cách sử dụng bất đẳng thức AM − GM , ta có

p2 ≥ 3 q; p2 q + 3 pr ≥ 4 q2 ; p2 q ≥ 3 pr + 2 q2 ; q3 + 9 r 2 ≥ 4 pqr
q2 ≥ 3 pr, pq2 + 3 qr ≥ 4 p2 r ; pq2 ≥ 2 p2 r + 3 qr ; 2 q3 + 9 r 2 ≥ 7 pqr
p3 ≥ 27 r ; p4 + 3 q2 ≥ 4 p2 q; 2 p3 + 9 r ≥ 7 pq; p3 r + q3 ≥ 6 pqr

B. BÀI TẬP ÁP DỤNG


BÀI 1. Chứng minh rằng với mọi số thực dương a, b, c ta có

a2 + bc b2 + ca c2 + ab a b c
+ + ≥ + + .
( b + c)2 ( c + a)2 (a + b)2 b + c c + a a + b

BÀI 2 (VMO 2015). Cho các số thực a, b, c ≥ 0. Chứng minh rằng


p p p
3(a2 + b2 + c2 ) ≥ (a + b + c)( ab + bc + ca) + (a − b)2 + ( b − c)2 + ( c − a)2 ≥ (a + b + c)2 .

ÓNguyễn Thành Nhân 94 Chuyên Hùng Vương-BD


Các Bài Giảng 2023-2024 Đại Số 10 Chuyên

BÀI 3. Chứng minh rằng với mọi số thực dương a, b, c ta có


1 1 1 b+c c+a a+b
+ + ≥ + + .
a b c a2 + bc b2 + ca c2 + ab

BÀI 4. Chứng minh rằng với mọi số thực dương có tích bằng 1 thì

a2 + bc b2 + ca c2 + ab
+ + ≥ ab + bc + ca.
a2 ( b + c ) b 2 ( c + a) c 2 ( a + b )

BÀI 5 (Đề tuyển sinh chuyên KHTN 2020-Đề chung).


Với a, b, c là những số thực dương thỏa mãn a + b + c = 3. Chứng minh rằng

a(a + bc)2 b( b + ca)2 c( c + ab)2


+ + ≥ 4.
b(ab + 2 c2 ) c( bc + 2a2 ) a( ca + 2 b2 )

BÀI 6 (Chuyên KHTN TST 2019). Cho các số thực dương a, b, c là độ dài ba cạnh của một tam giác. Chứng
ming rằng
a b c 6a 6b 6c
+ + + + + ≥ 6. (1)
b + c c + a a + b 2a + b + c 2 b + c + a 2 c + a + b
BÀI 7 (Chuyên KHTN TST 2011). Cho các số thực dương a, b, c có tổng bằng 1. Chứng minh rằng
13 X a
(ab + bc + ca) ≤ 1 + 4abc · .
4 (a + 1)2

BÀI 8 (Balkal). Cho a, b, c là các số thực dương và thỏa mãn abc = 1. Chứng minh rằng

2(a2 + b2 + c2 ) + 12 ≥ 3(a + b + c) + 3(ab + bc + ca).

TOÁN CHUYÊN
BÀI 9. Cho a, b, c là các số thực không âm và thỏa mãn ab + bc + ca + abc = 4. Chứng minh rằng

3(a2 + b2 + c2 ) + abc ≥ 10.

BÀI 10. Cho a, b, c là các số thực không âm và thỏa mãn ab + bc + ca + abc = 4. Chứng minh rằng

a2 + b2 + c2 + 5abc ≥ 8.

BÀI 11. Cho a, b, c là những số thực không âm. Chứng minh rằng
p p p
a 4a2 + 5 bc + b 4 b2 + 5 ca + c 4 c2 + 5ab ≥ (a + b + c)2 .

BÀI 12 (Olympic chuyên KHTN 2022). Cho a, b, c là các số thực không âm, không có hai số nào đồng thời
bằng 0 và thỏa mãn a2 + b2 + c2 = 1. Chứng minh rằng

a3 b3 c3 1
p +p +p ≥p .
b2 + c2 c 2 + a2 a2 + b 2 2(ab + bc + ca)

BÀI 13. Cho a, b, c độ dài ba cạnh tam giác. Chứng minh rằng

3(a4 + b4 + c4 ) ab + bc + ca
+ 2 ≥ 2.
( a 2 + b 2 + c 2 )2 a + b2 + c2

BÀI 14 (Iran 1996). Cho x, y, z ≥ 0. Chứng minh rằng


1 1 1 9
· ¸
( x y + yz + zx) + + ≥ .
( x + y)2 ( y + z)2 ( z + x)2 4

C. BÀI TẬP
BÀI 15. Cho a, b, c là các số thực không âm. Chứng minh rằng
1 1 1 a+b+c 3
+ + ≥ + .
a + b b + c c + a 2(ab + bc + ca) a + b + c

BÀI 16. Cho a, b, c là các số thực không âm. Chứng minh rằng
p3
a b c 3 abc
+ + + ≥ 2.
b + c c + a a + b 2(a + b + c)

ÓNguyễn Thành Nhân 95 Chuyên Hùng Vương-BD


Các Bài Giảng 2023-2024 Đại Số 10 Chuyên

BÀI 17. Cho a, b, c là các số thực không âm. Chứng minh rằng
(a4 + b4 + c4 )(ab + bc + ca) ≥ (a2 + b2 + c2 )(a2 b2 + b2 c2 + c2 a2 ).
BÀI 18. Cho a, b, c là các số thực không âm và ab + bc + ca + 6abc = 9. Tìm hằng số k tốt nhất sao cho
a + b + c + kabc ≥ k + 3
đúng với mọi a, b, c không âm.
BÀI 19. Chứng minh rằng nếu a, b c là các số dương có tổng bằng 3 thì
1 1 1 3 3 3
+ + ≥ + + .
a b c 2a2 + bc 2 b2 + ca 2 c2 + ab
BÀI 20. Chứng minh rằng nếu a, b c là các thực có tổng bằng 3 thì
a3 + abc b3 + abc c3 + abc 3
+ + ≥ .
( b + c)2 ( a + c )2 ( c + a)2 2
BÀI 21. Với a, b, c không âm. Chứng minh rằng
a2 + bc b2 + ac c2 + ab
+ + ≥ 2.
b2 + bc + c2 a2 + ac + c2 a2 + ab + b2
BÀI 22. Chứng minh rằng nếu a, b c là các số thực không âm thì
a2 + 2 bc b2 + 2 ca c2 + 2ab 9
LATEX:Nguyễn Thành Nhân

+ + ≥ .
( b + c)2 ( c + a)2 (a + b)2 4
BÀI 23 (IMO). Chứng minh rằng nếu a, b c, d e là các số thực tùy ý thì
(a − b)(a − c)(a − d )(a − e) + ( b − a)( b − c)( b − d )( b − e) +
( c − a)( c − b)( c − d )( c − e) + ( d − a)( d − b)( d − c)( d − e) + ( e − a)( e − b)( e − c)( e − d ) ≥ 0.
BÀI 24 (Thi Olympic chuyên KHTN 2015). Cho x, y, z > 0 và x y + yz + zx = 1. Chứng minh rằng
x y z 1
p p +p p +p p ≤ p .
yz + 3 xz + 3 x y + 3 4 3 x yz

BÀI 7. CÁC KỸ THUẬT CĂN BẢN CHỨNG MINH BẤT ĐẲNG


THỨC
Nguyễn Thành Nhân- Chuyên Hùng Vương- Bình Dương

A. CƠ SỞ LÝ THUYẾT

1. Kỹ thuật chuẩn hóa


1 Biểu thức f (a, b, c) được gọi là thuần nhất bậc k ( k ∈ N∗ ) theo ba biến a, b, c nếu ta có

f ( ta, tb, tc) = t k · f (a, b, c), t ∈ R. (1)

2 Bất đẳng thức


f (a, b, c) ≥ 0 hoặc f (a, b, c) ≤ 0
với f (a, b, c) là biểu thức thuần nhất được gọi là bất đẳng thức thuần nhất ba biến.
3 Kỹ thuật chuẩn hóa
Xét bất đẳng thức thuần nhất
f (a, b, c) ≥ 0 (2)
với biến a, b, c thỏa mãn điều kiện D nào đó.
Bằng cách đặt a = ta 1 , b = tb1 , c = tc 1 với t > 0, do tính thuần nhất nên ta có bất đẳng thức tương đương
t k f (a 1 , b 1 , c 1 ) ≥ 0 ⇔ f (a 1 , b 1 , c 1 ) ≥ 0 (3)
Về mặt hình thức thì bất đẳng thức (3) và bất đẳng thức (2) giống nhau. Tuy nhiên với phép đặt a = ta 1 ,
b = tb 1 , c = tc 1 thì điều kiện D trở thành điều kiện D1 nào đó. Chúng ta hoàn toàn có thể điều chỉnh t
để điều kiện D1 là theo chủ quan. Chẳng hạn là bất đẳng thức (2) với điều kiện kèm theo là a, b, c > 0 ta
có thể điều chỉnh t để có điều kiện tương ứng của bất đẳng thức (3) là a 1 + b1 + c 1 = 1. Kỹ thuật chuyển
đổi điều kiện trong bất đẳng thức thuần nhất được gọi là kỹ thuật chuẩn hóa.

ÓNguyễn Thành Nhân 96 Chuyên Hùng Vương-BD


Các Bài Giảng 2023-2024 Đại Số 10 Chuyên

4 Một số phép chuẩn hóa thường sử dụng.


Chúng ta thường sử dụng một số phép chuẩn hóa thông dụng sau đây

1 a + b + c = m; 2 ab + bc + ca = m;

3 a n + b n + c n = m; 4 abc = m.

Chú ý 18. Bản chất của phép chuẩn hóa là ta chuyển từ bất đẳng thức không có điều kiện về bất đẳng thức
có điều kiện.
BÀI 1. Giả sử a, b, c là các số thực dương. Chứng minh rằng
s s
3 (a + b)( b + c)( c + a) ab + bc + ca
≥ . (4)
8 3

BÀI 2 (VMO 2002- Bảng B). Chứng minh rằng với mọi số thực dương a, b, c thì ta có
3
6(a + b + c)(a2 + b2 + c2 ) ≤ 27abc + 10(a2 + b2 + c2 ) 2 . (8)

BÀI 3. Cho a, b, c là các số thực dương thỏa mãn 8 a2 + b2 + c2 = 9(ab + bc + ca). Tìm giá trị lớn nhất và giá trị
¡ ¢

a+b b+c c+a


nhỏ nhất của biểu thức T = + + .
c a b
BÀI 4. Cho x, y, z là các số thực dương thỏa mãn x y + yz + zx = x yz( x + y + z). Chứng minh rằng
1 1 1
+ + ≥ 1.
2x + 1 2 y + 1 2z + 1

TOÁN CHUYÊN
2. Kỹ thuật đánh giá đại diện
BÀI 5. Cho a, b, c là các số dương. Chứng minh rằng

a3 b3 c3 a+b+c
2 2
+ 2 2
+ 2 2
≥ . (10)
a + ab + b b + bc + c c + ca + a 3

BÀI 6. Cho a, b, c dương. Chứng minh rằng


p p p p
a2 + ab + b2 + b2 + bc + c2 + c2 + ca + a2 ≥ 3 (a + b + c) . (11)

BÀI 7. Cho a, b, c là các số thực dương. Chứng minh rằng


1 1 1 1
a) + + ≤ ;
a3 + b3 + abc b3 + c3 + abc c3 + a3 + abc abc
a3 + b 3 b3 + c3 c 3 + a3 2
b) + + ≥ (a + b + c).
a2 + ab + b2 b2 + bc + c2 c2 + ca + a2 3

3. Kỹ thuật hệ số bất định


BÀI 8. Cho a, b, c là các số thực dương. Chứng minh rằng

5 b 3 − a3 5 c 3 − b 3 5 a3 − c 3
+ + ≤ a + b + c. (12)
ab + 3 b2 bc + 3 c2 ca + 3a2

BÀI 9. Cho các số thực dương a, b, c thỏa mãn a + b + c = 3. Chứng minh rằng

1 1 1 2(a2 + b2 + c2 )
+ + + ≥ 5. (15)
a2 b 2 c 2 3

4. Sử dụng phần tử cực hạn


1 Bất đẳng thức hoán vị
Dạng f (a 1 ; a 2 ; , . . . ; a n ) ≥ m
(17), trong đó m là hằng số còn f (a 1 ; a 2 ; , . . . ; a n ) là một biểu thức nhận giá
trị không đổi khi ta hoán vị vòng quanh bộ số (a 1 ; a 2 ; , . . . ; a n ).
X
Ký hiệu (chữ cyc là viết tắt của cyclic: Hoán vị vòng quanh).
c yc

ÓNguyễn Thành Nhân 97 Chuyên Hùng Vương-BD


Các Bài Giảng 2023-2024 Đại Số 10 Chuyên

2 Bất đẳng thức đối xứng


Dạng f (a 1 ; a 2 ; , . . . ; a n ) ≥ n
(18), trong đó n là hằng số còn f (a 1 ; a 2 ; , . . . ; a n ) là một biểu thức nhận giá trị
không đổi khi ta hoán vị bất kỳ bộ số (a 1 ; a 2 ; , . . . ; a n ).
X
Ký hiệu (chữ sym là viết tắt của symetry: Đối xứng).
s ym

3 Phương pháp phần tử cực hạn


+ Trong bất đẳng thức đối xứng, ta thường giả sử a 1 ≥ a 2 ≥ · · · ≥ a n hoặc a 1 ≤ a 2 ≤ · · · ≤ a n .
+ Trong bất đẳng thức hoán vị, ta thường giả sử a 1 = max{a 1 , a 2 , . . . , a n } hoặc a 1 = min{a 1 , a 2 , . . . , a n }.

BÀI 10 (Toán học và Tuổi trẻ 10-2004). Cho a, b, c là các số không âm và thỏa mãn a + b + c = 1. Tìm giá trị lớn
nhất của biểu thức
a2 + 1 b 2 + 1 c 2 + 1
P= + + . (19)
b 2 + 1 c 2 + 1 a2 + 1
Chú ý 19 (Bài toán tổng quát). Cho a, b, c là các số thực không âm và a + b + c = 1. Chứng minh rằng với n là
số nguyên dương và n ≥ 2, ta có
an + 1 bn + 1 cn + 1 7
P= + + ≤ . (20)
bn + 1 cn + 1 an + 1 2
BÀI 11 (Toán học tuổi trẻ 6-2000). Cho các số a, b, c ∈ [1; 2]. Chứng minh rằng

a3 + b3 + c3 ≤ 5abc. (21)
LATEX:Nguyễn Thành Nhân

5. Kỹ thuật tiếp tuyến hóa


Giả sử ta cần chứng minh bất đẳng thức f ( x) ≥ g( x) với x thỏa điều kiện nào đó. Xét hàm số y = f ( x) có tiếp
tuyến là y = g( x). Khi đó do phương trình hoành độ giao điểm f ( x) = g( x) có nghiệm kép x = x0 với x0 là hoành
độ tiếp điểm nên ta có
f ( x) − g( x) = 0 ⇔ ( x − x0 )2 · h( x).

Ta thấy nếu f ( x) là đa thức thì bậc của h( x) đã giảm đi 2 bậc. Do đó việc đánh giá h( x) trở nên dễ dàng.
BÀI 12 (HongKong 2005). Cho các số thực dương a, b, c, d thỏa mãn a + b + c + d = 1. Chứng minh rằng

1
6(a3 + b3 + c3 + d 3 ) − (a2 + b2 + c2 + d 2 ) ≥ .
8

BÀI 13 (USA-2003). Chứng minh rằng với a, b, c > 0 ta có

( b + c + 2 a )2 ( c + a + 2 b)2 ( a + b + 2 c )2
+ + ≤ 8.
2a2 + ( b + c)2 2 b2 + ( c + a)2 2 c2 + (a + b)2

BÀI 14 (Romania 2005). Cho a, b, c > 0 và a + b + c = 3. Chứng minh rằng

1 1 1
2
+ 2 + 2 ≥ a2 + b 2 + c 2 .
a b c

BÀI 15. Cho bốn số thực không âm a, b, c, d thỏa mãn a + b + c + d = 4. Chứng minh rằng

a b c d 1
+ + + ≤ .
5 + 3 a2 5 + 3 b 2 5 + 3 c 2 5 + 3 d 2 2

BÀI 16. Cho a, b, c là độ dài ba cạnh của một tam giác. Chứng minh rằng

1 1 1 9 1 1 1
µ ¶
+ + + ≥4 + + .
a b c a+b+c a+b b+ c c+1

B. BÀI TẬP

KỸ THUẬT CHUẨN HÓA


BÀI 17 (Olympic 30/4/2006). Chứng minh rằng với a, b, c > 0 ta có

a( b + c ) b ( c + a) c(a + b) 6
2 2
+ 2 2
+ 2 2
≤ .
( b + c) + a ( c + a) + b (a + b) + c 5

ÓNguyễn Thành Nhân 98 Chuyên Hùng Vương-BD


Các Bài Giảng 2023-2024 Đại Số 10 Chuyên

BÀI 18. Chứng minh rằng với a, b, c > 0 ta có

a2 b2 c2 3
2 2
+ 2 2
+ 2 2
≥ .
a + ( b + c) b + ( c + a) c + (a + b) 5

BÀI 19. Chứng minh rằng với a, b, c > 0 ta có

( b + c − 3 a )2 ( c + a − 3 b)2 (a + b − 3 c)2 1
2 2
+ 2 2
+ 2 2
≥ .
2a + ( b + c) 2 b + ( c + a) 2 c + (a + b) 2

BÀI 20 (USA-2003). Chứng minh rằng với a, b, c > 0 ta có

( b + c + 2a)2 ( c + a + 2 b )2 (a + b + 2 c)2
2 2
+ 2 2
+ 2 ≤ 8.
2a + ( b + c) 2 b + ( c + a) 2 c + ( a + b )2

BÀI 21. Cho ba số thực không âm x, y, z thỏa mãn x + y + z > 0. Tìm giá trị lớn nhất và giá trị nhỏ nhất của
biểu thức
x3 + y3 + 16 z3
P= .
( x + y + z )3

KỸ THUẬT ĐÁNH GIÁ DẠI DIỆN


BÀI 22 (Học sinh giỏi Đồng Nai). Cho 0 < x, y, z < 1 và x y + yz + zx = 1. Chứng minh rằng
x y z 3
+ + ≥ .
1 − x 2 1 − y2 1 − z 2 2

TOÁN CHUYÊN
BÀI 23. Chứng minh rằng nếu a, b, c là độ dài ba cạnh của một tam giác và x, y, z là độ dài các đường phân
giá trong thì
1 1 1 1 1 1
+ + > + + .
x y z a b c

KỸ THUẬT HỆ SỐ BẤT ĐỊNH


BÀI 24. Cho a, b, c > 0 và a3 + b3 + c3 = 3. Chứng minh rằng
1 1 1
µ ¶
5(a2 + b2 + c2 ) ≥ 27 − 4 + + .
a b c

BÀI 25 (Chuyên Vĩnh Phúc). Cho a, b, c > 0 và a2 + b2 + c2 = 3. Chứng minh rằng


1 1 1 3 15
+ + + (a + b + c) ≥ .
a b c 2 2

PHƯƠNG PHÁP PHẦN TỬ CỰC HẠN


BÀI 26 (VMO 2020- Bài 2- Ngày 1).
a) Cho ba số thực a, b, c thỏa mãn a2 + b2 + c2 = 1. Chứng minh rằng
p
| a − b | + | b − c | + | c − a | ≤ 2 2.

b) Cho 2019 số thực a 1 , a 2 , . . ., a 2019 thỏa mãn a21 + a22 + · · · + a22019 = 1. Tìm giá trị lớn nhất của biểu thức

S = |a 1 − a 2 | + |a 2 − a 3 | + · · · + |a 2019 − a 1 |.

BÀI 27 (Toán học tuổi trẻ 3-2008). Cho a, b, c ∈ [1; 2]. Chứng minh rằng
1 1 1 45
µ ¶
(3a + 2 b + c) + + ≤ .
a b c 2

BÀI 28 (Đề 150-Bộ đề TSĐH). Cho các số thực a, b, c ∈ [0; 1]. Tìm giá trị lớn nhất của biểu thức
a b c
F= + + .
b+ c+1 c+a+1 a+b+1

ÓNguyễn Thành Nhân 99 Chuyên Hùng Vương-BD


Các Bài Giảng 2023-2024 Đại Số 10 Chuyên

BÀI 29. Cho a, b, c là độ dài ba cạnh của một tam giác. Chứng minh rằng

P = a 2 b ( a − b ) + b 2 c ( b − c ) + c 2 a ( c − a ) ≥ 0. (22)

BÀI 30. Tìm tất cả các số nguyên dương x, y, z, n thỏa mãn đẳng thức

x! + y! + z! = n!. (23)

BÀI 31. Tìm giá trị lớn nhất và giá trị nhỏ nhất của biểu thức

P = max{a, b, c} − min{a, b, c},

trong đó a, b c là các số thực thỏa mãn a + b + c = a3 + b3 + c3 − 3abc = 2.


LATEX:Nguyễn Thành Nhân

ÓNguyễn Thành Nhân 100 Chuyên Hùng Vương-BD


CHƯƠNG 6 PHƯƠNG TRÌNH-HỆ PHƯƠNG TRÌNH

BÀI 1. ĐẠI CƯƠNG VỀ PHƯƠNG TRÌNH, BẤT PHƯƠNG


TRÌNH
Nguyễn Thành Nhân- Chuyên Hùng Vương- Bình Dương

A. CƠ SỞ LÝ THUYẾT

1. Khái niệm về phương trình một ẩn


Định nghĩa 1. Cho hai hàm số y = f ( x) và y = g( x) có tập xác định lần lượt là D f và D g . Đặt D = D f ∩ D g .
Mệnh đề chứa biến “ f ( x) = g( x) ” được gọi là phương trình một ẩn; x được gọi là ẩn số (hay ẩn) và D gọi là
tập xác định của phương trình.
Số x0 ∈ D được gọi là một nghiệm của phương trình nếu mệnh đề “ f ( x0 ) = g( x0 ) ” là một mệnh đề đúng.
Chú ý 20.
• Khi giải phương trình, để đơn giản, ta không cần ghi tập xác định mà chỉ cần tìm điều kiện của phương
trình là được.
• Khi giải phương trình, ta có thể dùng máy tính để tìm các nghiệm gần đúng.

TOÁN CHUYÊN
• Nghiệm của phương trình f ( x) = g( x) chính là hoành độ giao điểm của hai đồ thị y = f ( x) và y = g( x).

2. Phương trình tương đương


Định nghĩa 2. Hai phương trình (cùng ẩn) được gọi là tương đương nếu chúng có cùng tập nghiệm. Nếu
phương trình f 1 ( x) = g 1 ( x) tương đương với phương trình f 2 ( x) = g 2 ( x). Ta viết

f 1 ( x) = g 1 ( x) ⇔ f 2 ( x) = g 2 ( x).

Chú ý 21. Từ định nghĩa ta thấy, hai phương trình vô nghiệm thì tương đương, vì tập nghiệm của chúng là

Phép biến đổi tương đương


Định nghĩa 3. Phép biến đổi tương đương là phép biến đổi biến một phương trình thành một phương trình
tương đương với nó.

Các phép biến đổi tương đương


Định lí 1. Cho phương trình f ( x) = g( x) có tập xác định D; y = h( x) là một hàm số xác định trên D (h( x) có thể
là một hằng số). Khi đó trên D, phương trình đã cho tương đương với mỗi phương trình sau
1 f ( x ) + h ( x ) = g ( x ) + h ( x );

2 f ( x) · h( x) = g( x) · h( x) nếu h( x) ̸= 0 với mọi x ∈ D.

Nhận xét 10. Từ định lý trên, ta thấy rằng quy tắc chuyển vế đổi dấu, quy tắc nhân hai vế với một số khác 0
là các phép biến đổi tương đương.

3. Phương trình hệ quả


Định nghĩa 4. Phương trình f 1 ( x) = g 1 ( x) được gọi là phương trình hệ quả của phương trình f ( x) = g( x) nếu
tập nghiệm của nó chứa tập nghiệm của phương trình f ( x) = g( x). Ta viết

f ( x) = g( x) ⇒ f 1 ( x) = g 1 ( x).

Các nghiệm của phương trình f 1 ( x) = g 1 ( x) mà không phải là nghiệm của phương trình f ( x) = g( x) được gọi là
các nghiệm ngoại lai.

101
Các Bài Giảng 2023-2024 Đại Số 10 Chuyên

Phép biến đổi hệ quả


Định lí 2. Khi bình phương hai vế của một phương trình, ta được phương trình hệ quả của phương trình đã
cho
f ( x) = g( x) ⇒ [ f ( x)]2 = [ g( x)]2 .
Chú ý 22.
1 Có thể chứng minh được rằng: Nếu hai vế của một phương trình luôn cùng dấu thì khi bình phương
hai vế của nó, ta được phương trình tương đương.
2 Nếu phép biến đổi một phương trình dẫn đến phương trình hệ quả, thì sau khi giải phương trình hệ
quả, ta phải thử lại nghiệm và loại bỏ nghiệm ngoại lai.

VÍ DỤ 1. Bình phương hai vế để giải các phương trình sau đây, ta thu được phương trình hệ quả
p
1 x = 2− x 2 | x − 1| = x − 3.

4. Phương trình nhiều ẩn


Định nghĩa 5. Phương trình có từ hai ẩn trở lên được gọi là phương trình nhiều ẩn. Mỗi bộ số gồm các số
ứng với các ẩn mà thỏa mãn phương trình đã cho được gọi là một nghiệm của phương trình đó.
LATEX:Nguyễn Thành Nhân

VÍ DỤ 2. Phương trình x2 + y2 − 2 x + 4 y − 10 = 0.

5. Phương trình có chứa tham số


Phương trình ngoài các ẩn còn có những chữ khác gọi là tham số.

VÍ DỤ 3. Phương trình x2 + 2mx + 3m − m2 + 4 = 0 thì ẩn là x và m là tham số.

BÀI 2. PHƯƠNG TRÌNH, BẤT PHƯƠNG TRÌNH BẬC NHẤT VÀ


BẬC HAI
Nguyễn Thành Nhân- Chuyên Hùng Vương- Bình Dương

A. CƠ SỞ LÝ THUYẾT
1. Giải và biện luận phương trình dạng ax + b = 0
Xét phương trình dạng ax + b = 0 (1).
b
1 Nếu a ̸= 0 thì phương trình có nghiệm duy nhất x = − .
a
2 Nếu a = 0 và b ̸= 0 thì phương trình vô nghiệm.

3 Nếu a = b = 0 thì phương trình có nghiệm là ∀ x ∈ R.

2. Giải và biện luận bất phương trình dạng ax + b ≥ 0


Xét bất phương trình dạng ax + b ≥ 0 (2).
b
1 Nếu a > 0 thì bất phương trình có nghiệm x ≥ − .
a
b
2 Nếu a < 0 thì bất phương trình có nghiệm x ≤ − .
a
3 Nếu a = 0 và b ≥ 0 thì bất phương trình có nghiệm là ∀ x ∈ R.

4 Nếu a = 0 và b < 0 thì bất phương trình vô nghiệm.

ÓNguyễn Thành Nhân 102 Chuyên Hùng Vương-BD


Các Bài Giảng 2023-2024 Đại Số 10 Chuyên

3. Dấu của nhị thức bậc nhất


Định nghĩa 1. Nhị thức bậc nhất ẩn x là biểu thức có dạng f ( x) = ax + b với a, b ∈ R và a ̸= 0.

DẤU CỦA NHỊ THỨC BẬC NHẤT


b b
Nhị thức f ( x) cùng dấu với a khi x > − , trái dấu với a khi x < − .
a a

Quy tắc về dấu tam thức


“Trước tái-Sau cùng.”

4. Giải và biện luận phương trình dạng ax2 + bx + c = 0


Xét phương trình dạng ax2 + bx + c = 0 (3).

1 Nếu a = 0 thì phương trình đưa về bx + c = 0.

2 Nếu a ̸= 0 ta xét ∆ = b2 − 4ac.


p
−b ± ∆
+ Nếu ∆ > 0, phương trình có hai nghiệm phân biệt x = .
2a
−b
+ Nếu ∆ = 0, phương trình có nghiệm kép x = .
2a
+ Nếu ∆ < 0, phương trình vô nghiệm.

TOÁN CHUYÊN
5. Dấu của tam thức bậc hai
Định nghĩa 2. Tam thức bậc hai ẩn x là biểu thức có dạng f ( x) = ax2 + bx + c với a, b, c ∈ R và a ̸= 0.
Dấu của tam thức được thể hiện qua định lý sau đây gọi là định lý về dấu của tam thức bậc hai
Định lí 1 (Định lý về dấu của tam thức bậc hai).
Cho tam thức f ( x) = ax2 + bx + c (a ̸= 0).

• Nếu ∆ < 0 thì f ( x) cùng dấu với a với mọi x ∈ R.

b
• Nếu ∆ = 0 thì f ( x) cùng dấu với a với mọi x ̸= − .
2a

• Nếu ∆ > 0 thì


+ f ( x) cùng dấu với a với mọi x nằm ngoài đoạn hai nghiệm [ x1 ; x2 ] của f ( x).
+ f ( x) trái dấu với a với mọi x nằm trong khoảng hai nghiệm ( x1 ; x2 ) của f ( x).

Quy tắc
“Trong trái- Ngoài cùng.”

6. Định lý đảo về dấu của tam thức bậc hai


Định lí 2. Cho tam thức bậc hai f ( x) = ax2 + bx + c với a ̸= 0 và số thực α. Điều kiện cần và đủ để tam thức f ( x)
có hai nghiệm x1 , x2 thỏa mãn x1 < α < x2 là a · f (α) < 0.
Chú ý 23. Điều kiện a f (α) < 0 tương đương với ∆ > 0 nên không cần tìm điều kiện để phương trình có nghiệm.
Điều kiện a f (α) > 0 không tương đương với ∆ > 0 nên lúc này cần tìm điều kiện ∆ > 0 để phương trình có
nghiệm.

7. Các hệ quả
Hệ quả 1. Điều kiện cần và đủ để tam thức f ( x) = ax2 + bx + c có hai nghiệm trái dấu là a · c < 0.
Hệ quả 2. Điều kiện cần và đủ để tam thức f ( x) = ax2 + bx + c có một nghiệm nằm trong khoảng (α; β) và một
nghiệm nằm ngoài đoạn [α; β] là f (α) · f (β) < 0.

ÓNguyễn Thành Nhân 103 Chuyên Hùng Vương-BD


Các Bài Giảng 2023-2024 Đại Số 10 Chuyên

8. Các dạng toán thường gặp


Sau đây chúng ta xét một số dạng toán so sánh nghiệm của tam thức bậc hai với một số thường gặp.

1 f ( x) có duy nhất một nghiệm x > α


Ta có ba trường hợp

• Trường hợp 1:
f ( x) có nghiệm x1 < α < x2 ⇔ a f (α) < 0.
• Trường hợp 2:
∆ = 0

f ( x) có nghiệm α < x1 = x2 ⇔ S
 > α.
2
• Trường hợp 3:
 f (α) = 0

f ( x) có nghiệm α = x1 < x2 ⇔ S
 > α.
2
2 f ( x) có ít nhất một nghiệm x > α
Ta có ba trường hợp

• Trường hợp 1:
LATEX:Nguyễn Thành Nhân

f ( x) có nghiệm x1 < α < x2 ⇔ a f (α) < 0.


• Trường hợp 2:
 f (α) = 0

f ( x) có nghiệm α = x1 < x2 ⇔ S
 > α.
2



 ≥ 0

a f (α) > 0

• f ( x) có nghiệm α < x1 , x2 ⇔
 S > α.



2
Chú ý 24. Có thể sử dụng phương pháp gián tiếp để giải. Ta tìm điều kiện để f ( x) không có nghiệm
x > α. Lúc này có hai trường hợp xảy ra

• Trường hợp 1:
f ( x) vô nghiệm ⇔ ∆ < 0.
• Trường hợp 2:
∆≥0




a f (α) ≥ 0

f ( x) có nghiệm x1 , x2 ≤ α ⇔
S

 ≤ α.

2
Sau đó loại đi các giá trị của tham số thỏa mãn trường hợp gián tiếp, ta được các giá trị tham số thỏa
mãn yêu cầu.

3 f ( x) có ít nhất một nghiệm thuộc đoạn [α; β]


Ta có ba trường hợp

• Trường hợp 1:
f ( x) có nghiệm α hoặc nghiệm là β ⇔ f (α) · f (β) = 0.
• Trường hợp 2:
f ( x) có một nghiệm nằm trong khoảng (α; β) và một nghiệm nằm ngoài đoạn [α; β] ⇔ f (α) · f (β) < 0.
• Trường hợp 3:
∆ ≥ 0



a f (α) > 0



f ( x) có cả hai nghiệm thuộc khoảng (α; β) ⇔ a f (β) > 0


α < S < β.




2
4 f ( x) có ít nhất một nghiệm thuộc khoảng (α; β)
Ta có bốn trường hợp

ÓNguyễn Thành Nhân 104 Chuyên Hùng Vương-BD


Các Bài Giảng 2023-2024 Đại Số 10 Chuyên

• Trường hợp 1: (
f (α) = 0
f ( x) có nghiệm α, nghiệm còn lại thuộc (α; β) ⇔
α < S − α < β.
• Trường hợp 2: (
f (β) = 0
f ( x) có nghiệm β, nghiệm còn lại thuộc (α; β) ⇔
α < S − β < β.
• Trường hợp 3:
f ( x) có một nghiệm nằm trong khoảng (α; β) và một nghiệm nằm ngoài đoạn [α; β] ⇔ f (α) · f (β) < 0.
• Trường hợp 4:
∆≥0




a f (α) > 0



f ( x) có cả hai nghiệm thuộc khoảng (α; β) ⇔ a f (β) > 0


S


α < < β.


2
5 f ( x) có ít nhất một nghiệm ngoài (α; β)
Ta dùng phương pháp gián tiếp. Tìm điều kiện để f ( x) không có nghiệm ngoài (α; β). Có các trường
hợp sau
• Trường hợp 1:
f ( x) vô nghiệm ⇔ ∆ < 0.
• Trường hợp 2:
∆≥0




a f (α) > 0

TOÁN CHUYÊN


f ( x) có cả hai nghiệm thuộc khoảng (α; β) ⇔ a f (β) > 0


S


α < < β.


2
Loại đi các giá trị của tham số m thỏa mãn trường hợp gián tiếp, ta được các giá trị cần tìm thỏa bài
toán.

9. Một số kết quả thường dùng


Xét tam thức f ( x) = ax2 + bx + c (a ̸= 0). Ta có một số kết quả sau đây
( (
a>0 a<0
1 f ( x) > 0, ∀ x ∈ R ⇔ 2 f ( x) < 0, ∀ x ∈ R ⇔
∆ < 0. ∆ < 0.
( (
a>0 a<0
3 f ( x) ≥ 0, ∀ x ∈ R ⇔ 4 f ( x) ≤ 0, ∀ x ∈ R ⇔
∆ ≤ 0. ∆ ≤ 0.

Tổng quát hơn


Xét biểu thức có dạng f ( x) = ax2 + bx + c với a có thể bằng 0. Khi đó
( (
a=b=0 a=b=0
 c>0  c<0
 
1 f ( x) > 0, ∀ x ∈ R ⇔ 
( 2 f ( x) < 0, ∀ x ∈ R ⇔ 
(
 a>0  a<0
∆ < 0. ∆ < 0.
( (
a=b=0 a=b=0
 

 c≥0  c≤0
 
3 f ( x) ≥ 0, ∀ x ∈ R ⇔ 
( 4 f ( x) ≤ 0, ∀ x ∈ R ⇔ 
(
 a>0  a<0
∆ ≤ 0. ∆ ≤ 0.

Chú ý 25. Yêu cầu f ( x) không đổi dấu ∀ x ∈ R dẫn đến ∆ < 0 hoặc ∆ ≤ 0.

VÍ DỤ 1. Cho phương trình f ( x) = x2 − (m + 2) x + 5m + 1 = 0. Tìm m sao cho


a) Phương trình chỉ có một nghiệm x > 1.

ÓNguyễn Thành Nhân 105 Chuyên Hùng Vương-BD


Các Bài Giảng 2023-2024 Đại Số 10 Chuyên

ĐS: m = 16 hoặc m < 0


b) Phương trình có ít nhất một nghiệm thỏa | x| > 4.
25
ĐS: m ≥ 16 hoặc m < −
9
c) Phương trình có ít nhất một nghiệm thuộc (−1; 1).
2
ĐS: − < m < 0
3
d) Phương trình có ít nhất một nghiệm x < 2.
ĐS: m ≤ 0

BÀI 3. BÀI TẬP


Nguyễn Thành Nhân- Chuyên Hùng Vương- Bình Dương

A. BÀI TẬP
BÀI 1. Tìm tất cả các giá trị của tham số m để
LATEX:Nguyễn Thành Nhân

1 f ( x) = (3 m − 3) x2 − (3 m + 6) x + m − 3 luôn dương với mọi x.


ĐS: m > 28.
2 f ( x) = ( m − 4) x2 + (5 m − 20) x − 2 m − 1 luôn âm với mọi x.
32
ĐS: < m ≤ 4.
11
p
BÀI 2. Tìm tất cả các giá trị của tham số m để hàm số f ( x) = ( m + 4) x2 − ( m − 4) x − 2 m + 1 xác định với mọi x.
20
ĐS: − ≤ m ≤ 0.
9
BÀI 3. Chứng minh rằng phương trình 4 x2 + 4( m + 5) x + 16 m + 2 = 0 luôn có nghiệm với mọi m.
p p
BÀI 4. Chứng minh rằng với mọi m, phương trình x2 + (1 + m 3) x + m2 + m 3 + 2 = 0 vô nghiệm.
BÀI 5. Tìm tất cả các giá trị của tham số m để các biểu thức sau luôn dương.
1 2
f ( x) = x + 2( m − 2) x + m2 .
m
f ( x) = ( m − 2) x2 + 2( m − 2) x + m + 4.
BÀI 6. Tìm tất cả các giá trị của tham số m để các biểu thức sau luôn âm.
f ( x) = ( m − 2) x2 + 2 x − 4.
f ( x) = m( m + 8) x2 − 2( m + 8) x + 8 m + 1.
BÀI 7. Tìm m để bất phương trình (m − 1) x2 + 2(m + 1) x + 3 m − 6 > 0 nghiệm đúng với mọi x.
ĐS: m > 5
2
BÀI 8. Tìm m để bất phương trình mx + 6 mx + 8m − 10 ≥ 0 vô nghiệm.
ĐS: −10 < m ≤ 0.
2 2
BÀI 9. Chứng minh rằng 3 x − 8 x y + 9 y − 4 x − 2 y + 5 ≥ 0 với mọi x, y.
3
BÀI 10. Tìm m để hàm số f ( x) = p xác định với mọi x ∈ R.
( m + 1) x2 + 2 mx + 5 + 9 m
¯ 2 ¯
¯ 3 x − x + 12 ¯
BÀI 11. Tìm m để bất phương trình ¯ 2
¯ ¯ ≥ 2 có tập nghiệm là R.
x + mx + 4 ¯
5 3
ĐS: − ≤ m ≤ .
2 2
x2 + 5 x + m
BÀI 12. Tìm m để sao cho với mọi x ta có −1 ≤ < 7.
2 x2 − 3 x + 2
5
ĐS: − ≤ m < 1.
3
x2 + 2 mx + m2 − 1 ≤ 0
(
BÀI 13. Tìm m để hệ có nghiệm.
x2 + 8 x + 16 − m2 ≤ 0
BÀI 14. Định m để ∀ x ∈ [−1; 2], bất phương trình (m2 + 1) x − m( x + 3) + 1 > 0 luôn nghiệm đúng.

ÓNguyễn Thành Nhân 106 Chuyên Hùng Vương-BD


Các Bài Giảng 2023-2024 Đại Số 10 Chuyên

BÀI 4. PHƯƠNG TRÌNH ĐA THỨC BẬC CAO ĐẶC BIỆT


Nguyễn Thành Nhân- Chuyên Hùng Vương- Bình Dương

A. CƠ SỞ LÝ THUYẾT

1. Phương trình dạng hồi quy


Dạng phương trình
ax4 + bx3 + cx2 ± kbx + k2 a = 0 ( ka ̸= 0) (1)
Phương pháp giải

• Nhận xét x = 0 không là nghiệm.

• Chia hai vế cho x2 ̸= 0 ta được


k2 k
µ ¶ µ ¶
a x2 + 2 + b x ± + c = 0.
x x

k k2
µ ¶
Đặt t = x ± thì x2 + 2 = t2 ∓ 2k, phương trình trở thành bậc hai ẩn t.
x x

Chú ý 26. Ta xét hai trường hợp đặc biệt của số k đó là k = ±1.

• Nếu k = 1 ta có phương trình đối xứng

TOÁN CHUYÊN
ax4 + bx3 + cx2 + bx + a = 0. (2)

• Nếu k = −1 ta có phương trình phản xứng

ax4 + bx3 + cx2 − bx + a = 0. (3)

VÍ DỤ 1. Giải phương trình 2 x4 + 3 x3 − 16 x2 + 3 x + 2 = 0.


1 p
ĐS: x = 2; x = ; x = −2 ± 3.
2

VÍ DỤ 2. Giả sử a, b, c là các số thực sao cho đa thức

P ( x) = x4 + ax3 + bx2 + cx + 1

có ít nhất một nghiệm thực. Tìm tất cả các bộ (a; b; c) để a2 + b2 + c2 đạt giáµtrị nhỏ nhất.
2 2 2 2 2 2
¶ µ ¶
ĐS: − ; − ; − và ; − ; .
3 3 3 3 3 3

VÍ DỤ 3. Giải phương trình x4 + 4 = 5 x( x2 − 2). p


ĐS: x = −1; x = 2; x = 2 ± 6.

2. Phân tích thành các nhân tử bậc hai


Nguyên lý
Mọi phương trình đa thức bậc cao (bậc lớn hơn 2) luôn phân tích được thành ít nhất một nhân tử bậc hai.
Phương pháp phân tích
Dùng phương pháp hệ số bất định và kỹ thuật đồng nhất hệ số hai vế.

VÍ DỤ 4. Giải phương trình x4 + 4 x3 − 10 x2 + 37 x − 14 = 0.


ĐS: x4 + 4 x3 − 10 x2 + 37 x − 14 = ( x2 − 5 x + 2) · ( x2 + x − 7).

ÓNguyễn Thành Nhân 107 Chuyên Hùng Vương-BD


Các Bài Giảng 2023-2024 Đại Số 10 Chuyên

3. Phương trình lũy thừa bậc 4


Dạng phương trình
( x + a)4 + ( x + b)4 = c. (4)
Phương pháp giải
a+b
Đặt t = x + , biến đổi đưa về phương trình trùng phương ẩn t.
2

VÍ DỤ 5. Giải phương trình ( x + 1)4 + ( x − 5)4 = 162.


ĐS: x = 2, hãy tìm nghiệm còn lại.

4. Phương trình ghép cặp


a) Dạng phương trình
( x + a)( x + b)( x + c)( x + d ) = e, (5)
ở đó a + b = c + d .
Phương pháp giải
Ghép từng cặp ( x + a)( x + b) = x2 + (a + b) x + ab và ( x + c)( x + d ) = x2 + ( c + d ) x + cd , sau đó đặt t = x2 + (a + b) x =
x2 + ( c + d ) x.
b) Dạng phương trình
LATEX:Nguyễn Thành Nhân

( x + a)( x + b)( x + c)( x + d ) = ex2 , (6)


ở đó ab = cd .
Phương pháp giải

• Nhận xét x = 0 có nghiệm hay không?

• Chia hai vế cho x2 ̸= 0

[ x2 + (a + b) x + ab][ x2 + ( c + d ) x + cd ] = ex2
ab cd
⇔ [x + + (a + b)][ x + + ( c + d )] = e.
x x
ab cd
Đặt t = x + = t+ , đưa về phương trình bậc hai ẩn t.
x x

VÍ DỤ 6. Giải các phương trình


a) ( x + 4)( x + 5)( x + 7)( x + 8) = 4. p
ĐS: x = −6; x = −6 ± 5.

b) (5 x + 4)2 (5 x2 + 8 x) = 16.
c) ( x − 3)( x − 9)( x + 4)( x + 12) = 147 x2 .
ĐS: Giải như lý thuyết.

5. Phương trình đẳng cấp theo hai biến


Đây là dạng phương trình mà sau khi đổi biến ta đưa về

aX 2 + bX Y + cY 2 = 0. (7)

Để giải phương trình này ta thường thực hiện hai bước


• Kiểm tra Y = 0 có thỏa mãn phương trình hay không? Nếu thỏa thì những x làm cho Y = 0 là nghiệm.

• Chia hai vế cho Y 2 ̸= 0, ta được


¶2
X X
µ µ ¶
a +b + c = 0.
Y Y
X
Đây là phương trình bậc hai theo .
Y

ÓNguyễn Thành Nhân 108 Chuyên Hùng Vương-BD


Các Bài Giảng 2023-2024 Đại Số 10 Chuyên

VÍ DỤ 7. Giải phương trình 2( x2 − x + 1)2 + 5( x + 1)2 = 11( x3 + 1).

VÍ DỤ 8. Giải phương trình ( x2 + 2 x + 4)2 − 5( x3 − 8) + 6( x − 2)2 = 0.

¶2 ¶2
x−2 x+2 x2 − 4
µ µ µ ¶
VÍ DỤ 9. Giải phương trình 2 +2 −5 = 0.
x+1 x−1 x2 − 1

B. BÀI TẬP TỔNG HỢP


BÀI 1. Giải phương trình ( x2 + 4 x + 2)( x2 − 3 x + 2)2 + 36 x3 = 0.
BÀI 2. Giải phương trình x4 − 4 x3 + 8 x + 3 = 0.
BÀI 3. Giải phương trình (2 x − 1)(4 x + 5)(8 x + 3)(16 x − 15 = 99 x2 .
BÀI 4. Giải phương trình 4( x + 5)( x + 6)( x + 10)( x + 12) = 3 x2 .

MỘT SỐ BÀI GTLN-GTNN


BÀI 5. Cho các số thực a, b thỏa ab ̸= 0. Tìm giá trị nhỏ nhất của biểu thức

a2 b 2 a b
P= + − − + 1.
b 2 a2 b a

TOÁN CHUYÊN
BÀI 6. Cho các số thực x, y thỏa mãn x2 + y2 = 1 + x y. Tìm giá trị lớn nhất và giá trị nhỏ nhất của biểu thức

F = x4 + y4 − x2 y2 .

BÀI 7. Cho các số thực x, y thỏa mãn 2( x2 + y2 ) = 1 + x y. Tìm giá trị lớn nhất và giá trị nhỏ nhất của biểu thức

F = 7( x4 + y4 ) + 4 x2 y2 .

BÀI 8. Cho các số thực không âm x , y thỏa mãn x + y = 1. Tìm giá trị lớn nhất và giá trị nhỏ nhất của biểu
thức
Q = (3 x2 + 2 y) · (3 y2 + 2 x) + 5 x y.
BÀI 9. Cho phương trình x2 + bx + c = 0 có hai nghiêm dương x1 , x2 thỏa mãn x1 · x2 ≥ 1.

a) Chứng minh rằng b2 ≥ 4.


3 b2 − 4 c + b + 2
b) Tìm giá trị nhỏ nhất của biểu thức T = .
b2 + 1

BÀI 10. Giả sử phương trình ax2 + bx + c = 0 có hai nghiệm thuộc đoạn [0; 3]. Tìm giá trị lớn nhất và giá trị
nhỏ nhất của biểu thức
18a2 − 9ab + b2
P= .
9a2 − 3ab + ac

BÀI 5. BÀI TẬP TAM THỨC BẬC HAI


Nguyễn Thành Nhân- Chuyên Hùng Vương- Bình Dương

A. BÀI TẬP
BÀI 1. Cho phương trình x2 − 2(m + 1) x + 3m − 3 = 0. Tìm m để

1 Phương trình có hai nghiệm x1 , x2 và tìm hệ thức liên hệ giữa hai nghiệm không phụ thuộc m.
ĐS: 3( x1 + x2 ) − 2 x1 x2 = −12.
x12 (2 − x2 ) + x22 (2 − x1 )
2 Tìm GTLN và GTNN của biểu thức P = .
x12 + x22 + x1 x2 + 1

ÓNguyễn Thành Nhân 109 Chuyên Hùng Vương-BD


Các Bài Giảng 2023-2024 Đại Số 10 Chuyên

BÀI 2. Cho ba phương trình x2 + ax + 1 = 0 (1); x2 + bx + 1 = 0 (2); x2 + cx + 1 = 0 (3). Biết tích một nghiệm
của phương trình (1) với một nghiệm của phương trình (2) là một nghiệm của phương trình (3). Chứng minh
rằng
a2 + b2 + c2 + abc = 4. (4)
BÀI 3. Cho x và y là hai số thực thỏa mãn x2 + y2 = 1. Tìm GTLN và GTNN của biểu thức

2( x2 + 6 x y)
P= .
1 + 2 x y + 2 y2

BÀI 4. Cho phương trình mx2 − 2(m + 1) x + m − 3 = 0. Tìm m để phương trình có hai nghiệm phân biệt x1 , x2
thỏa mãn x12 + x22 = 22.
BÀI 5. Tìm tất cả các giá trị của tham số m để phương trình 2 x2 + 2mx − m − 1 = 0 có hai nghiệm phân biệt x1 ,
x2 sao cho biểu thức
2019 2019
P= 2
+ ,
(2 x1 − 1) (2 x2 − 1)2
đạt giá trị nhỏ nhất.
BÀI 6. Cho x và y là các số thực thỏa mãn x2 + 2 y2 − 2 x y = 1. Tìm giá trị lớn nhất và giá trị nhỏ nhất của biểu
thức
1 + x y − y2
P= .
1 + 3 x y − y2
LATEX:Nguyễn Thành Nhân

BÀI 7. Tìm tất cả các số nguyên dương a, b sao cho các phương trình x2 − 2ax − 3 b = 0 ; x2 − 2bx − 3a = 0 đều có
nghiệm nguyên dương.
ĐS: (a; b) ∈ {(1; 1); (16; 11); (11; 16)}.
p
BÀI 8. Giải phương trình x4 + ax3 + bx2 + 2ax + 4 = 0 (1) biết rằng 9(a2 + b2 ) = 16. ĐS: x = ± 2.
BÀI 9. Cho các số a, b, c dương và thỏa mãn a4 + b4 + c4 ≤ 2(a2 b2 + b2 c2 + c2 a2 ). Chứng minh rằng ít nhất một
trong các phương trình sau vô nghiệm

ax2 + 2 bx + 2 c = 0; bx2 + 2 cx + 2a = 0; cx2 + 2ax + 2 b = 0.

BÀI 10. Cho phương trình ax2 + bx + c = 0 (các hệ số a, b, c nguyên và a > 0). Biết rằng phương trình có hai
nghiệm dương phân biệt bé hơn 1. Tìm giá trị nhỏ nhất của hệ số a.
BÀI 11. Cho tam thức bậc hai f ( x) = x2 − 6 x + 12. Giải phương trình

f ( f ( f ( f ( x)))) = 65539.

. 111960
BÀI 12. Cho tam thức bậc hai f ( x) = 20 x2 −11 x+2016. Chứng minh rằng tồn tại số nguyên a sao cho f (a)..220 .
BÀI 13. Biết a, b là các số thực thay đổi sao cho tam thức f ( x) = x2 − 2ax + 2a2 + b2 − 5 có nghiệm. Hãy tìm giá
trị nhỏ nhất của biểu thức
P = (a + 1)( b + 1).

BÀI 14. Có tồn tại hay không cặp số nguyên dương (a; b) sao cho hai phương trình x2 + 2ax − b − 2 = 0 và
x2 + bx − a = 0 đồng thời có nghiệm nguyên.
BÀI 15 (Italia MO-THTT Tháng 4/2016). Cho a, b, c là ba số thực thuộc đoạn [0; 1]. Chứng minh rằng

a 2 + b 2 + c 2 ≤ a 2 b + b 2 c + c 2 a + 1.

BÀI 16. Cho tam thức tam thức f ( x) = a2 2016 x2 + bx + a2016 c − 1 với a, b, c là các số nguyên. Giả sử phương
f 2 (1) + f 2 (−1)
trình f x) = −2 có hai nghiệm nguyên dương. Chứng minh rằng khi đó A = là hợp số, ở đây
2
f 2 (a) = [ f (a)]2 .
BÀI 17 (Trường Đông Toán học Bắc Trung Bộ 2019 (Vinh)).
Cho số nguyên dương k. Giả sử P ( x) ∈ R[ x] là đa thức bậc hai có hai nghiệm thực phân biệt, đồng thời với mọi
số thực a, b thỏa mãn |a| , | b| ≥ k thì P (a2 + b2 ) ≥ P (2ab). Chứng minh rằng P ( x) có ít nhất một nghiệm âm.

BÀI 6. PHƯƠNG TRÌNH, BẤT PHƯƠNG TRÌNH CHỨA CĂN


Nguyễn Thành Nhân- Chuyên Hùng Vương- Bình Dương

ÓNguyễn Thành Nhân 110 Chuyên Hùng Vương-BD


Các Bài Giảng 2023-2024 Đại Số 10 Chuyên

A. CƠ SỞ LÝ THUYẾT

1. Một số phép biến đổi tương đương phương trình chứa căn
Ta xét một số dạng phương trình chứa căn sử dụng biến đổi tương đương

1 Phương trình dạng (


p p f ( x) = g ( x)
f ( x) = g ( x) ⇔
g ( x ) ≥ 0.

2 Phương trình dạng


f ( x) = g( x)2
(
p
f ( x) = g ( x) ⇔
g( x) ≥ 0.

3 Phương trình dạng




 f ( x) ≥ 0
p p p 
f ( x) + g ( x) = h( x) ⇔ g ( x) ≥ 0

 p
h ( x ) = f ( x ) + g ( x ) + 2 f ( x ) · g ( x ).

VÍ DỤ 1. Giải các phương trình


p p p p
1 2 x − 1 − x − 4 = 3 x − 11. 2 x2 + x2 + 11 = 11.

TOÁN CHUYÊN
p p p p
3 x+ x − 1 − x = 1. 4 x2 − 3 x + 2 x − 1 + 1 = 0.
p 2p p p
5 ( x + 3) x2 + 11 = x2 + 3 x + 11. 6 1+ x − x2 = x + 1 − x.
3
p p p p p p
7 x( x − 1) + x( x + 2) = 2 x2 . 8 x2 + 3 x + 2 + x2 + 5 x + 6 = 2 x2 + 9 x + 7.
p p p p p p p p
9 x − 2 x − 1 + x + 6 x − 9 = 5. 10 x + 3 + 3 x + 1 = 2 x + 2 x + 2.

VÍ DỤ 2. Giải các phương trình


p p p p p x2
1 x 2 − 1 + x 2 + x − 2 = 2 x 2 − 3 x + 2. 2 1− x+ 1+ x = 2− .
4
p p p p
3 x2 + x + 2 = 2. 4 x + 2 7 − x = 2 x − 1 + − x2 + 8 x − 7.
p ¢2 p p p
6 2 x − 5 + 5 x − 1 + 2 x + 2 = 4 x2 + x − 2.
¡p
5 x 2 x + 1 − 2 x + 17 = 16.
s
p p p x3 + 1 p p p
7 x + 3 + 2 x x + 1 = 2 x + x 2 + 4 x + 3. 8 + x + 1 = x 2 − x + 1 + x + 3.
x+3

VÍ DỤ 3. Tìm m để phương trình p


2 x2 − mx + 2 m − 1 − x = 2
có hai nghiệm phân biệt.

VÍ DỤ 4. Chứng minh rằng với mọi giá trị dương của tham số m thì phương trình

x2 + 2 x − 8 =
p
m( x − 2)

có hai nghiệm phân biệt.

VÍ DỤ 5. Giải các phương trình

ÓNguyễn Thành Nhân 111 Chuyên Hùng Vương-BD


Các Bài Giảng 2023-2024 Đại Số 10 Chuyên

p p p p p
1 x + x + 11 + x − x + 11 = 4. 2 3 x − 2(2 x − 1) + x − 2 = 0.
p p p p p p x+5
3 2 x + 2 + 2 x + 1 − x + 1 = 4. 4 x+2+2 x+1+ x+2−2 x+1 = .
2
p p p p
5 (4 x − 11) x2 + 1 = 2 x2 + 2 x + 1. 6 2 x2 + 6 x + 11 + x2 − x + 4 = x2 + 15 x + 9.

2. Một số phép biến đổi tương đương bất phương trình chứa căn
Ta xét một số dạng bất phương trình chứa căn sử dụng biến đổi tương đương

1 Dạng bất phương trình


(
g ( x) < 0
 f ( x) ≥ 0

p
f ( x) ≥ g ( x) ⇔ 
 ( g ( x) ≥ 0

f ( x) ≥ g( x)2 .

2 Dạng bất phương trình


f ( x) ≥ 0

LATEX:Nguyễn Thành Nhân



p
f ( x) ≤ g ( x) ⇔ g ( x) ≥ 0

f ( x) ≤ g( x)2 .

3 Dạng bất phương trình


g ( x) ≥ 0



p p p
f ( x) ≥ g ( x) + h( x) ⇔ h( x) ≥ 0

f ( x) ≥ ( g( x) + h( x))2 .

4 Dạng bất phương trình




 f ( x) ≥ 0

p p p  g ( x) ≥ 0

f ( x) ≤ g ( x) + h( x) ⇔

 h( x) ≥ 0


f ( x) ≤ ( g( x) + h( x))2 .

3. Phương trình, bất phương trình chứa căn nâng cao


BÀI 1 (Đề Duyên Hải Bắc Bộ 2017-Khối 10). Giải phương trình
p p
x2 + 2 2 x + 7 = 2 3 − 2 x + 5( x ∈ R).

BÀI 2 (Chuyên Hùng Vương, Phú Thọ DHBB 2019).


Giải phương trình ³ p ´
1 + 3 x = ( x − x2 ) · 5 + 15 + 6 x − 9 x2 .
p
BÀI 3. Giải phương trình 2 x2 − 6 x − 1 = 4 x + 5
p p
BÀI 4. Giải phương trình x + x + 3 + x3 + x − 5 = 0.
p
BÀI 5. Giải bất phương trình x3 − 3 x2 + 2 ( x + 2)3 − 6 x ≥ 0 ( x ∈ R).
p p
BÀI 6. Giải phương trình ( x − 3) 1 + x − x 4 − x = 2 x2 − 6 x − 3.
p
BÀI 7. Giải bất phương trình x3 + (3 x2 − 4 x − 4) x + 1 ≤ 0.
p 4
BÀI 8. Giải phương trình 3 81 x − 8 = x3 − 2 x2 + x − 2; ( x ∈ R).
3
x+2 1
BÀI 9. Giải bất phương trình q ¡ ≥ .
4 2
2 x − x +1 −1
¢ x − 1
p p
BÀI 10. Giải phương trình 2 2 x − 5 + 2 3 x − 5 = x2 − 8 x + 21.
p p hp p i p
BÀI 11. Giải phương trình 1 + 1 − x2 (1 + x)3 − (1 − x)3 = 2 + 1 − x2 .

ÓNguyễn Thành Nhân 112 Chuyên Hùng Vương-BD


Các Bài Giảng 2023-2024 Đại Số 10 Chuyên

BÀI 12. Giải bất phương trình p


x2 − 2 x + 2 x2 − 2 x − 2 − 5 ≥ 0.
BÀI 13. Giải phương trình p p3
5 x − 1 + 9 − x = 2 x 2 + 3 x − 1.
1 1
BÀI 14. Giải bất phương trình p p ≤p .
x+2− 3− x 5 − 2x
p p ³ p ´
BÀI 15. Giải phương trình ( x + 3 − x + 1) x2 + x2 + 4 x + 3 = 2 x.

BÀI 16. Giải bất phương trình x2 − 3 x + 2 x2 − 12 x + 32 ≤ 4 x2 .


¡ ¢¡ ¢

BÀI 17. Giải các bất phương trình sau:


3 4 x2 − 9
¡ ¢
p
2
a) x − 4 x − 6 ≥ 2 x2 − 8 x + 12. b) p ≤ 2 x + 3.
3 x2 − 3
BÀI 18. Giải phương trình:
2x 13 x
+ = 6.
2 x2 − 5 x + 3 2 x2 + x + 3
p p
BÀI 19. Giải phương trình 3 5 − x + 3 5 x − 4 = 2 x + 7.
p p p
BÀI 20. Giải bất phương trình 11 x2 − 19 x − 19 − x2 − x − 6 < 2 2 x + 1.
³p ´p
BÀI 21. Giải bất phương trình sau trên tập số thực 2 x2 + 4 − x − 2 x2 − 5 x + 6 ≥ 0.
p p
2+ x 2− x 3x − 2
BÀI 22. Giải phương trình: p − =p .
2 2 2 9 x2 + 16

TOÁN CHUYÊN
BÀI 7. TỔNG HỢP VỀ PHƯƠNG TRÌNH VÔ TỈ
Nguyễn Thành Nhân- Chuyên Hùng Vương- Bình Dương

CÁC BÀI TOÁN


p p
BÀI 1. Giải phương trình x2 + 2 2 x + 7 = 2 −2 x + 3 + 5.
p p p
BÀI 2. Giải phương trình 5 x2 + 14 x + 9 − x2 − x − 20 = 5 x + 1.
BÀI 3. Giải phương trình ³ p ´
1 + 3 x = ( x − x2 ) 5 + 15 + 6 x − 9 x2 .

BÀI 4. Giải phương trình


p p p p p p
2x − 1 = 2 − x 10 − 4 x + 5 − 2 x 6 − 2 x + 2 3 − x 2 − x.
p p
BÀI 5. Giải phương trình x2 + 2 2 x + 7 = 2 3 − 2 x + 5( x ∈ R).
BÀI 6 (T5/479-Toán học & Tuổi trẻ, tháng 5 năm 2017).
Giải phương trình p
x2 + 28 x + 4 x+4
+8 = p + 2 x. (1)
x+2 x−1
BÀI 7 (T6/480-Toán học & Tuổi trẻ, tháng 6 năm 2017).
Giải phương trình p
4 x3 − 24 x2 + 45 x − 26 = − x 2 + 4 x − 3.
BÀI 8 (T5/481, Toán học & tuổi trẻ, tháng 7 năm 2017).
Giải phương trình p
(1 − 2 x) 2 − x2 = x − 1.
BÀI 9 (T6/482-Toán học & Tuổi trẻ, tháng 8 năm 2017).
Giải phương trình
1 1 1 1 1
p
3
+p
3
=p
3
+p
3
với x > .
x 3x + 1 2x − 1 2x + 2 2
BÀI 10 (Bài toán P232,
p Tạp chí Piptháng 11 năm 2018).
3
Giải phương trình x2 − 1 + x = x3 − 2.

ÓNguyễn Thành Nhân 113 Chuyên Hùng Vương-BD


Các Bài Giảng 2023-2024 Đại Số 10 Chuyên

BÀI 11 (Bài toán P325, Tạp chí Pi tháng 8 năm 2019).


Giải phương trình: p p p
3
2x − 1 + x − 1 = 3 x − 2.
BÀI 12 (T3/509 Toán học & tuổi trẻ số 509, tháng 11 năm 2019).
Giải phương trình
1 1 1 1 4
+ + + = với x > 0.
5 x2 − x + 3 5 x2 + x + 7 5 x2 + 3 x + 13 5 x2 + 5 x + 21 x2 + 6 x + 5

BÀI 13 (T5/506 Toánrhọc & tuổi trẻ số 506, tháng 8 năm 2019).
x−1 2x + 6
Giải phương trình + = 2.
x + 1 ¡p x − 1 + p x + 3¢2

BÀI 14 (T6/505 Toán học & tuổi trẻ số 505, tháng 7 năm 2019).
Tìm tất cả các nghiệm thực của phương trình
s p s p
3 x3 − 3 x + ( x2 − 1) x2 − 4 3 x3 − 3 x − ( x2 − 1) x2 − 4
+ = x2 − 2.
2 2

BÀI 15 (T7/505 Toán học & tuổi trẻ số 505, tháng 7 năm 2019).
Cho phương trình
1 5
x + 2 x4 − 5 x3 − 7 x2 + 12 x − 1 = 0.
LATEX:Nguyễn Thành Nhân

(1)
3
a) Chứng minh (1) luôn có 5 nghiệm phân biệt.
5
X xi − 1
b) Giả sử x i ( i = 1, 5) là các nghiệm của (1). Tính tổng S = 5 4
.
i =1 x i + 6 x i − 3

BÀI 16 (T5/502 Toán học & tuổi


p trẻ số 502, tháng 4 năm 2019).
Giải phương trình x6 − 7 x2 + 6 = 0.

BÀI 1. BÀI TẬP TAM THỨC BẬC HAI


Nguyễn Thành Nhân- Chuyên Hùng Vương- Bình Dương

A. BÀI TẬP
BÀI 1. Cho phương trình x2 − 2(m + 1) x + 3 m − 3 = 0. Tìm m để
1 Phương trình có hai nghiệm x1 , x2 và tìm hệ thức liên hệ giữa hai nghiệm không phụ thuộc m.
ĐS: 3( x1 + x2 ) − 2 x1 x2 = −12.
x12 (2 − x2 ) + x22 (2 − x1 )
2 Tìm GTLN và GTNN của biểu thức P = .
x12 + x22 + x1 x2 + 1

BÀI 2. Cho ba phương trình x2 + ax + 1 = 0 (1); x2 + bx + 1 = 0 (2); x2 + cx + 1 = 0 (3). Biết tích một nghiệm
của phương trình (1) với một nghiệm của phương trình (2) là một nghiệm của phương trình (3). Chứng minh
rằng
a2 + b2 + c2 + abc = 4. (4)
BÀI 3. Cho x và y là hai số thực thỏa mãn x2 + y2 = 1. Tìm GTLN và GTNN của biểu thức

2( x2 + 6 x y)
P= .
1 + 2 x y + 2 y2

BÀI 4. Cho phương trình mx2 − 2(m + 1) x + m − 3 = 0. Tìm m để phương trình có hai nghiệm phân biệt x1 , x2
thỏa mãn x12 + x22 = 22.
BÀI 5. Tìm tất cả các giá trị của tham số m để phương trình 2 x2 + 2mx − m − 1 = 0 có hai nghiệm phân biệt x1 ,
x2 sao cho biểu thức
2019 2019
P= 2
+ ,
(2 x1 − 1) (2 x2 − 1)2
đạt giá trị nhỏ nhất.

ÓNguyễn Thành Nhân 114 Chuyên Hùng Vương-BD


Các Bài Giảng 2023-2024 Đại Số 10 Chuyên

BÀI 6. Cho x và y là các số thực thỏa mãn x2 + 2 y2 − 2 x y = 1. Tìm giá trị lớn nhất và giá trị nhỏ nhất của biểu
thức
1 + x y − y2
P= .
1 + 3 x y − y2
BÀI 7. Tìm tất cả các số nguyên dương a, b sao cho các phương trình x2 − 2ax − 3b = 0 ; x2 − 2 bx − 3a = 0 đều có
nghiệm nguyên dương.
ĐS: (a; b) ∈ {(1; 1); (16; 11); (11; 16)}.
p
BÀI 8. Giải phương trình x4 + ax3 + bx2 + 2ax + 4 = 0 (1) biết rằng 9(a2 + b2 ) = 16. ĐS: x = ± 2.
BÀI 9. Cho các số a, b, c dương và thỏa mãn a4 + b4 + c4 ≤ 2(a2 b2 + b2 c2 + c2 a2 ). Chứng minh rằng ít nhất một
trong các phương trình sau vô nghiệm

ax2 + 2 bx + 2 c = 0; bx2 + 2 cx + 2a = 0; cx2 + 2ax + 2 b = 0.

BÀI 10. Cho phương trình ax2 + bx + c = 0 (các hệ số a, b, c nguyên và a > 0). Biết rằng phương trình có hai
nghiệm dương phân biệt bé hơn 1. Tìm giá trị nhỏ nhất của hệ số a.
BÀI 11. Cho tam thức bậc hai f ( x) = x2 − 6 x + 12. Giải phương trình

f ( f ( f ( f ( x)))) = 65539.

. 111960
BÀI 12. Cho tam thức bậc hai f ( x) = 20 x2 −11 x+2016. Chứng minh rằng tồn tại số nguyên a sao cho f (a)..220 .
BÀI 13. Biết a, b là các số thực thay đổi sao cho tam thức f ( x) = x2 − 2ax + 2a2 + b2 − 5 có nghiệm. Hãy tìm giá
trị nhỏ nhất của biểu thức
P = (a + 1)( b + 1).
BÀI 14. Có tồn tại hay không cặp số nguyên dương (a; b) sao cho hai phương trình x2 + 2ax − b − 2 = 0 và

TOÁN CHUYÊN
x2 + bx − a = 0 đồng thời có nghiệm nguyên.
BÀI 15 (Italia MO-THTT Tháng 4/2016). Cho a, b, c là ba số thực thuộc đoạn [0; 1]. Chứng minh rằng

a2 + b2 + c2 ≤ a2 b + b2 c + c2 a + 1.

BÀI 16. Cho tam thức tam thức f ( x) = a2 2016 x2 + bx + a2016 c − 1 với a, b, c là các số nguyên. Giả sử phương
f 2 (1) + f 2 (−1)
trình f x) = −2 có hai nghiệm nguyên dương. Chứng minh rằng khi đó A = là hợp số, ở đây
2
f 2 (a) = [ f (a)]2 .
BÀI 17 (Trường Đông Toán học Bắc Trung Bộ 2019 (Vinh)).
Cho số nguyên dương k. Giả sử P ( x) ∈ R[ x] là đa thức bậc hai có hai nghiệm thực phân biệt, đồng thời với mọi
số thực a, b thỏa mãn |a| , | b| ≥ k thì P (a2 + b2 ) ≥ P (2ab). Chứng minh rằng P ( x) có ít nhất một nghiệm âm.

BÀI 2. HỆ PHƯƠNG TRÌNH BẬC NHẤT HAI ẨN, BA ẨN


Nguyễn Thành Nhân- Chuyên Hùng Vương- Bình Dương

A. CƠ SỞ LÝ THUYẾT

1. Hệ hai phương trình bậc nhất hai ẩn


Định nghĩa 1. Hệ phương trình bậc nhất hai ẩn là hệ phương trình có dạng
(
ax + b y = c
(I)
a′ x + b ′ y = c ′ .

trong đó a, b, c , d ∈ R đồng thời a và b, a′ và b′ là những số thực không đồng thời bằng 0.


Phương pháp giải
Ở lớp 9 ta đã biết cách giải hệ bằng phương pháp thế, phương pháp cộng đại số, phương pháp biến đổi tương
đương, phương pháp dùng đồ thị. Trong bài học này chúng ta sẽ dùng một phương pháp mới đó là phương
pháp sử dụng định thức Cramer.
¯ ¯
¯ a b ¯¯
• Định thức cấp hai là một số, được ký hiệu ¯¯ = ad − bc.
c d ¯

ÓNguyễn Thành Nhân 115 Chuyên Hùng Vương-BD


Các Bài Giảng 2023-2024 Đại Số 10 Chuyên

• Giải hệ bằng định thức


Ta lần lượt ký hiệu ¯ ¯ ¯ ¯ ¯ ¯
¯ a b ¯¯ ¯ c b ¯¯ ¯ a c ¯¯
D = ¯¯ ′ ′ ¯ ; D x = ¯
¯ c′ ′ ¯ ; D y¯
¯ ,
a b b a′ c′ ¯
khi đó
Dx

x =


D
+ Nếu D ̸= 0 thì hệ ( I ) có nghiệm duy nhất
 Dy
y =

.
D
+ Nếu D = 0 và có ít nhất D x ̸= 0 hoăc D y ̸= 0 thì hệ phương trình vô nghiệm.
+ Nếu D = D x = D y = 0 thì hệ phương trình có vô số nghiệm dạng
x∈R y∈R
( (
hoặc là
ax + b y = c. ax + b y = c.

Chú ý 27. Vui một chút


Để nhớ lâu hơn cách lập các định thức, ta dùng câu sau với các chữ cái đầu ứng với dòng trên các định thức
D, D x, D y.
“anh bạn- cầm bát- ăn -cơm ”.

VÍ DỤ 1. Giải và biện luận hệ phương trình theo tham số m


LATEX:Nguyễn Thành Nhân

( ( (
mx + y = m + 1 3x − m y = 1 ( m − 1) x − y = m + 2
1 2 3
x + m y = 1. − mx + 3 y = m − 4. ( m + 1) x + 2 y = m − 5.

VÍ DỤ 2. Tìm m để hệ phương trình có nghiệm duy nhất


(
( m − 1) x − y = m + 2
( m + 1) x + 2 y = m − 5.

VÍ DỤ 3. Tìm m để hệ phương trình có nghiệm nguyên


(
mx + 2 y = m
( m − 1) x + ( m − 1) y = 1.

VÍ DỤ 4. Tìm tất cả các cặp số nguyên (a; b) để hệ phương trình


(
ax + y = 2
6x + b y = 4

vô nghiệm.

(
x + my = 1
VÍ DỤ 5. Cho hệ phương trình:
mx − y = − m.

a) Chứng minh rằng với mọi giá trị của m hệ phương trình đã cho luôn có nghiệm duy nhất.

b) Tìm các giá trị của m để hệ phương trình có nghiệm ( x; y) thỏa mãn x < 1, y < 1.
c) Tìm hệ thức liên hệ giữa x, y không phụ thuộc vào m.

2. Hệ ba phương trình bậc nhất ba ẩn


Định nghĩa 2. Hệ ba phương trình bậc nhất ba ẩn là hệ phương trình có dạng

a 1 x + b 1 y + c 1 z = d 1

a2 x + b2 y + c2 z = d2 (II)

a3 x + b3 y + c3 z = d3 .

ÓNguyễn Thành Nhân 116 Chuyên Hùng Vương-BD


Các Bài Giảng 2023-2024 Đại Số 10 Chuyên

trong đó a i , b i , c i ,d i ∈ R, i = 1; 3 đồng thời a i và b i , i = 1; 3 là những số thực không đồng thời bằng 0.


Phương pháp giải
Một phương pháp giải điển hình của hệ ba phương trình bậc nhất ba ẩn là sử dụng phương pháp khử Gauss
(nhà toán học người Đức, được mệnh danh là ông vua Toán học) để đưa hệ về dạng tam giác, sau đó sử dụng
phép thế.
Dạng tam giác của hệ là dạng

 A1 x = D1

A 2 x + B2 y = D 2 (III)

A 3 x + B3 y + C 3 z = D 3 .

B. BÀI TẬP

 mx + y = 1

BÀI 1. Tìm m để hệ x + m y = 1 có nghiệm. ĐS: m = 1, m = −2.

x+ y= m

BÀI 2. Giả và biện luận các hệ phương trình


( (
(a + b) x + (a − b) y = a x sin 2α + y(1 + cos 2α) = sin 2α
1 2
(2a + b) y + (2a − b) x = b. x(1 + cos 2α) − y sin 2α = 0.

BÀI 3. Tùy theo a hãy tìm giá trị nhỏ nhất của biểu thức

1 F = ( x − 2 y + 1)2 + (2 x + a y + 5)2 .

TOÁN CHUYÊN
2 G = (3 x + y − 2)2 + (6 x − 2a y + 5a − 1)2 + 2019.

3 K = ( x − 2a + 1)2 + (ax − 2 y + a − 12 + 2020.

bx − y = ac2
(
BÀI 4. Cho hệ phương trình
( b − 6) x + 2 b y = c + 1.

1 Tìm a sao cho với mọi b luôn tồn tại c để hệ có nghiệm.

2 Tìm a sao cho tồn tại c để hệ có nghiệm với mọi b.



ax + b y = c

BÀI 5. Giả sử hệ bx + c y = a có nghiệm. Chứng minh rằng

cx + a y = b

a3 + b3 + c3 = 3abc.
(
ax + y = b
BÀI 6. Cho hệ phương trình Tìm b để với mọi a ta luôn tìm được c để hệ có nghiệm.
x + a y = c2 + c.
(
( m + 3) x + 4 y = 5a + 3 b + m
BÀI 7. Tìm a và b để hệ có nghiệm với mọi m.
x + m y = ma − 2 b + 2 m − 1

BÀI 8. Cho a, b, c là độ dài ba cạnh của một tam giác. Chứng minh rằng hệ phương trình

a y + bx = c

cx + az = b

bz + c y = a

có nghiệm là x = cos A , y = cos B, z = cos C .


(
(2 m + 1) x − 3 y = 3 m − 2
BÀI 9. Cho hệ
( m + 3) x − ( m + 1) y = 2 m.

1 Tìm m để hệ có nghiệm.

2 Tìm m để hệ có nghiệm duy nhất x ≥ 2 y.

3 Tìm m để hệ có nghiệm duy nhất ( x; y) thỏa mãn P = x2 + 3 y2 đạt giá trị nhỏ nhất.

ÓNguyễn Thành Nhân 117 Chuyên Hùng Vương-BD


Các Bài Giảng 2023-2024 Đại Số 10 Chuyên

(
( m + 1) x + m y = 2 m − 1
BÀI 10. Cho hệ
mx − y = m2 − 2.

1 Tìm m để hệ có nghiệm duy nhất.

2 Khi hệ có nghiệm duy nhất, tìm hệ thức độc lập giữa hai nghiệm đối với tham số m.

3 Khi hệ có nghiệm duy nhất, tìm giá trị lớn nhất của tích x y

BÀI 11. Tìm m, n, p sao cho cả ba hệ phương trình


( ( (
x− py = n − px + y = m nx + m y = 1
; ;
− px + y = m nx + m y = 1 x− py = n

đồng thời vô nghiệm.


(
ax + b y = m
BÀI 12. Giả a, b, c, d là bốn số nguyên sao cho hệ phương trình luôn có nghiệm nguyên với mọi
cx + d y = n
tham số nguyên m, n. Chứng minh rằng
|ad − bc| = 1.

3. HỆ PHƯƠNG TRÌNH BẬC CAO MẪU MỰC


LATEX:Nguyễn Thành Nhân

BÀI
Nguyễn Thành Nhân- Chuyên Hùng Vương- Bình Dương

A. CƠ SỞ LÝ THUYẾT

1. Hệ đối xứng kiểu I


Định nghĩa 1. Hệ đối xứng hai ẩn kiểu I là hệ phương trình có dạng
(
f ( x ; y) = 0
, (I)
g( x; y) = 0

trong đó f ( x; y) và g( x; y) là những phương trình đối xứng, tức là f ( x; y) = f ( y; x) và g( x; y) = g( y; x).

PHƯƠNG PHÁP GIẢI


Để giải hệ đối xứng hai ẩn kiểu I ta thường sử dụng phương pháp sau đây

• Đặt S = x + y và P = x y, để hệ có nghiệm thì S 2 ≥ 4P (∗).

• Biểu diễn hệ về qua S và P . Giải hệ theo S , P .

• Với mỗi bộ (S ; P ) thỏa (∗) thì x; y là nghiệm của phương trình

t 2 − S · t + P = 0. (1)

• Do tính đối xứng nên nếu hệ có nghiệm ( x0 ; y0 ) thì hệ cũng có nghiệm ( y0 ; x0 ). Do đó để hệ có nghiệm
duy nhất thì điều kiện cần là x0 = y0 .

MỘT SỐ HẰNG ĐẲNG THỨC THƯỜNG DÙNG


Sau đây là một số hẳng đẳng thức thường sử dụng để giải hệ đối xứng hai ẩn kiểu I

1 x 2 + y2 = S 2 − 2 P . 2 x3 + y3 = S 3 − 3SP .
¢2
3 x 4 + y4 = S 2 − 2 P − 2 P 2 . 4 ( x − y)2 = S 2 − 4P .
¡

VÍ DỤ 1. Giải các hệ phương trình

ÓNguyễn Thành Nhân 118 Chuyên Hùng Vương-BD


Các Bài Giảng 2023-2024 Đại Số 10 Chuyên

 1 1
x + y + x + y = 4  x2 + y2 + p2 x y = 8p2
q
2 2
( 
x y + x y = 30

1 2 3
x3 + y3 = 35. 1 1 p p
 x 2 + y2 + x+ y = 4.

 + .
x2 y2
1
 µ ¶
r r
 ( x + y) 1 + =5  x + y = p 7p + 1
( 
x y( x − y) = −2

 xy
4 5 6 y x x y
x3 − y3 = 2. 1
µ ¶
 ( x 2 + y2 ) 1 +

= 49.  p
 p
x x y + y x y = 78.

x 2 y2

VÍ DỤ 2. Giải các hệ phương trình

x2 + x + y2 + y = 18 x6 + y6 = 2
( ( (
x+ y=4
1 2 3
x y( x + 1)( y + 1) = 72. 3 3
x − 3 x = y − 3 y. ( x2 + y2 )( x3 + y3 ) = 280.

 4 y3 − 1 + p x = 3
q
x4 + y4 + 6 x2 y2 = 41 ( x2 + y2 )(1 + x2 y2 ) = 280 x2 y2
( (
4 5 6
x y( x2 + y2 ) = 10. x y(1 + x y) = 18 x y. 
x2 + y3 = 82.

VÍ DỤ 3. Tìm m để các hệ phương trình sau có nghiệm


( (p p (
x + y+ xy = m x+ y=1 x + y+ xy = m
1 2 2
2 p p 3
x +y =. x x + y y = 1 − 3 m. x2 y + x y2 = 3 m − 9.

TOÁN CHUYÊN
x2 + y2 + 4 x + 4 y = 10
( ( (p p
x + xy+ y = m x+1+ y−1 = m
4 5 2 2
6 2
x y( x + 4)( y + 4) = m. x y + x y = 3 m − 8. x + y = m − 4 m + 6.

VÍ DỤ 4. Tìm m để
x 2 + x y + y2 = m + 6
(
1 Hệ có nghiệm duy nhất. ĐS: m = −3, m = 21.
2x + x y + 2 y = m
(
x + xy+ y = m +1
2 Hệ có nghiệm duy nhất x > 0, y > 0.
x2 y + x y2 = m

x2 + y2 = 2( m + 1)
(
3 Hệ có hai nghiệm phân biệt.
( x + y)2 = 4
(
x + xy+ y = m +2 3
4 Hệ 2 2
có nghiệm duy nhất. ĐS: m = − , m = 1.
x y+ xy = m +1 4
(
x+ y= m
5 Hệ có nghiệm ( x; y). Tìm giá trị lớn nhất của biểu thức
x 2 + y2 = 6 − m 2

F = 2 x y + 4( x + y) + 1.

x+ y= m



6 Hệ x2 + y2 = m2 − 4 m + 6 có nghiệm ( x; y) và tìm giá trị lớn nhất và giá trị nhỏ nhất (nếu có) của


x ≥ 0; y ≥ 0; 0 ≤ m ≤ 2
biểu thức
T = ( x + y)3 + 6 x y( x + y) + 39 m + 2.

p p
VÍ DỤ 5. Cho các số thực x, y thỏa mãn x − 3 y+2 = 3 y + 1 − y. Tìm giá trị lớn nhất
p và giá trịpnhỏ
nhất của biểu thức A = x + y. ĐS: 9 − 3 15 A ≤ 9 + 3 15.

ÓNguyễn Thành Nhân 119 Chuyên Hùng Vương-BD


Các Bài Giảng 2023-2024 Đại Số 10 Chuyên

x 2 + y2 + z 2 = 8
(
VÍ DỤ 6. Cho x, y, z là các số thực thỏa mãn . Chứng minh rằng
x y + yz + zx = 4

8 8
− ≤ x, y, z ≤ .
3 3

VÍ DỤ 7. Cho x, y là các số thực khác 0 và x y( x + y) = x2 + y2 − x y. Tìm giá trị lớn nhất của biểu thức
1 1
P= 3
+ 3.
x y

ĐS: P ≤ 16.

VÍ DỤ 8. Giải các phương trình sau bằng cách sử dụng hệ đối xứng kiểu I
p
3
³ p
3
´ p p ¢2
1 x 35 − x3 x + 35 − x3 = 30. 33 − 2 x − x2 = 2 − x + 1 .
¡
2
LATEX:Nguyễn Thành Nhân

2. Hệ đối xứng kiểu II


Định nghĩa 2. Hệ đối xứng hai ẩn kiểu II là hệ phương trình có dạng
(
f ( x ; y) = 0
, (II)
g( x; y) = 0

trong đó f ( x; y) = g( y; x) và f ( y; x) = g( x; y).

PHƯƠNG PHÁP GIẢI


+ Để giải hệ đối xứng hai ẩn kiểu II ta thường trừ vế theo vế làm xuất hiện nhân tử chung x − y.
+ Do tính đối xứng nên nếu hệ có nghiệm ( x0 ; y0 ) thì hệ cũng có nghiệm ( y0 ; x0 ). Do đó để hệ có nghiệm duy
nhất thì điều kiện cần là x0 = y0 .

VÍ DỤ 9. Giải các hệ phương trình


 1 3 
3
2 x + y = x 2 x + y = 2 x3 + 1 = 2 y

 
 (
x
1 2 3
1 3 3 y3 + 1 = 2 x.
2 y + x = 2 .
 
2 y + = .
 
x y y
(p p (p p p (p p
x+ y+9 = 9 x+ 2− y = 2 x+5+ y−2 = 7
4 p p 5 p p p 6 p p
y + x + 9 = 9. y + 2 − x = x. y + 5 + x − 2 = 7.

VÍ DỤ 10. Giải các hệ phương trình

( x − 1)( y2 + 6) = y( x2 + 1) (5 x − 4 y)(3 x + 2 y)2 = 7 y − 2 x + 20


( (
1 2
( y − 1)( x2 + 6) = x( y2 + 1). (5 y − 4 x)(3 y + 2 x)2 = 7 x − 2 y + 20.

x3 + 1 = 2( x2 − x + y) x2 − x y = 3(2 + y)
( (
3 4
y3 + 1 = 2( y2 − y + x). y2 − yx = 3(2 + x).

VÍ DỤ 11. Tìm m để hệ phương trình có nghiệm


( p
2x + y − 1 = m
1 p ĐS: m ≥ 2.
2 y + x − 1 = m.

ÓNguyễn Thành Nhân 120 Chuyên Hùng Vương-BD


Các Bài Giảng 2023-2024 Đại Số 10 Chuyên

( p
x+ y−3 = m
2 p ĐS: m ≥ 2.
y + x − 3 = m.
(p p p
x+1+ y−2 = m
3 p p p ĐS: m ≥ 3.
y + 1 + x − 2 = m.

VÍ DỤ 12. Giải các phương trình sau bằng cách sử dụng hệ đối xứng kiểu II
p p
3
p
3
1 x2 + x + 5 = 5. 2 x3 − 3 3 x + 2 = 2. 3 x 3 + 1 = 2 2 x − 1.

VÍ DỤ 13. Tìm m để hệ có nghiệm duy nhất

x = y2 − y + m
(
1 ĐS: m = 1.
y = x2 − x + m.
( 2
3 x = y3 − 2 y2 + m y 25
2 2 3 2
ĐS: m > .
3 y = x − 2 x + mx. 4
( 2
y = x3 − 4 x2 + mx 25
3 2 3 2
ĐS: m > .
x = y − 4 y + m y. 4

TOÁN CHUYÊN
3. Hệ đẳng cấp
Sau đây ta xét một số hệ phương trình đẳng cấp và phương pháp giải tương ứng
a 1 x2 + b 1 x y + c 1 y2 = d 1
(
1 Hệ có dạng (I I I )
a 2 x2 + b 2 x y + c 2 y2 = d 2 .
Phương pháp giải

• Khử hệ số tự do đưa về phương trình đẳng cấp.


• Một cách khác là xét y = 0. Với y ̸= 0 thì đặt x = t y.

a 1 x2 + b 1 x y + c 1 y2 + d 1 x + e 1 y = 0
(
2 Hệ có dạng ( IV )
a 2 x2 + b 2 x y + c 2 y2 + d 2 x + e 2 y = 0.
Phương pháp giải

• Xét x = 0 (hoặc y = 0) có thỏa hệ không.


• Nếu xét x = 0 thì đặt y = tx, thế vào hệ, giản ước, đi đến
d1 + e 1 t d2 + e 2 t
= .
a 1 + b 1 t2 + c 1 t a 2 + b 2 t2 + c 2 t
Giải phương trình tìm được t.
a 1 x2 + b 1 x y + c 1 y2 + d 1 x + e 1 y + f 1 = 0
(
3 Hệ có dạng (V )
a 2 x2 + b 2 x y + c 2 y2 + d 2 x + e 2 y + f 2 = 0.
Phương pháp giải
Tư tưởng chính là ta khử hệ số tự do đưa về hệ trên. Do đó ta sử dụng phép đổi biến tịnh tiến.
• Nhẩm một nghiệm ( x0 ; y0 ).
(
u = x + x0
• Sử dụng phép đổi biến Biến đổi đưa về hệ trên.
v = y + y0 .

(a 1 x + b 1 )( c 1 x2 + d 1 x y + e 1 y2 ) = f 1
(
4 Hệ có dạng (V I )
(a 2 x + b 2 )( c 2 x2 + d 2 x y + e 2 y2 ) = f 2 .
Phương pháp giải

ÓNguyễn Thành Nhân 121 Chuyên Hùng Vương-BD


Các Bài Giảng 2023-2024 Đại Số 10 Chuyên

• Xét y = 0.
• Với y ̸= 0 thì đặt x = t y rồi rút ra phương trình bậc ba theo ẩn t.

VÍ DỤ 14. Giải các hệ phương trình

x 2 − 2 x y + 3 y2 = 9
( 2
3 x + 2 x y + y2 = 11
(
1 2
2 x2 − 13 x y + 15 y2 = 0. x2 + 2 x y + 3 y2 = 17.

14 x2 − 21 y2 + 22 x − 39 y = 0 x ( x 2 − x y + y2 ) = 6 y
( (
3 4
35 x2 + 28 y2 + 111 x − 10 y = 0. 2 y( x2 + x y + y2 ) = 7 x.

( x − y)( x2 + y2 ) = 13 x 2 + y2 = 2
( (
5 6
( x + y)( x2 − y2 ) = 25. 5 x2 y − 4 x y2 + 3 y3 − 2( x + y) = 0.

VÍ DỤ 15. Giải các hệ phương trình

x2 + 2 x y + y2 = 4
( 2
3 x + 5 x y − 4 y2 = 38
(
1 2
− x 2 + x y + 2 y2 = 4 . 5 x2 − 9 x y − 3 y2 = 15.
LATEX:Nguyễn Thành Nhân

( 2
3 x + 5 x y − 4 y2 = 38 ( x − y)( x2 − y2 ) = 3
(
3 4
5 x2 − 9 x y − 3 y2 = 15. ( x + y)( x2 + y2 ) = 15.

x2 − x y + y2 = 3( x − y) x3 − 8 x = y3 + 2 y
( (
5 6
x2 + x y + y2 = 7( x − y)2 . x2 − 3 = 3( y2 + 1).

BÀI 4. HỆ PHƯƠNG TRÌNH BẬC CAO KHÔNG MẪU MỰC


Nguyễn Thành Nhân- Chuyên Hùng Vương- Bình Dương

A. CƠ SỞ LÝ THUYẾT
1. Kỹ thuật sử dụng phép thế
Ta sử dụng phép thế dựa vào một trong các kỹ thuật sau đây
• Hệ gồm hai phương trình, trong đó có một phương trình có thể rút được một ẩn theo ẩn kia.

• Kết hợp cả hai phương trình bằng biến đổi đại số đưa về một phương trình đơn giản.

• Từ một phương trình tính biểu thức f ( x; y) rồi thế vào phương trình thứ hai.

• Có thể thế hằng số vào một phương trình nào đó.

VÍ DỤ 1. Giải các hệ phương trình


( (
x+ y−1 = 0 xy = x + 7y + 1
1 3 2 2 2 2
2
3 x + x (7 y − 3) + x (2 y − 7 y) = 2 y . x y2 = 10 y2 − 1.

x3 − 2 x y + 5 y = 7
( 3
2 x + y( x + 1) = 4 x2
(
3 4
3 x2 − 2 x + y = 3. 5 x 4 − 4 x 6 = y2 .

x4 + 2 x3 y + x2 y2 = 2 x + 9
( ( 2
(2 x + y)( x + y) + x(2 x + 1) = 7 − 2 y
5 (B − 08) 6
x2 + 2 x y = 6 x + 6. x(4 x + 1) = 7 − 2 y.

x3 + 7 y = ( x + y)2 + x2 y + 7 x + 4 x2 ( y + 1) = 6 y − 2
( (
7 8
3 x2 + y2 + 8 y + 4 = 8 x. x4 y2 + 2 x2 y2 + y( x2 + 1) = 12 y2 − 1.

ÓNguyễn Thành Nhân 122 Chuyên Hùng Vương-BD


Các Bài Giảng 2023-2024 Đại Số 10 Chuyên

VÍ DỤ 2. Giải các hệ phương trình

x3 + 2 x y2 = 5 x2 + x y + 2 x + 2 y = 16
( (
1 2
2 x2 + x y + y2 = 4 x + y. ( x + y)(4 + x y) = 32.
( 2
5 x y − 4 x y2 + 3 y3 − 2( x + y) = 0 y2 = (5 x + 4)(4 − x)
(
3 4
x y( x2 + y2 ) + 2 = ( x + y)2 . y2 − 5 x − 4 x y + 16 x − 8 y + 16 = 0.

2. Kỹ thuật biến đổi tương đương


Ta sử dụng kỹ thuật biến đổi tương đương bằng các biến đổi căn bản

• Cộng, trừ vế theo vế từng phương trình.

• Nhân, chia hai phương trình khi mà các biểu thức chia khác 0.

• Nhân thêm hai vế một phương trình với một biểu thức khác 0.

• Đưa về dạng bậc nhất hai ẩn một vế rồi khử.

VÍ DỤ 3. Giải các hệ phương trình


5

2 3 2
x + y + x y + x y + x y = − x3 (2 + 3 y) = 1
 (

TOÁN CHUYÊN

1
4 2
5 x( y3 − 2) = 3.
 x4 + y2 + x y(1 + 2 x) = − .


4
( 4
x − y4 = 240 x3 − 4 x y2 + 8 y3 = 1
(
3 4
x3 − 2 y3 = 3( x2 − 4 y2 ) − 4( x − 8 y). 2 x4 + 8 y4 = 2 x + y.

x2 + 2 = x( y − 1) 1 + x3 y3 = 19 x3
( (
5 6
y2 − 7 = y( x − 1). y + x y2 = −6 x2 .

x2 (1 − 2 y) = y2 (4 x + 2 y)
( 2
4 x + y4 − 4 x y3 = 1
(
7 8
2 x2 + x y − y2 = x. 4 x 2 + 2 y2 − 4 x y = 2 .

x3 − y3 = 35
(
VÍ DỤ 4. Giải hệ phương trình
2 x2 + 3 y2 = 4 x − 9 y.

3. Kỹ thuật đặt ẩn số phụ


Ta sử dụng kỹ thuật đặt ẩn số phụ theo các cách

• Đặt một ẩn ở một phương trình, đưa về phương trình đa thức.

• Đặt hai ẩn phụ ở một phương trình (áp dụng khi có một phương trình quá phức tạp hoặc là quá tối
giản).

• Đặt hai ẩn phụ ở hai phương trình, đưa về hệ mới.

VÍ DỤ 5. Giải các hệ phương trình


 q p
2 − x2 y4 + 2 x y2 − y4 + 1 = 2(3 − 2 − x) y2
( p
2 x − y = 1 + x( y + 1)

1 2
x3 − y2 = 7.
q
 x − y2 + x = 3.

ÓNguyễn Thành Nhân 123 Chuyên Hùng Vương-BD


Các Bài Giảng 2023-2024 Đại Số 10 Chuyên

 p 6x q
2 + 3 y + 3 x − y =
  5 y4 − x4 + 6 y2 = 2 x( y + 3 x)
3 y 4 p p
(5 y2 + x2 )2 + 2 x y( x2 + 5 y2 ) = 12 x y + 36.
 q p 

2 3 x + 3 x − y + 4 = 6 x + 3 y.

12 x y + 12( x2 + y2 ) + 9
y2 + x + x y − 6 y + 1 = 0
(
= 85
5 ( x + y)2 6

6 x( x + y) + 3 = 13( x + y). y3 x − 8 y2 + x2 y + x = 0.
s s
2 2
 x + xy+ y

 x2 + y2 (
x 2 + x y + y2 = 3 y − 1
+ = x+ y
7 3 2 8

 p
 x3 + x2 y = x2 − x + 1.
x 2 x y + 5 x + 3 = 4 x y − 5 x − 3.

x4 + 4 x2 + y2 = 2(2 y + 1)
(
9
y( x2 + 6) + 2 x2 = 23.

4. Kỹ thuật đưa về phương trình tích


Ta sử dụng kỹ thuật đưa về phương trình tích bằng một trong các cách
LATEX:Nguyễn Thành Nhân

• Sau khi biến đổi, một phương trình nào đó có dạng A 2 − B2 ; A 3 − B3 .

• Biến đổi, đổi biến đưa về được các dạng A + B = 1 + AB; Au + Bv = AB + uv.

• Xem một phương trình nào đó là bậc hai theo ẩn x hoặc ẩn y, tính được biệt thức là một bình phương
đủ ∆ = A ( y)2 hoặc ∆ = B( x)2 .
• Một phương trình có dạng đẳng cấp và có thể phân tích được thành tích.

VÍ DỤ 6. Giải các hệ phương trình


( 2
5 x y − 4 x y2 + 3 y3 − 2( x + y) = 0 y2 = (5 x + 4)(4 − x)
(
1 2
x y( x2 + y2 ) + 2 = ( x + y)2 . y2 − 5 x2 − 4 x y + 16 x − 8 y + 16 = 0.

x3 − 4 y3 = 6 x2 y − 9 x y2 x y + x + y = x 2 − 2 y2
( (
3 p p 4 p p
x + y + x − y = 2. x 2 y − y x − 1 = 2( x − y).
( 2
4 x + y4 = 4 − 4 x y2 x2 + 2 x + ( x y − 1)2 = 2 x2 y
(
5 6
x2 − 2( x y2 + 8) = − y4 . x3 y3 + 3 x y2 − 7 y3 = 1.
(p p  q
x − 5 + 2y − 4 = x − y + 1 3 y2 + 1 + 2( x + 1) y = 4 y x2 + 2 y + 1
7 8
8 y( x − 2) + 4 − 8 y = ( y − x)2 .
p
y( y − x) = 3 − 3 x.

 y + 3 = x2 − x − y − 3
p
y2 = ( x + 8)( x2 + 2)
(
9 10
y2 − (8 + 4 x) y − 5 x2 + 16 x + 16 = 0. 12 y = x + 3 − 2 4 y − x.
p
x

5. Kỹ thuật sử dụng tính đơn điệu của hàm số


Thông thường ta thường biến đổi đưa về có một phương trình dạng f ( u) = f (v), nếu chứng minh được f ( t) là
hàm đơn điệu thì suy ra u = v.
Để biến đổi đưa về f (u) = f (v), ta thường
• Biến đổi một phương trình.

• Phối hợp cả hai phươn trình.

• Đặt ẩn phụ rồi mới đưa về dạng f ( u) = f (v).

Sử dụng điều kiện f ( u) = f (v) theo các bước như sau

ÓNguyễn Thành Nhân 124 Chuyên Hùng Vương-BD


Các Bài Giảng 2023-2024 Đại Số 10 Chuyên

• Biến đổi đưa về được f ( u) = f (v).

• Xét hàm đại diện f ( t), chứng minh f ( t) là một hàm đơn điệu.

• Từ f ( t) đơn điệu mà f ( u) = f (v) ⇒ u = v.

VÍ DỤ 7. Giải các hệ phương trình

x(4 x2 + 1) + ( y − 3) 5 − 2 y = 0 x 3 − 3 x = y3 − 3 y
( p (
1 p 2
4 x 2 + y2 + 2 3 − 4 x = 7 . x6 + y6 = 1.

VÍ DỤ 8. Giải các hệ phương trình


( p p ( 2 p
(17 − 3 x) 5 − x + (3 y − 14) 4 − y = 0 ( x + 1) x + ( y − 4) 3 − y = 0
1 2 p
2 2 x + y + 5 + 3 3 x + 2 y + 11 = x2 + 6 x + 13. 22 x2 + 9 y2 + 18 4 − 3 x = 76.
p p

 2 2
 x ( x + y2 ) = y4 ( y2 + 1)
p
 x2 + 91 = y − 2 + y2
p
3 4
p p4 x + 5 + y2 + 8 = 6.
q q
y2 + 91 = x − 2 + x2 .

BÀI 5. PHƯƠNG TRÌNH-HỆ PHƯƠNG TRÌNH GIẢI BẰNG

TOÁN CHUYÊN
PHƯƠNG PHÁP ĐÁNH GIÁ
Nguyễn Thành Nhân- Chuyên Hùng Vương- Bình Dương

A. CƠ SỞ LÝ THUYẾT
1. Các tình huống thường gặp
Để giải một phương trình bằng cách đánh giá, ta có thể gặp một trong những tình huống sau đây

1 Phương trình có dạng


A 21 + A 22 + · · · + A 2n ≤ 0.
Khi đó dẫn đến A 1 = A 2 = · · · = A n = 0.

2 Phương trình f ( x) = g( x), trong đó f ( x) ≥ A và g( x) ≤ A . Khi đó dẫn đến f ( x) = g( x) = A .

2. Các bất đẳng thức thường sử dụng


1 Bất đẳng thức Cauchy.

2 Bất đẳng thức Cauchy-Schwarz.

3 Bất đẳng thức Minkowski dạng I


q q q q
a21 + b21 + a22 + b22 + · · · + a2n + b2n ≥ (a 1 + a 2 + · · · + a n )2 + ( b 1 + b 2 + · · · + b n )2 .

a1 a2 an
Dấu đẳng thức xảy ra khi và chỉ khi = = ··· = . Quy ước rằng nếu có b i = 0 thì dẫn đến a i = 0
b1 b2 bn
tương ứng.
Có thể nhìn bất đẳng thức Minkowski dưới góc độ hình học thì tổng độ dài các đoạn thẳng của một
đường gấp khúc lớn hơn hoặc bằng độ dài đoạn thẳng nối điểm đầu đến điểm cuối của đường gấp khúc
đó.
Vài trường hợp cụ thể
q q p
• a21 + b21 + a22 + b22 ≥
(a 1 + a 2 )2 + ( b 1 + b 2 )2 .
q q q p
• a21 + b21 + a22 + b22 + a23 + b23 ≥ (a 1 + a 2 + a 3 )2 + ( b 1 + b 2 + b 3 )2 .

ÓNguyễn Thành Nhân 125 Chuyên Hùng Vương-BD


Các Bài Giảng 2023-2024 Đại Số 10 Chuyên

4 Bất đẳng thức Minkowski dạng I I .


Với a 1 , a 2 , . . . , a n và b1 , b2 , . . . , b n là các số dương. Ta có
pn
p
n
p
n
a 1 a 2 · · · a n + b 1 b 2 · · · b n ≤ (a 1 + b 1 )(a 2 + b 2 ) · · · (a n + b n ).

B. CÁC BÀI TOÁN


1. Các bài toán phương trình
s
x2 − 2 x + 2
3
BÀI 1. Giải phương trình 3 + 2 x = 5.
2x − 1
p
BÀI 2. Giải phương trình x3 + x + 6 = 2( x + 1) 3 + 2 x − x2 .
BÀI 3. Giải phương trình p
3p 4
x2 − 3 x + 1 = − x + x2 + 1.
3
p
3 p ¢p
BÀI 4. Giải phương trình 12 x2 + 4 2 x − 3 = x2 + 16 x − 12 3 x − 1.
¡

BÀI 5. Giả sử đa thức f ( x) = x3 + ax2 + bx + c có ba nghiệm không âm. Tìm số thực a lớn nhất thỏa mãn
f ( x) ≥ a ( x − a)3 , ∀ x ≥ 0.
r
x−1 2x + 6
BÀI 6. Giải phương trình + = 2.
x + 1 ¡p x − 1 + p x + 3¢2
LATEX:Nguyễn Thành Nhân

BÀI 7. Giải phương trình


1 1 1 1 4
+ + + = với x > 0.
5 x2 − x + 3 5 x2 + x + 7 5 x2 + 3 x + 13 5 x2 + 5 x + 21 x2 + 6 x + 5
BÀI 8. Giải phương trình
p p
3 9− x
2x − 2 + x − 2 = p
3
·
8 x − 16
BÀI 9. Giải phương trình
1 1 1 1 1
p
3
+p
3
=p
3
+p
3
với x > .
x 3x + 1 2x − 1 2x + 2 2
BÀI 10. Giải phương trình sau trên tập số thực
q q
3 x2 + y2 − 2 x − 4 y + 5 + 2 5 x2 + 5 y2 + 10 x + 50 y + 130
q q
+ 5 x2 − 5 y2 − 30 x + 45 = 102 x2 + 102 y2 − 204 x + 204 y + 1360.

BÀI 11. Giải phương trình p


x2 + 28 x + 4 x+4
+8 = p + 2 x. (1)
x+2 x−1

2. Các bài toán hệ phương trình


BÀI 12. Giải hệ phương trình  ¡
2 2
¢ 14 13
2 y − x = x − y (1)

4 ¡ x2 + y2 ¢ = 14 + 13 .

 (2)
x y
BÀI 13. Giải hệ phương trình  p
x + 1 + x2
= x2 y (1)


 p
1 + 1 + y2
 1 + y4 − 4 y + 3 = 0.

(2)

x x4
BÀI 14. Giải hệ phương trình:
p 2 3

 x− p + p =1



 y z
3 1

p
y− p + p = 2


 z x

p 1 2
 z − p + p = 3.


x y

ÓNguyễn Thành Nhân 126 Chuyên Hùng Vương-BD


Các Bài Giảng 2023-2024 Đại Số 10 Chuyên

BÀI 15. Giải hệ phương trình


(
x 2 + y2 = 4 y + 1
x3 + ( y − 2)3 = 7.

BÀI 16. Giải hệ phương trình: ( 2


5 x + 2 y2 + z2 = 2 (1)
x y + yz + zx = 1. (2)

BÀI 17. Giải hệ phương trình ba ẩn số thực x, y, z:


 3

 x + 2 y3 = 2 x2 + z2

2 x 3 + 3 x 2 = 3 y3 + 2 z 2 + 7

 3
x + x2 + y2 + 2 x y = 2 xz + 2 yz + 2.

BÀI 18. Giải hệ phương trình


x 4 + 3 x = y4 + y
(

x2 − y2 = 2.

Nhận xét 11. Với hệ phương trình có chứa x2 − y2 ; x4 − y4 , việc đổi biến số như sau nhiều khi rất hữu ích: Đặt
x + y = a, x − y = b suy ra

a+b a−b a2 + b 2
✓ x= ;y= ; ✓ x 2 + y2 = ;
2 2 2

TOÁN CHUYÊN
µ 2
a + b2

✓ x2 − y2 = ab; ✓ x4 − y4 = ab .
2

BÀI 19. Giải hệ phương trình

 x2 + x y + y2 + y2 + yz + z2 + z2 + zx + x2 = p3( x + y + z)
q q p

p p p p
x yz − ( x + y + z) + 2 = 0.

BÀI 20. Giải hệ phương trình


q q q
 2( x2 + 4 y2 − 8) = ( y2 + x − 3 + 1 − y)( y2 + x − 3 + 1 + y)

(1)
q
2 4 y4 + 5 = x

(2)

BÀI 21. Giải hệ phương trình


q q q
 2( x2 + 4 y2 − 8) = ( y2 + x − 3 + 1 − y)( y2 + x − 3 + 1 + y)

(1)
q
2 4 y4 + 5 = x

(2)

BÀI 22. Giải hệ phương trình q


p
 4 x2 + 5 + 4 y2 + 5 = 6| x y|
 18 + 24 1
+ 2
2 x y + 2 y8 x4 = 16.

BÀI 23. Giải hệ phương trình


q q q
 2( x2 + 4 y2 − 8) = ( y2 + x − 3 + 1 − y)( y2 + x − 3 + 1 + y)

(1)
q
2 4 y4 + 5 = x

(2)

BÀI 24. Tìm tất cả các số thực dương x, y, z thỏa mãn hệ phương trình
2 2 2

 x + y + z + x yz = 4

1 1 1
µ ¶
 9 + 9 + 9 (1 + 2 x yz) = 9.

x y z

ÓNguyễn Thành Nhân 127 Chuyên Hùng Vương-BD


Các Bài Giảng 2023-2024 Đại Số 10 Chuyên

BÀI 25. Giải hệ phương trình sau trên tập số thực


1 1 1 1

 + + =

x y z x yz

 x + y + z = x yz + 8 ( x + y + z)3 .


27

BÀI 26. Giải hệ phương trình q


p
 4 x2 + 5 + 4 y2 + 5 = 6| x y|
1 2 1 2
+ +
2 x8 y4 + 2 y8 x4

= 16.

BÀI 27. Giải hệ phương trình ( 5


2 x − 2 x3 y − x2 y + 10 x3 + y2 − 5 y = 0 (1)
( x + 1) y − 5 = y − 3 x2 + x − 2.
p
(2)

BÀI 28. Giải hệ phương trình


 3
 x + x + 2 = 8 y3 − 6 x y + 2 y
q q
 x2 − 2 y + 2 + 2 · 4 x3 (5 − 4 y) = 2 y2 − x + 2.

yz ( y + z − x)

LATEX:Nguyễn Thành Nhân


 =a
x+ y+ z





 zx ( z + x − y)
BÀI 29. Cho hệ phương trình =b

 x+ y+ z

x y ( x + y − z)


=c



x+ y+ z
với a, b, c là các số dương.
1 Chứng minh rằng hệ luôn có nghiệm dương.

2 Giải hệ khi a = 2; b = 5; c = 10.


 q
 x − 1 = 4 9 + 12 y − 6 y2
BÀI 30. Giải hệ phương trình p
4
y − 1 = 9 + 12 x − 6 x2 .

BÀI 6.HỆ BA PHƯƠNG TRÌNH BA ẨN QUA CÁC ĐỀ OLYMPIC


Nguyễn Thành Nhân- Chuyên Hùng Vương- Bình Dương

A. CÁC DẠNG TOÁN

LỜI MỞ ĐẦU
Hệ ba phương trình ba ẩn là nội dung thuộc lĩnh vực Đại số, thường xuất hiện trong các đề thi chọn đội
tuyển học sinh giỏi các tỉnh, khu vực và đề thi học sinh giỏi quốc gia. Tuy xuất hiện với mức độ dễ hoặc
trung bình trong các đề thi VMO, nhưng vẫn gây ra những khó khăn nhất định cho một số lượng lớn thí
sinh, đặc biệt là các thí sinh thuộc các đội tốp dưới. Những khó khăn thí sinh thường gặp phải đó là sự kết
hợp qua lại giữa các phương trình, mối quan hệ giữa hệ phương trình và phương trình đa thức thông qua hệ
thức Viète, tính chất giải tích như tính đơn điệu của hàm số được cài cắm vào mà học sinh chưa phát hiện
ra được. Trong bài viết nhỏ này, tác giả sẽ đưa ra một số kỹ thuật để xử lý hệ ba phương trình ba ẩn.

1. Hệ hoán vị vòng quanh


a)
Ta xét hệ hoán vị vòng quanh ba ẩn dạng

 x = f ( y)
 (1)
y = f ( z) (2) (I)

z = f ( x) (3)

ÓNguyễn Thành Nhân 128 Chuyên Hùng Vương-BD


Các Bài Giảng 2023-2024 Đại Số 10 Chuyên

Hàm f ( t) xét ở trên gọi là hàm đặc trưng của hệ. Ta có các tính chất rất quan trọng sau đây.

1 Nếu hàm f có tập giá trị là A , khi đó x, y, z, f ( x), f ( f ( x)), f ( f ( f ( x))) cũng thuộc A .

2 Nếu f ( t) đồng biến trên A thì ta có x = f ( x). Và dĩ nhiên ta cũng có y = f ( y), z = f ( z).
Chứng minh:
Từ hệ ta có x = f ( f ( f ( x))). Do đó
+ Nếu x > f ( x) thì do f đồng biến nên ta suy ra

x > f ( x) > f ( f ( x)) ⇒ x > f ( x) f ( f ( x)) > f ( f ( f ( x))) = x (vô lý).

+ Nếu x < f ( x), từ tính đồng biến của f ta suy ra

x < f ( x) < f ( f ( x)) < f ( f ( f ( x))) = x (vô lý).

Chứng minh:
Lập luận tương tự như khi f đồng biến.

3 Nếu f ( t) đơn điệu thì x = y = z.


Chứng minh:
Ta chứng minh khẳng định trên với hàm f ( t) đồng biến. Trường hợp nghịch biến được chứng minh
tương tự.
Không mất tính tổng quát ta giả sử x = max{ x; y; z}. Khi đó ta có x ≥ y, x ≥ z.
Ta có
z = f ( x) ≥ f ( y) = x ⇒ z = x.

Từ đó suy ra

TOÁN CHUYÊN
f ( z) = f ( x) ⇒ y = z.

Từ đó dẫn đến x = y = z.

b)
Ta xét hệ hoán vị vòng quanh dạng

 f ( x) = g( y)

f ( y) = g ( z ) (II)

f ( z ) = g ( x)

các hàm f ( t) và g( t) đồng biến trên tập giá trị A của nó. Với hệ có cấu trúc như thế, ta chứng minh x = y = z.
Không mất tính tổng quát ta giả sử x = max{ x; y; z}, thế thì x ≥ y và x ≥ z. Ta có

x ≥ y ≥ 1 ⇒ f ( x ) ≥ f ( y) ⇒ g ( y) ≥ g ( z ) ⇒ y ≥ z
⇒ f ( y) ≥ f ( z) ⇒ g( z) ≥ g( x) ⇒ z ≥ x.

Từ đó suy ra x = z.
Với x = z ta suy ra
f ( x) = f ( z) ⇒ g( y) = g( x) ⇒ y = x.

Vậy ta có x = y = z.

BÀI 1 (Chọn đội dự tuyển 10-Chuyên Hùng Vương).


Tìm x, y, z nguyên thỏa mãn hệ phương trình
 3 2
 x − 4 x − 16 x + 60 = y


y3 − 4 y2 − 16 y + 60 = z .

 3
z − 4 z2 − 16 z + 60 = x

BÀI 2 (Gặp Gỡ Toán Học 2010). Giải hệ phương trình


3 2

 x = y − 7 y + 17 y − 12


(1)
y = z3 − 7 z2 + 17 z − 12 (2) .

z = x3 − 7 z2 + 17 z − 12

(3)

ÓNguyễn Thành Nhân 129 Chuyên Hùng Vương-BD


Các Bài Giảng 2023-2024 Đại Số 10 Chuyên

BÀI 3 (Gặp Gỡ Toán Học 2012). Giải hệ phương trình

2 x = y3 − y2 + 2


 (1)

2 y = z3 − z2 + 2 (2) .

2 z = x3 − x2 + 2

(3)

BÀI 4 (Trường Đông Toán học miền Nam 2013).


Giải hệ phương trình
x( y + z ) = x2 + 2


 (1)

y( z + x) = y2 + 3 (2) .

z ( x + y) = z 2 + 4

(3)

BÀI 5 (Trường Đông Toán học miền Nam 2013).


Giải hệ phương trình
x2 ( x + 2) = 3( y3 − x) + 3


 (1)

y2 ( y + 2) = 3( z3 − y) + 3 (2) .

 2
z ( z + 2) = 3( x3 − z) + 3

(3)

BÀI 6 (VMO 2006-Bảng B). Giải hệ phương trình


LATEX:Nguyễn Thành Nhân

 3

 x + 3 x2 + 2 x − 5 = y (1)

y3 + 3 y2 + 2 y − 5 = z (2) .

 3
z + 3 z2 + 2 z − 5 = x

(3)

2. Sử dụng phép cộng đại số


BÀI 7 (VMO 2016). Giải hệ phương trình

6 x − y + z2 = 3


 (1)

x2 − y2 − 2 z = −1 (2) .

 2
6 x − 3 y2 − y − 2 z2 = 0

(3)

BÀI 8 (Phú Thọ TST). Giải hệ phương trình


 2 2
( x + y) − 1 = 15 z
 (1)
x + 2 = z ( x + y) (2) .


y− z = 1 (3)

3. Giải hệ dựa vào đánh giá


x + 3 y = x3 − 12




BÀI 9 (Chuyên KHTN TST 2010). Giải hệ phương trình − y + 4 z = y3 − 6

9 z + 2 x = z3 + 32.

BÀI 10 (45 năm THTT). Giải hệ phương trình


 5

 x − x4 + 2 x2 y = 2 (1)

y5 − y4 + 2 y2 z = 2 (2) .

 5
z − z4 + 2 z2 x = 2

(3)

BÀI 11 (Trại hè Phương Nam 2017). Xét hệ phương trình


 p p
 x − x = yz
 (1)
 p p
y − y = zx (2) .

 p p

z − z = xy (3)

a) Chứng minh rằng nếu x, y, z thỏa mãn hệ phương trình trên thì trong chúng có ít nhất một số bằng 0.

ÓNguyễn Thành Nhân 130 Chuyên Hùng Vương-BD


Các Bài Giảng 2023-2024 Đại Số 10 Chuyên

b) Giải hệ phương trình trên.

BÀI 12 (Olympic 30/4/2019- Khối 10). Giải hệ phương trình


 1

 x + = y2 + 1 (1)


 x
1


y + = z2 + 1 (2) .

 y

 z + 1 = x2 + 1


(3)

z

BÀI 13 (45 năm THTT). Giải hệ phương trình


 3 2
 y − 6 x + 12 x − 8 = 0


(1)
z3 − 6 y2 + 12 y − 8 = 0 (2) .

 3
x − 6 z2 + 12 z − 8 = 0

(3)

BÀI 14 (Middle European Mathematica Olympiad 2012).


Giải hệ phương trình
2 x3 + 1 = 3 zx


 (1)

2 y3 + 1 = 3 x y (2) .

 3

2 z + 1 = 3 yz (3)

4. Hệ có tham số

TOÁN CHUYÊN
BÀI 15 (VMO 2020- Ngày 2- Bài 5). Cho hệ phương trình với tham số a:

 x − a y = yz

y − az = zx ( x, y, z ∈ R).

z − ax = x y

a) Giải hệ khi a = 0.

b) Chứng minh rằng hệ trên có 5 nghiệm phân biệt với mọi số thực a > 1.
1
µ ¶
BÀI 16 (Đề đề nghị Duyên hải Bắc Bộ). Cho số thực a ∈ ; 1 . Giải hệ phương trình
2
 2

 ax + 1 = x y (1)

a y2 + 1 = yz (2) .

 2

az + 1 = zx (3)

BÀI 17 (Đề đề nghị Olympic 30/4/2009). Tùy theo tham số a, giải hệ phương trình

2 x( y2 + a2 ) = y( y2 + 9a2 )


 (1)

2 y( z 2 + a 2 ) = z ( z 2 + 9 a 2 ) (2) .

2 z ( x2 + a2 ) = x( x2 + 9 a2 )

(3)

B. BÀI TẬP RÈN LUYỆN


BÀI 18 (Nguyễn Tài Chung). Giải các hệ phương trình sau

x = 3 z3 − 2 z2



−1 −1 −1
 µ ¶
1 y = 3 x3 − 2 x2 ĐS: (0; 0; 0); ; ; , (1; 1; 1)
 3 3 3
z = 3 y3 − 2 y2 .


x y = z − x − y

2 xz = y − x − z ĐS: (0; 0; 0), (−1; −1; −1), (−2; −2; 0)

yz = x − y − z.

ÓNguyễn Thành Nhân 131 Chuyên Hùng Vương-BD


Các Bài Giảng 2023-2024 Đại Số 10 Chuyên

 3

 x − 3x = y

3 y3 − 3 y = z ĐS: (2; 2; 2)

 3

z − 3 z = 4 − x.
 2

 4 x − 11 x + 9 = y
3 3 3
 µ ¶
4 4 y2 − 10 y + 9 = 2 z ĐS: ; ;

 2
 2 2 2
4 z − 9 z + 9 = 3 x.
 2

 x + 2 yz = x
1 1 1 1 1 2
 µ ¶ µ ¶
5 y2 + 2 zx = y ĐS: ; ; , (0; 0; 0), (1; 0; 0) và các hoán vị; − ; − ; và các hoán vị.

 2
 3 3 3 3 3 3
z + 2 x y = z.
 2

 x y + x + z = y2 + 2

6 yz2 + y + x = z2 + 2 ĐS: (1; 1; 1)

 2
zx + z + y = x2 + 2.

 2
4 x − 11 x + 9 = y

3 3 3
 µ ¶
7 4 y2 − 10 y + 9 = 2 z ĐS: ; ;

 2
 2 2 2
LATEX:Nguyễn Thành Nhân

4 z − 9 z + 9 = 3 x.

x + y2 + z3 = 3


 (1)

8 y + z 2 + x3 = 3 (2) (với x > 0, y > 0, z > 0). ĐS: (1; 1; 1)

z + x2 + y3 = 3

(3)

BÀI 7. ÔN TẬP PHƯƠNG TRÌNH- HỆ PHƯƠNG TRÌNH


Nguyễn Thành Nhân- Chuyên Hùng Vương- Bình Dương

A. MỘT SỐ DẠNG TOÁN

1. Đặt ẩn số phụ cho biểu thức đồng dạng


BÀI 1. Giải phương trình p
( x2 + 1)2 = x x 2 + 2 + 3. (1)
p p p p
ĐS: − −1 + 2, x = − −1 + 5.
BÀI 2. Giải phương trình p
3
x2 + x 4 − x 2 = 2 x + 1. (2)
p
−1 ± 5
ĐS: x = .
2
BÀI 3. Giải bất phương trình p p
x x 2
p − p ≥
x− x+1 x+ x+1 3
ĐS: x = 1 là nghiệm duy nhất.
BÀI 4. Giải bất phương trình
³ p ´2 p
4 x + 1 − 2 x + 1 + 3 2 x + 1 ≥ 2 x + 4.

3
ĐS: ≥ .
2
BÀI 5. Giải bất phương trình q q
4 p p 9
x− x2 − 1 + x+ x2 − 1 ≤ .
2
257
ĐS: 1 ≤ x ≤ .
32

ÓNguyễn Thành Nhân 132 Chuyên Hùng Vương-BD


Các Bài Giảng 2023-2024 Đại Số 10 Chuyên

BÀI 6. Giải bất phương trình


p p
3 x − 1 − x2 − 6 x + 6 ≥ x.
5
ĐS: ≤ x ≤ 5.
4

2. Đặt ẩn phụ cho phương trình đối xứng


BÀI 7. Giải bất phương trình
1 1 2
p −p − x ≥ 1.
x+2 −x − 1 3
ĐS: −2 < x < −1.
BÀI 8. Giải phương trình p ³ p ´
8x 1 − x2 = 5 − 16 x + 1 − x2 .
p
1± 7
ĐS: x = .
4
BÀI 9. Giải phương trình
p p p
6 x + x + 2 + 2 3 − x = 8 6 + x − x2 .
ĐS: x = 2.

3. Đặt ẩn phụ cho phương trình đẳng cấp


BÀI 10. Giải bất phương trình

TOÁN CHUYÊN
¢p
2 x 3 ≤ 1 + 2 x − 3 x 2 2 x + 1.
¡

p
1 1+ 5
ĐS: − ≤ x ≤ .
2 4
BÀI 11. Giải bất phương trình
p p p
x5 + x3 + x ≤ ( x2 + 1)3 − x2 ( x2 − x + 1).

ĐS: x ≥ 0.
BÀI 12. Giải bất phương trình
p p p
x+ 1 − x2 ≥ 2 − 3 x − 4 x2 .
BÀI 13. Giải bất phương trình
p ¢¡ p
x2 ≥ 1 − x 2 x − 3 x + 3 .
¡ ¢

BÀI 14. Giải bất phương trình p


2( x2 + 2) ≥ 5 x3 + 1.
BÀI 15. Giải bất phương trình p
2( x2 + 2) ≥ 5 x3 + 1.

4. Đặt hai ẩn phụ đưa về phương trình tích


BÀI 16. Giải bất phương trình ³ p ´
( x + 1)2 ≤ 4 2 + x3 − 2 x2 − 3 x + 4 .

BÀI 17. Giải phương trình p


5 x3 + 1 = 2( x2 + 2).
BÀI 18. Giải phương trình p p
x2 + 3 x2 − 1 = x 4 − x 2 + 1.
BÀI 19. Giải phương trình p p
x2 + 3 x2 − 1 = x 4 − x 2 + 1.
BÀI 20. Giải phương trình p p p
5 x2 + 14 x + 9 − x2 − x − 20 = 5 x + 1.

ÓNguyễn Thành Nhân 133 Chuyên Hùng Vương-BD


Các Bài Giảng 2023-2024 Đại Số 10 Chuyên

BÀI 21. Giải bất phương trình p ³ ´


3
p
14 − x3 + x ≥ 2 1 + x2 − 2 x − 1 .

BÀI 22. Giải phương trình


p
3
p
3 p
3
p
3
x+1+ x2 = x+ x2 + x.
BÀI 23. Giải phương trình p
3x x3 + 1 = x3 + x2 − 19 x − 16.
BÀI 24. Giải bất phương trình ³p p ´ p
x−1 ≥ x x − 1 − x + x2 − x.

5. Sử dụng biểu thức liên hợp


BÀI 25. Giải phương trình p p
4 x 2 + 5 x + 1 − 2 x 2 − x + 1 = 9 x − 3.
BÀI 26. Giải bất phương trình
p p 6x − 4
2x + 4 − 2 2 − x ≥ p .
x2 + 4
BÀI 27. Giải bất phương trình p
LATEX:Nguyễn Thành Nhân

x+1 1
p p > x− .
x+1− 3− x 2
BÀI 28. Giải bất phương trình
p p p
x − 1 + x + 3 + 2 x3 − 4 x2 + 8 x − 5 ≥ 2 x.
BÀI 29. Giải các phương trình và bất phương trình
p p 1
1 3 x2 − 14 x − 8 + 3 x + 1 − 6 − x ≤ 0 ĐS: − ≤ x ≤ 5.
3
p p
2 ( x + 1) x + 2 + ( x + 6) x + 7 ≥ x2 + 7 x + 12. ĐS: −2 ≤ x ≤ 2.
p p
3 4 x + 1 + 2 2 x + 3 ≤ ( x − 1)( x2 − 2). ĐS: x = −1, x ≥ 3.

ÓNguyễn Thành Nhân 134 Chuyên Hùng Vương-BD

You might also like